Wajo 1

You might also like

Download as pdf or txt
Download as pdf or txt
You are on page 1of 192

Western Australian

Junior Mathematics Olympiad 2000


Individual Questions 100 minutes
General instructions: No working need be given for Questions 1 to 9. Calculators are not
permitted. Write your answers on the answer sheet provided.

(1) How many non-overlapping equilateral triangles with 1 centimetre sides can we fit inside
an equilateral triangle with 10 centimetre sides? [1 mark]

(2) Four accused people face trial. It is known that:


(a) If A is guilty, then B is guilty,
(b) If B is guilty, then C is guilty or A is not guilty,
(c) If D is not guilty, then A is guilty and C is not guilty,
(d) If D is guilty, then A is guilty.
How many of the accused must be guilty? [1 mark]

(3) Find the number of integers between 100 and 500 such that the sum of their digits is
10. [1 mark]

(4) In the expression S = a − b + c − d the symbols a, b, c, d are replaced by 1, 2, 3, 4 in any


order with no repetitions allowed. There are 24 possible replacements.
In how many of these will S be greater than 0? [1 mark]

(5) Let ABC be an acute angled triangle. Let M be a point on BC such that AM is
perpendicular to BC. Let N be a point on AB such that CN is perpendicular to AB.
If H is the intersection point of AM and CN and it is given that HM = HN and
BC = 20, find AB. [2 marks]

(6) The visibility at sea, on a certain day, is 5 kilometres. Ships A and B (which start a
long way apart) are travelling in opposite directions on courses which are parallel and
3 kilometres apart. The two ships are in sight of one another for 24 minutes.
If ship A is travelling at 8 kilometres per hour, how fast is ship B travelling? [2 marks]

(7) How many integers between 100 and 1000 are there that are not exactly divisible by 2,
3 or 5? [2 marks]

(8) The number 6 is divisible by 1, 2, 3 and 6; so 6 has 4 divisors.


How many divisors has 6718464 = 210 × 38 ? [2 marks]

(9) Let M be the point on the extension of the side BC of a parallelogram ABCD such
that B is between M and C and M B = BC.
If P and N are the intersection points of M D with AC and AB, respectively, and
P N = 3, find DP . [2 marks]

(10) Ten students have altogether 35 coins. It turns out that at least one of them has exactly
one coin, at least one has exactly two coins, and at least one has exactly three coins.
Explain why you can be sure that at least one student has 5 or more coins. [4 marks]
Western Australian
Junior Mathematics Olympiad 2000
Team Questions 45 minutes

(1) A cube, consisting of 125 cubes each with side 1 centimetre, is drilled through in three
places. The holes are rectangular-shaped with cross-section 1 centimetre by 3 centime-
tres (see picture) and go all the way through the large cube. How many small cubes
remain after the drilling?
(2) Instead of a 5 × 5 × 5 cube, say you have a 7 × 7 × 7 cube, with three slots having
cross-section 1 centimetre by 5 centimetre drilled through the middle. How many small
cubes remain?
(3) Now suppose you have an n × n × n cube, with n an odd number. Three slots, each with
cross-section 1 centimetre by n − 2 centimetres are drilled through the middle. Find a
formula for the number of small cubes that remain.
(4) Briefly explain how you obtained your answer to question 3.

Write your answers on the accompanying sheet.


Western Australian Junior Mathematics Olympiad
October 28, 2000

Problem Solutions

1. The answer is 100. You can see this by drawing a diagram. Alternatively, using
Pythagoras’
√ Theorem or trigonometry, you can show
√ that the area of the large triangle
100 3 3
is and the area of each small triangle is . The answer follows since
4 4

100 3
√4 = 100 .
3
4

2. Looking at (c) and (d) together we see that A must be guilty regardless of D’s guilt
or innocence. Part (a) then implies that B is guilty. So now we know that both A
and B are guilty. Part (b) says that C is guilty or A is innocent, but we know A is
not innocent, so C must be guilty. Now we know A, B and C are all guilty. Part (c)
says that if D were not guilty then C would be not guilty, which we know is not true.
So D must be guilty as well and the answer is 4.

3. We put the numbers in blocks. We have 10 numbers with first digit being 1: 109,
118, 127,..., 190, then 9 with first digit being 2: 208, 217,..., 280, then 8 with first
digit being 3, and 4 with first digit being 4. The total number of integers is then 10
+ 9 + 8 + 7 = 34.

4. We could do this by writing down all 24 possibities, but this is a bit tedious. A
smarter way is to notice that the sum can only equal 0 in 8 ways: the numbers with
a plus sign being 1 and 5 and the numbers with a minus sign being 2 and 3 (which
can happen in 4 ways) or the other way round (another 4 ways). This leaves 24 - 8
= 16 non-zero sums. There must be an equal number of positive and negative sums,
since swapping the values of a and b, and the values of c and d will change the sum
from positive to negative or vice versa. Therefore the number of positive solutions is
16 / 2 = 8.

5. In triangles CM H and AN H we have M H = N H, 6 CHM = 6 AHN (opposite


angles) and 6 CM H = 6 AN H = 90◦ . Thus triangles CM H and AN H are congruent
by the Angle-Side-Angle rule. Thus CH = AH and so CN = AM . Angles AM B
and CN B both equal 90◦ and angles BCN and BAM both equal 90◦ - 6 ABC. So
triangles CN B and AM B are congruent (Angle-Side-Angle again), and therefore
AB = CB = 20.

1
6. The answer is 12 km per hour. Indeed, at a certain time the ships will be 5 km
apart for the first time. Denote by A0 and B0 their respective positions at that
time, and let C0 be the point on the line determined by the direction of the ship A
such that B0 C0 is perpendicular to this line. By Pythagoras’ Theorem, A0 C0 = 4
km. If A0 and B 0 are the positions of the ships A and B after 24 minutes, then the
distance travelled by A is A0 A0 = 8 × 2460
= 16
5
km. Hence A0 C0 = 4 − 16 5
= 45 . If
D0 is the point on the line determined by the direction of the ship A such that B 0 D0
is perpendicular to this line, then by Pythagoras’ Theorem for 4B 0 D0 A0 one gets
D0 A0 = 4, so B 0 B0 = D0 C0 = D0 A0 + A0 C0 = 4 + 45 = 24
5
km. Therefore the speed of
24 24
ship B is 5 : 60 = 12 km.

7. The answer is 240. Clearly 100 is divisible by 2 (and 5), so we have to check the
integers 101, 102, . . . , 999, 1000. Consider the first 30 of them: 101, 102, 103, . . . , 130.
By a direct inspection, one can see that exactly 8 of them are not divisible by 2, 3 and
5. Dividing the sequence 101, 102, . . . , 1000 into 30 separate sets of 30 consecutive
integers and using the fact that in each of these sets of 30 integers there will be
again exactly 8 integers not divisible by 2, 3 and 5, one gets that the total number
of integers between 101 and 1000 not divisible by 2, 3 and 5 is 30 × 8 = 240.

8. The answer is 99. The divisors of 210 are 1 = 20 , 2 = 21 , 22 , . . ., 210 , while these of
38 are 1 = 30 , 3 = 31 , . . ., 38 . Every divisor of 210 × 38 has the form 2k × 3m for
some k = 0, 1, . . . , 10 and m = 0, 1, . . . , 8. So, there are 11 different ways to choose
k and 9 different ways for m. Altogether the number of ways to choose k and m is
11 × 9 = 99. Thus, there are 99 different divisors of 210 × 38 .

9. The answer is 6. Since 4M BN ∼ 4M CD (BN k CD) and M C = 2M B, it follows


that CD = 2BN . This and AB = CD gives AN = BN , so CD = 2AN . On the
other hand, 4AN P ∼ 4CDP (AN k CD), so N P
DP
= AN
CD
= 12 . This and N P = 3
imply DP = 6.

10. Removing one person that has exactly one coin, one person that has exactly two coins
and one person that has exactly three coins from the group of ten, we get a group of
seven students that altogether have 35 − 1 − 2 − 3 = 29 coins. If everyone of these 7
students has 4 coins or less, then altogether they would have ≤ 7 × 4 = 28 coins. So,
at least one of these 7 students must have 5 coins or more.

2
Team Questions

1. The answer is 88. The number of cubes removed is 15 + 12 + 10 = 37 (see the solution
of problem 3 below) so the number of cubes that remain is 125 − 37 = 88.

2. The answer is 252. The number of cubes removed is 35 + 30 + 26 = 91 (see the


solution of problem 3 below) so the number of cubes that remain is 343 − 91 = 252.

3. The answer is n3 − 3n2 + 9n − 7. The number of small (i.e. of size 1 × 1 × 1) cubes


contained in the vertical slot removed from the initial cube is n × (n − 2). Consider
one of the horizontal slots removed from the cube. It contains n(n − 2) small cubes,
however n − 2 of them have already been counted in the removal of the vertical slot.
So, the number of additional small cubes removed by removing the horizontal slot
is n(n − 2) − (n − 2). Finally, consider the other horizontal slot. Again it contains
n(n − 2) small cubes. However n − 2 of them are contained in the vertical slot,
while another n − 3 are contained in the first horizontal slot removed (and not in
the vertical one). So, by removing the second horizontal slot we remove an extra
n(n − 2) − (n − 2) − (n − 3) small cubes. Thus, the total number of small cubes
removed is

n(n−2)+[n(n−2)−(n−2)]+[n(n−2)−(n−2)−(n−3)] = 3n(n−2)−3n+7 = 3n2 −9n+7 .

Hence the number of small cubes remaining is

n3 − [3n2 − 9n + 7] = n3 − 3n2 + 9n − 7 .

3
Western Australian Junior Mathematics Olympiad
October 27, 2001

Individual Questions

General instructions: No working need be given for Questions 1 to 9. Calculators are


not permitted. For Questions 1 to 9, write the answer in the answer grid. Write your
answer to Question 10 in the space provided.

1. N degrees Celsius is the same temperature as 95 N + 32 degrees Fahrenheit. What


temperatures have the same measure on both scales? (1 mark)

2. bxc means the greatest integer which is not more than x, so that b4.9c = 4 and
b7c = 7, and dxe means the least integer which is not less than x, so that d4.9e =
5 and d7e = 7. We call bxc the floor of x and dxe the ceiling of x. Evaluate
d13.5 + 2.7 × b3.8ce. (1 mark)

3. A certain number of points are marked on the circumference of length 2001 of a circle
in such a way that each marked point is distance 1 from exactly one marked point
and distance 2 from exactly one marked point, all distances being measured around
the circle. How many points are there? (1 mark)

4. Let ABC be a right-angled triangle with 6 ACB = 90 degrees, and let AL be the
bisector of angle BAC, so that L is a point on BC. Let M be the point on AB such
that LM is perpendicular to AB. If LM = 3 and M B = 4, find AB. (2 marks)

5. How many solution pairs x, y) are there of the equation 2x + 3y = 763 if both x and
y are positive integers? (2 marks)

6. In a computer game, you have to score the largest possible number of points. You
score 7 points each time you find a jewel and 4 points each time you find a sword.
There is no limit to the number of points you can score. Of course it is impossible to
score 5 or 6 points. What is the largest number of points it is impossible to score?
(2 marks)

7. Find the least possible value of the expression x2 − 8xy + 19y 2 − 6y + 10. (3 marks)

8. A shop sells hamburgers which contain some of the following: meat burger, vegetable
burger, lettuce, tomato, carrot, mayonnaise and tomato sauce.
(a) You must have a meat burger or a vegetable burger, but cant have both.
(b) You can also have any number of the other ingredients, even none, but:
(c) If you have a meat burger you can also have tomato sauce, but not if you have a

1
vegetable burger.
(d) If you have lettuce or tomato or both you can have mayonnaise, but not otherwise.

How many different hamburgers can be constructed according to these rules?


(3 marks)

9. Let ABCD be a trapezium with AB parallel to CD, AB = 2CD and the diagonal
BD = 72 cm. If N is the midpoint of AB and M and P are the intersection points
of BD with N C and AC, respectively, find M P . (3 marks)

10. Two buses start travelling at the same time – bus 1 from city A to city B, and bus 2
from city B to city A using the same road. Both buses travel with constant speeds.
For the first time they meet 7km from A. After both buses reach their destinations
(cities B and A respectively, possibly at different times), they immediately start
travelling back along the same road and with the same speeds. They meet again 4
km from B. Find the distance between the cities A and B. Explain how you obtained
your answer. (4 marks)

Team Questions
1. Find six consecutive positive integers whose sum is 513.

2. Find a set of at least two consecutive positive integers whose sum is 30.

3. There are three possible solutions to question 2. Can you find them all?

4. Find a set of at least two consecutive positive integers whose sum is 56.

5. Show how any odd integer can be written as the sum of at least 2 consecutive integers.

6. Some positive integers cant be written in this way. What are they?

7. Can you prove your answer to question 6?

2
Western Australian Junior Mathematics Olympiad
October 27, 2001
Problem Solutions
1. We must solve 9N/5 + 32 = N . So 9N + 160 = 5N and hence 4N = −160. The solution to
this is N = −40.
2. d13.5 + 2.7 × b3.8ce = d13.5 + 2.7 × 3e = d13.5 + 8.1e = d21.6e = 22.
3. The gaps between adjacent points must be alternately 1 unit and 2 units, so any pair of
consecutive gaps totals 3 units. Since there must be 667 pairs of gaps, and so 1334 gaps
altogether and therefore 1334 points.
4. From 4LM B one finds LB 2 = 32 + 42 = 25, so LB = 5. Next, 4AM L ∼ = 4ACL (AL =
6 6 6 ◦ 6
AL, M AL = CAL, AM L = 90 = ACL), so CL = LM = 3. Now observe that
BC MB
4ABC ∼ 4LBM (6 ABC = 6 LBM , 6 ACB = 6 LM B), therefore = . This gives
AB LB
BC × LB 8×5
AB = = = 10.
MB 4
5. It’s clear that y must be odd so we can write y = 2Y + 1 for some non-negative integer Y .
Also 763 − 2x must be divisible by 3. Now 763 − 2x = (3 × 254) + (1 − 2x) so 1 − 2x must
be divisible by 3. This means x has the form 3X + 2 with X a non-negative integer. Thus
2(3X + 2) + 3(2Y + 1) = 763, which simplifies to X + Y = 126. Then X can be any integer
from 0 to 126, and so there are 127 solutions.
6. The answer is 17. By trial and error we find that 17 can’t be expressed as the sum of a multiple
of 7 plus a multiple of 4. However 18 = 2 × 7 + 4, 19 = 7 + 3 × 4, 20 = 5 × 4 and 21 = 3 × 7.
After this we can get 22 by adding 4 onto 18, 23 by adding 4 onto 19 and so on.
7. We note that:
x2 − 8xy + 19y 2 − 6y + 10 = x2 − 8xy + 16y 2 + 3(y 2 − 2y + 1) + 7 = (x − 4y)2 + 3(y − 1)2 + 7 .
Each of the squared terms is at least 0, so the whole expression must be at least 7, and we can
get 7 if we set y = 1 and x = 4. So the answer is 7.
8. We have 3 basic types of burgers: vegetable, meat with sauce or meat without sauce. Each
of these is accompanied by one of the following 7 lettuce, tomato, mayonnaise combinations:
LTM, LM, TM, LT, L, T, none of these. This gives 21 possibilities. Each of these 21 can be
served with or without carrot, giving a total of 42 possibilities.
DM
9. First, notice that 4N BM ∼ 4CDM (6 N M B = 6 CM D, 6 N BM = 6 CDM ). Hence =
MB
CD
= 1, i.e. DM = M B = 36 cm. Next, we have 4ABP ∼ 4CDP (6 AP B = 6 CP D,
NB
DP DC 1
6 ABP = 6 CDP ), so = = . That is, DP = 13 DB = 24 cm. Hence M P =
PB AB 2
DM − DP = 12 cm.
10. Let v1 and v2 be the speeds of bus 1 and bus 2 respectively, let t1 and t2 be the times at which
they pass each other and let x be the distance between the towns. By considering the first time
7 x−7 v2 x−7
they pass we see that = v1 , = v2 , which implies that = . By considering
t1 t1 v1 7
x+4 2x − 4 v2 2x − 4
the second time they pass we get = v1 , = v2 , which implies that = .
t2 t2 v1 x+4
x−7 2x − 4
Thus we have = , which gives x2 = 17x and so x = 17.
7 x+4

1
Solutions to Team Questions

1. 513 = 83 + 84 + 85 + 86 + 87 + 88.

2. 30 = 9 + 10 + 11 = 6 + 7 + 8 + 9 = 4 + 5 + 6 + 7 + 8.

3. See 2. above.

4. 56 = 5 + 6 + 7 + 8 + 9 + 10 + 11.

5. Any odd number can be written as 2n + 1 for some integer n. But 2n + 1 = n + (n + 1) which
is the sum of two consecutive integers.

6. Powers of 2 (including 1 = 20 ).

7. If a number is not a power of 2 then it has an odd factor greater than 1. So, say our number
is n = ab, where a = 2k + 1 is odd. Then

n = (b − k) + (b − k + 1) + (b − k + 2) + . . . + b + (b + 1) + . . . + (b + k − 1) + (b + k) ,

which has the required form. So, anything that is not a power of 2 can be written in the
required way.
If n can be written in the required form there must be positive integers a and m such that

n = a + (a + 1) + . . . + (a + m)
1
= (m + 1) a + (1 + 2 + . . . + m) = (m + 1) a + m (m + 1)
2
1
= (m + 1)(m + 2a) .
2
One of m + 1 and m + 2a must be odd and the other even, so n has an odd factor. This means
n is not a power of 2.

2
WA Junior Mathematics Olympiad 2002
Individual Questions 100 minutes (one hour and 40 minutes)

General instructions: No working need be given for Questions 1 to 9. Calculators are not
permitted. For Questions 1 to 9, write the answer in the answer grid. Write your answer to
Question 10 in the space provided.

(1 + 24 + 25 )(1 + 23 + 24 ) + 26
p
1. Simplify √ . [1 mark]
1 + 23
2. Four different positive integer numbers a, b, c, d satisfy the following relations:
1 1 1 1 1 1 1 1 1
+ + =1 , + + =1 , + + =1.
a a a a b c b d d
Find d. [2 marks]
3. Three athletes Ahmad, Bill and Claire are preparing to take part in a high jump competition.
At the same time some of the spectators are discussing their chances:
spectator X : ‘I think Ahmad will be first’,
spectator Y: ‘I am sure that Claire will not be the last’,
spectator Z : ‘Bill will not take first place’.
After the competition it turned out that only one of the spectators was right, while the
other two were wrong. Where did Claire finish? [2 marks]
4. In a rectangle ABCD, O is the intersection point of the diagonals AC and BD and BD = 10
cm. Find the length of BC if it is known that the point D lies on the perpendicular bisector
of the segment AO. [2 marks]
5. Three positive numbers are given such that :
(i) the first of the numbers is half the second;
(ii) the product of the first and the second number is equal to the sum of the second and
the third number;
(iii) the third number is three times as large as the second.
Find the first of the given numbers. [2 marks]
6. Find the smallest positive integer divisible by 15 whose every digit is 0 or 1. [2 marks]
7. A rectangle ABCD has sides AB = CD = 34 cm. E is a point on CD such that CE = 9
cm, ED = 25 cm, and 6 AEB = 90◦ . What is the length of BC? [3 marks]
8. I have 6 cats, two white, 2 black and 2 orange, with a male and female of each colour. I
want to put them in a row of 6 boxes. The orange cats are friends and have to be put side
by side, but the black cats fight and must not be side by side. In how many ways can I
arrange the cats? [3 marks]
9. Five different integer numbers a, b, c, d, e (not necessarily positive) are such that
(4 − a)(4 − b)(4 − c)(4 − d)(4 − e) = 12 .
Find the sum a + b + c + d + e. [4 marks]
10. All faces of a cube are divided into four equal squares and each small square is painted red,
blue or green in such a way that any two small squares that have a common side are painted
in different colours. Is it true that the numbers of red, blue and green squares must be the
same? Give reasons for your answer. [4 marks]

1
Western Australian
Junior Mathematics Olympiad 2002

Team Questions 45 minutes

1. You are given 9 coins of the same denomination, and you know that one of them is
counterfeit and that it is lighter than the others. You have a pan balance which means
you can put any number of coins on each side and the balance will tell you which side
is heavier, but not how much heavier. Explain how you can find the counterfeit coin
in exactly two weighings. [4 marks]

2. If you are given 25 coins of the same denomination, and you know that one of them is
counterfeit and that it is lighter than the others, explain how to find the counterfeit
coin by using at most 3 weighings on the pan balance. [8 marks]

3. It is known that there is one counterfeit coin in a collection of 70 and that it is lighter
than the others. What is the least number of weight trials on a pan balance necessary
to identify the counterfeit coin? Explain how you obtained your answer. [12 marks]

4. It is known that there is one counterfeit coin in a collection of 9 and it is known that
its weight is different from that of a genuine coin, however it is not known whether the
counterfeit coin is lighter or heavier than a genuine one. Show that by using at most
3 weighings in the pan balance you can identify the counterfeit coin and determine
whether it is lighter or heavier than a genuine coin. [16 marks]

1
WA Junior Mathematics Olympiad 2002
Solutions to the Individual Questions
1. Since 1 + 24 + 25 = 49, 1 + 23 + 24 = 25 and 26 = 64, it follows that

(1 + 24 + 25 )(1 + 23 + 24 ) + 26
p
49 · 25 + 64 99
√ = = = 33 .
1+2 3 3 3

Answer: 33
1 1 2
2. The first equation gives 3/a = 1, so a = 3, and the second equation becomes + = .
b c 3
Hence b ≥ 2 and c ≥ 2. If both b > 2 and c > 2, then (since the numbers are different and
a = 3) b ≥ 4 and c ≥ 4, so 1/b + 1/c ≤ 1/2, impossible. Thus, either b = 2 or c = 2. If
c = 2, then b = 6 and the last equation becomes 2/d = 5/6, impossible since d is an integer.
Thus b = 2, c = 6, and then the 3rd equation gives 2/d = 1/2, so d = 4.
Answer: 4

3. There are three possible cases to consider:


Case 1. X is right, while Y and Z are wrong. Then Ahmad must be first, Claire is last and
Bill is first, impossible.
Case 2. Y is right, while X and Z are wrong. Then Claire is not last, Ahmad is not first
and Bill is first. Hence Claire is second and Ahmad must be last. This case is possible.
Case 3. Z is right, while X and Y are wrong. Then Bill is not first, Ahmad is not first and
Claire is last, impossible.

Thus, only the second case is possible, so Claire must have taken second place.
Answer: 2

4. If M is the midpoint of AO, then 4AM D ∼


= 4OM D (AM = M O, M D = M D, 6 AM D =
6 OM D). Hence AD = OD = BD/2 = 5 cm, and therefore BC = 5 cm.

Answer: 5

5. If x, y, z are the given numbers, we have x = y/2, xy = y + z, z = 3y. Substituting x and


z in the second equation gives y 2 /2 = y + 3y, so y 2 = 8y. Since y > 0, this implies y = 8,
and therefore x = 4.
Answer: 4

6. If N is such a number then N is divisible by 5, so its last digit must be 0. Since N is


divisible by 3, the sum of its digist must be divisible by 3, so N must have at least three
digits 1. The smallest such number is N = 1110.
Answer: 1110

7. Since 6 AED = 90◦ − 6 BEC = 6 EBC, we have 4AED ∼ 4EBC. If a = BC, it follows
a 9
that = , so a2 = 9 · 25, i.e. a = 15.
25 a
(Alternative solution: use Pythagoras Theorem.)
Answer: 15

8. Begin by ignoring the sex of the cats.


If the 2 Os are in the first 2 boxes there are 3 ways OOWBBW, OOBWBW, OOWBWB.

1
If they’re in the next 2 boxes there are 4 ways: BOOWBW, BOOWWB, BOOBWW,
WOOBWB.
If they’re in middle 2 boxes there are 4 ways: WBOOWB, WBOOBW, BWOOWB, BWOOBW.
4th and 5th is the same as 2nd and 3rd: 4 ways.
5th and 6th is same as 1st and 2nd : 3 ways.
Total number = 3 + 4 + 4 + 4 + 3 = 18.
Since each colour can be M, F or F, M we have to multiply by 23 = 8, so number of ways
= 8 × 18 = 144.
Alternative Solution: Let the cats be W1 , W2 , O1 , O2 , B1 , B2 . We will denote by O the
pair O1 , O2 , they must be side by side (later we will take into account the fact that O1 and
O2 can swap places in O). Disregarding for a moment the fact that B1 and B2 must not be
side by side, the number of possible ways we can order W1 , W2 , O, B1 , B2 is 5·4·3·2·1 = 120.
Since O1 and O2 can swap places in O, the number of ways we can order the cats so that
O1 and O2 are side by side is 2 · 120 = 240.
From this number we have to subtract the number of cases when B1 and B2 are side by
side. If we denote the pair B1 and B2 by B, the number of ways we can order W1 , W2 , O, B
is 4 · 3 · 2 · 1 = 24. Since B1 and B2 can swap places in B, and O1 and O2 can swap places
in O, the total number of such cases is 24 · 4 = 96. Thus, the number of ways we can order
the cats satisfying both requirements is 240 − 96 = 144.
Answer: 144

9. Renaming the numbers if necessary, we may assume that

A=4−a<B =4−b<C =4−c<D =2−d<E =4−e.

The numbers A, B, C, D, E are different integers satisfying A B C D E = 12 , so each of


these numbers is equal to ±1, ±2, ±3, ±4, ±6 or ±12. If some of the numbers (that must
be A or E) is equal to 12 or −12, then all other numbers must be equal to ±1, so at least
two of them will be equal, impossible. Similarly, if some of the numbers is equal to 6 or −6,
some other number must be equal to 2 or −2, and the remaining three numbers must be
equal to ±1, so at least two of them will be equal, impossible again. Thus, all numbers are
between −4 and 4. Next, if some of the numbers is equal to 4 or −4, some other number
must be equal to 3 or −3, and the remaining three numbers must be equal to ±1, so at least
two of them will be equal, impossible again. Thus, A, B, C, D, E are five of the numbers
−3, −2, −1, 1, 2, 3. Obviously if A = −3, then E ≤ 2, and if E = 3, then A ≥ −2. Since the
product of the five numbers is positive, we cannot have three of them negative, so the only
possible case is A = −2, B = −1, C = 1, D = 2, E = 3. Then A + B + C + D + E = 3, so
a + b + c + d + e = 20 − (A + B + C + D + E) = 17.
Answer: 17

10. The answer is yes – there must be exactly 8 red, 8 blue and 8 green squares.
Let A be an arbitrary vertex of the cube. There are three small squares with vertex A and
any two of them have a common side, so they must be painted differently. Hence one of
the three squares with vertex A must be red, another must be blue, and the third must be
green.
This applies to any of the 8 vertices of the cube, so there must be at least 8 red small
squares, at least 8 blue and at least 8 green.
However the total number of small squares is 6 · 4 = 24, so there must be exactly 8 small
squares of each coulour.

2
WA Junior Mathematics Olympiad 2002
Solutions to the Team Questions
1. Divide the coins into three groups of 3 coins each. Place e.g. the coins of group 1 on one of
the pans of the pan ballance and the coins of group 2 on the other pan.
If the pans do not balance, the counterfeit coin is in the lighter pan. If the pans balance, the
counterfeit coin is in the third group. So, with one weight trial we determine a group of 3
coins that contains the counterfeit coin.
For the second trial, choose any two of these 3 coins and place them on the two pans. If
the pans balance, the third coin is the counterfeit one; if not, then the leighter coin is the
counterfeit one.

2. Divide the coins into 3 groups, the first two groups containing 9 coins each, while the third
group contains 7 coins. (Other divisions are possible, e.g. 8+8+9.) Place the coins of group
1 on one pan of the pan balance and the coins of group 2 on the other pan.
If the pans do not balance, the counterfeit coin is in the lighter pan. If the pans balance, the
counterfeit coin is in the third group. In the latter case, take two coins from group 1, say, to
get a group of 9 coins containing the counterfeit one.
So, with one weight trial we determine a group of 9 coins that contains the counterfeit coin.
Then proceed as in Problem 1 above to find the counterfeit coin using 2 weighings. Thus,
with a total of 3 weighing one can determine the counterfeit coin.

3. Divide the coins into 3 groups, the first two groups containing 27 coins each, while the third
group contains 16 coins. (There are other possible divisions that are good enough, in fact any
division so that there is no group with more than 27 coins is a good one.) Place the coins of
group 1 on one pan of the pan balance and the coins of group 2 on the other pan.
If the pans do not balance, the counterfeit coin is in the lighter pan. If the pans balance, the
counterfeit coin is in the third group. In the latter case, take 9 coins from group 1, say, to
get a group of 27 coins containing the counterfeit one.
So, with one weight trial we determine a group of 27 coins that contains the counterfeit coin.
Then proceed as in Problem 2 above (dividing the 27 coins into 3 groups of 9 coins each) to
find the counterfeit coin using 3 weighings. Thus with a total of 4 weighing one can determine
the counterfeit coin.
Let us now show that 4 is the minimal number of weighings that guarantee finding the
counterfeit coin under all circumstances.
Suppose in the first trial we weigh two groups of k coins each; the remaining coins are then
70 − 2k. If k = 23, then the third group contains 24 coins; if k < 23, then the third group
contains more than 24 coins; if k = 24 or larger, then the third group contains 22 coins or less.
Assuming that the result of our weight trial is the least favorable, the best we can achieve (in
any circumstances) from trial 1 is to determine a group of 24 coins containing the counterfeit
one.
In the same way, using a second trial the best one can achieve (in any circumstances) is to
determine a group of 24
3 = 8 coins containing the counterfeit one.
Similarly, the third trial (assuming least favorable results again) will at best give us a group
of 3 coins containing the counterfeit one, so we need one more trial.

4. Divide the coins into 3 groups of 3 coins each, and for a first weight trial place the coins of
group 1 on one pan of the pan balance and the coins of group 2 on the other pan. For a
second weight trial do the same, say, with groups 2 and 3. As a result of these two trials one

1
determines which of the three groups contains the counterfeit coin and whether this coin is
lighter or heavier than a genuine coin. For example, if group 1 is heavier (or lighter) than
group 2, and group 2 has the same weight as group 3, then the counterfeit coin is in group 1
and it is heavier (resp. lighter) than a genuine coin. If group 1 is heavier than group 2, and
group 2 is lighter (it cannot be havier) than group 3, then the counterfeit coin is in group 2
and it is lighter than a genuine coin.
Consider now the group of 3 coins found after the first two trials to contain the counterfeit
coin, and assume for clarity that the counterfeit coin is determined to be lighter than a genuine
one (the other case is similar). For a third trial place two of the coins on the two pans of the
pan balance. If the pans balance, the third coin is the counterfeit one. If one pan is lighter,
the coin in it is the counterfeit one.

Alternative Solution. Divide the coins into 3 groups of 3 coins each and place the coins of
group 1 on one pan of the pan balance and the coins of group 2 on the other pan. There are
two possibilities.

Case 1. The pans balance. Then all coins in groups 1 and 2 are genuine and the counterfeit coin
is in group 3.
Let the coins in group 3 be A, B, C. For a second weight trial, place coins A and B on
one pan and two genuine coins (say from group 1) on the other. If the pans balance,
then coin C is the counterfeit one, and a third weight trial (comparing C with a genuine
coin) will determine whether C is lighter or heavier than a genuine coin.
If the pans do not balance, then either A or B is counterfeit. Moreover at this stage
we will already know whether the counterfeit coin is lighter or heavier (e.g. if the pan
containing A and B is lighter, then the counterfeit coin is lighter than a genuine one).
For a third weight trial, place A and B on different pans of the balance, this will show
which of them is the counterfeit one. E.g. if the previous step showed that the counterfeit
coin is lighter than a genuine one, and A is lighter than B, then A is the counterfeit
coin.
Case 2. One pan is heavier. Then the coins in group 3 are all genuine.
Let the coins in the heavier pan be A, B, C (if one of these coins is counterfeit, then it is
heavier than a genuine one), and let these in the other pan be A0 , B 0 , C 0 (if one of these
coins is counterfeit, then it is lighter than a genuine one).
For a second weight trial, place e.g. A and A0 on one pan and B and B 0 on the other.
Then we have the following possibilities.
Subcase 2.1. The pans balance. Then A, B, A0 , B 0 are all genuine coins, and either C or C 0 is
counterfeit.
For a third weight trial, place C on one pan and a genuine coin (say, A) on the
other. If the pans balance, then C 0 is the counterfeit coin and it is lighter than a
genuine one. If the pans do not balance, then C is the counterfeit coin and it must
be heavier than a genuine one.
Subcase 2.2. The pan containing A, A0 is heavier than the pan containing B, B 0 . Then A0 and B
must be genuine, so the counterfeit coin is either A or B 0 .
For a third weight trial, place A on one pan and a genuine coin (say, C) on the
other. If the pans balance, then B 0 is the counterfeit coin and it is lighter than a
genuine one. If the pans do not balance, then A is the counterfeit coin and it must
be heavier than a genuine one.
Subcase 2.3. The pan containing B, B 0 is heavier than the pan containing A, A0 . This case is
considered in the same way as Subcase 2.2.

2
WA Junior Mathematics Olympiad 2003
Individual Questions 100 minutes (one hour and
40 minutes)
General instructions: No working need be given for Questions 1 to
9. Calculators are not permitted. Write your answers on the answer
sheet provided.

(1) In a test of 20 questions 5 marks are given for each correct


answer and 2 are deducted for each incorrect answer. Alan did
all the questions and scored 58. How many correct answers did
he have?
[1 mark]

(2) Jane typed a 6-digit number into a faulty computer in which


the 1 (one) key was broken. The number appearing on the
screen was 2003, possibly with some blank spaces. How many
different 6-digit numbers could Jane have typed?
[1 mark]

(3) A whole number between 1 and 99 is not greater than 90, not
less than 30, not a perfect square, not even, not a prime, not
divisible by 3 and its last digit is not 5. What is the number?
[2 marks]

(4) What is the units digit of 1! + 2! + 3! + ... + 2003!? [3! means


1 × 2 × 3]
[2 marks]

(5) How many triples (x, y, z) of positive integers satisfy (xy )z =


64?
[2 marks]

(6) Douglas is 23 as old as he will be 8 years before he is twice as


old as he is now. How old is Douglas?
[2 marks]

1
(7) In a triangle ABC, ∠C = 90◦ . A perpendicular is produced
from the midpoint D of AB to meet the side BC at E. The
length of AB is 20 and the length of AC is 12.
What is the area of triangle ACE?

[3 marks]

(8) If 2x + 3y + z = 48 and 4x + 3y + 2z = 69 what is 6x + 3y + 3z?


[4 marks]

(9) The sum of six consecutive positive odd integers starting with
n is a perfect cube. Find the smallest possible n. [4 marks]

(10) ABCD is a parallelogram. E is the mid point of AB. Join E to


D. ED intersects AC at P.
How many times larger is the area of the parallelogram ABCD
than the area of the triangle AEP?
Give reasons for your solution. [4 marks]
Western Australian
Junior Mathematics Olympiad 2003
Team Questions 45
minutes

A. Suppose you have a string of 2003 beads which you cut between
two beads as close to the middle as possible, so that you now
have two strings, one with 1001 beads and one with 1002. The
beads are glued onto the string so they won’t slide off. You
now take the shorter string of the two and cut it as close to the
middle as possible and then keep repeating the process, at each
step choosing a shorter string. You may choose either if they
are equal. Stop when you have a string with only one bead.
How many strings have you now got?
B. Find all possible initial string lengths which will finish with 5
pieces.
C. How many strings would you have if the original string had
999,999 beads?
D. Can you give a formula for the longest string and a formula for
the shortest string which finish in n pieces?
E. What would happen if you cut the original string of 2003 beads
as nearly as possible into thirds, and then took a smallest length
to continue?
Western Australian Junior Mathematics Olympiad
November 1, 2003

Problem Solutions

1. Suppose Alan solved n questions correctly. Then 58 = 5n − 2(20 − n) = 7n − 40,


so 7n = 98 and therefore n = 14.

2. There are 6 places in which to type two 1’s, the remaining 4 places being filled with
2003.
So there are 6 choices for the place of the first 1 and then 5 for the second 1, a total of
30. But only half of these look different from each other so there are 15 possibilities.

3. From the first line of the question, we are looking for a number between 30 and 90.
From the second line it is odd and not 49 or 81.
Also it is not prime, so 31, 37, 41, 43, 47, 53, 57, 59, 61, 67, 71, 73, 79 83, 87 and 89
are also excluded.
Since it is not divisible by 3 or 5, 33, 35, 39, 45, 51, 55, 63, 65, 69, 75, 85 are also
excluded.
The only remaining possibility is 77.

4. Since 5! and n! for all n > 5 are divisible by 10, the required units digit is the units
digit of 1! + 2! + 3! + 4! = 1 + 2 + 6 + 24 = 33. hence the answer is 3.

5. First notice that 64 = 26 so x must be a power of 2 such that xyz = 64. The only
possibilities are x = 2, yz = 6 or x = 4, yz = 3, or x = 8, yz = 2 or x = 64, yz = 1.
So altogether there are 9 solutions, (x, y, z) = (2, 1, 6), (2, 2, 3), (2, 3, 2), (2, 6, 1), (4, 1, 3),
(4, 3, 1), (8, 1, 2), (8, 2, 1) and (64, 1, 1).

6. Suppose Douglas is x years old. Eight years before he is twice as old as x, he will be
2x − 8 years old. So x = 32 (2x − 8).
Hence 3x = 4x − 16, so x = 16.

7. Note that triangle ABC is four times as big as a right–angled 3 − 4 − 5 triangle, so,
|BC| = 16.
Suppose |EC| = x so |BE| = 16 − x. Since BEA is isosceles, |AE| = 16 − x also.
By Pythagoras’ Theorem, x2 = |EC|2 = (16 − x)2 − 144, so 144 = 256 − 32x. Hence
x = 112/32 = 3.5 so the area of triangle ACE is 6 × 3.5 = 21.

1
8. First notice that by adding the left sides and the right sides of the equations, you get
6x + 6y + 3z = 117. We need to subtract 3y to get 6x + 3y + 3z. But if you subtract
69 = 4x + 3y + 2z from 96 = 4x + 6y + 2z you get 3y = 27.
Hence 6x + 3y + 3z = 117 − 27 = 90.

9. The sum of the six consecutive odd integers starting with n = n + (n + 2) + (n + 4) +


(n + 6) + (n + 8) + (n + 10) = 6n + 30 = 6(n + 5).
The smallest cube of the form 6(n + 5) occurs when n + 5 = 36, so n = 31.

10. DP C + DP A = 1/2ABCD

But DP C is similar to and twice the size of AP E. Hence 4AP E+DP A = 1/2ABCD.
DP A + AP E = 1/4ABCD. Hence 3AP E = 1/4ABCD so ABCD is 12 times the
area of AEP .
Another way to do it is to join PB. Then AEP = EP B and DP O = BP O, while
AOD = AOB. Hence AP D = AP B = DP B so they each form one sixth of ABCD.
Therefore ABCD is 12 times AEP .

Team Problem Solutions

A. Step 1 gives 2 strings, shortest length 1001.


Step 2 gives 3 strings, shortest length 500.
Step 3 gives 4 strings, shortest length 250.
Step 4 gives 5 strings, shortest length 125.
Step 5 gives 6 strings, shortest length 62.
Step 6 gives 7 strings, shortest length 31.
Step 7 gives 8 strings, shortest length 15.

2
Step 8 gives 9 strings, shortest length 7.
Step 9 gives 10 strings, shortest length 3.
Step 10 gives 11 strings, shortest length 1.

B. The shortest string length to gve 5 pieces is 16, the lengths being 8, 4, 2, 1 and 1.
The longest string length to give 5 pieces is 31, the lengths being 16, 8, 4, 2, 1.
Any string length between 16 and 31 also gives 5 pieces.

C. The answer is the lowest power of 2 ≥ 106 . Since 210 = 1024, 220 > 106 and 219 < 106
so the answer is 20.

D. 2n−1 and 2n − 1.

E. If you start with a string length of 2003, continually cut as nearly as possible into
thirds and continue with a smallest piece, you get in successive cuts:
3 strings, shortest length 667
5 strings, shortest length 222
7 strings, shortest length 74
9 strings, shortest length 24
11 strings, shortest length 8
13 strings, shortest length 2
You cannot proceed any further. In general, if you start with a string of any length,
you will finish with a shortest string of length 1 or 2.
If you start with a string of length x, the number of strings you get will be 2k + 1,
where k is the highest power of 3 ≤ x.
To finish with n strings, n must be odd, say n = 2k + 1. The smallest string you can
start with has length 3k and the largest string you can start with has length 3k+1 − 1.

3
Western Australian
Junior Mathematics Olympiad 2004
Individual Questions 100 minutes

General instructions: No working need be given for Questions 1 to


9. Calculators are not permitted. Write your answers on the answer
sheet provided. Each solution is a positive integer less than 100.

(1) Paul likes dogs. At present all his adult dogs are dalmations
while some of his puppies are dalmations and some are not. In
all he has 11 dogs of which 7 are dalmations and 8 are puppies.
How many dalmation puppies has he?
[1 mark]

(2) If a hen and a half lay an egg and a half in a day and a half,
how many eggs will 6 hens lay in 12 days?
[1 mark]

(3) Triangles ABC, ACD, ADE, AEF , AF G are right-angled with


hypotenuses AC, AD, AE, AF , AG respectively. If AB = 2
and BC = CD = DE = EF = F G = 3, what is the length of
AG?
E D

F C

G
A B
[2 marks]

(4) How many perfect squares divide 7200 exactly?


[2 marks]

(5) How many 5 digit numbers consisting only of 1s and 2s have no


adjacent 2s?
[2 marks]
(6) In a circle with centre O, a chord AB is extended to a point
C such that the length of BC is equal to the radius of the
circle. CO is drawn and extended to a point D on the circle’s
circumference. If angle BCO = 15◦ , what is angle AOD?
[3 marks]

(7) A chessboard is made up of 64 squares each with 3 cm sides. A


circle is drawn with its centre at the centre of the chessboard
and radius 12 cm. How many of the 64 squares lie entirely inside
the circle?
[3 marks]

(8) There was a young lady called Chris


Who when asked her age answered this:
“Two thirds of its square
Is a cube, I declare.”
So what was the age of this Miss?
[3 marks]

(9) Will has 12 square tiles. Using all the tiles each time, he can
make three different shaped rectangles, like this:

What is the least number of tiles he needs, so that, using all


the tiles each time, he can make five different rectangles?
[4 marks]

(10) There was little traffic that day, too little to interfere with the
steady progress of the 3 km column of armoured vehicles. Head-
ing the column, Tom turned his jeep and drove back to check
the rear. All was well and he was able to maintain a steady
speed there and back without any delays. On returning to his
lead position, Tom noted that the column had advanced just
4 km while he was away. How far had he driven in that time?
Give reasons for your solution. [4 marks]
Western Australian
Junior Mathematics Olympiad 2004

Team Questions 45 minutes

A. On Genevieve’s computer screen there are two dials, as shown


below.
s s
0 0
8 1
4 1 7 2
6 3
3 2 5 4

Each time she clicks her mouse the black dot on each dial moves
one position clockwise. So that if she clicked her mouse 12 times
the dot on the left hand dial would be in position 2 and the dot
on the right hand dial would be in position 3. What is the least
number of clicks she needs to make to get from the position above
to one with the left dot at 1 and the right at 8?

B. On Ahmed’s computer the dots start in the positions shown be-


low.
0 0
8 1
4 1 7 2
s6 3
3 2s 5 4

How many clicks must he make to get the left dot at 1 and the
right at 4?

C. The left hand dial on Nelson’s computer is numbered from 0 to


16 and the right dial from 0 to 23. Initially the left dot is at
position 9 and the right at position 11. How many clicks does
Nelson need to make to get the left hand dot in position 7 and
the right at position 15?
D. Yenicca’s screen has 3 dials, which start in the positions shown
below.
0 0 s 0
5 1 6 1
4 1
5 2
s4 2
3 2s 3 4 3

When she clicks her mouse each dot moves one position clockwise.
How many clicks will she need to make to finish with the left
hand dot at position 0, the middle at position 2 and the right at
position 4?
E. Roxanne’s screen also shows 3 dials which start in the same posi-
tion as Yenicca’s. However when she clicks the left mouse button
only the left hand and middle dots move one position clockwise.
If she clicks the right mouse button the middle and right hand
dots move one position clockwise. How many times must she
click each button to finish with the left hand dot at position 0,
the middle at position 2 and the right at position 4?
Solutions
Solutions to Individual Questions

(1) Since Paul has 11 dogs and 8 puppies he must have 3 adult dogs.
We are told all his adult dogs are dalmations; so he has 3 adult
dalmations. The rest of his 7 dalmations must be puppies; so he
has 4 dalmation puppies. (This question can also be answered
using Venn diagrams.) [1 mark]

(2) If a hen and a half lay an egg and a half in a day and a half,
6
then 6 hens will lay 1.5 × 1.5 = 6 eggs in a day and a half. So
12
in 12 days they lay 1.5 × 6 = 48 eggs. [1 mark]

(3) In order to avoid confusion about the meaning of expressions


like AC 2 , we use the convention in our proof that |AC| denotes
the length of the line segment AC.
By Pythagoras’ Theorem (several times)
√ √
|AC| = 22 + 32 = 13
q√ √
|AD| = ( 13)2 + 32 = 22
q√ √
|AE| = ( 22)2 + 32 = 31
q√ √
|AF | = ( 31)2 + 32 = 40
q√ √
|AG| = ( 40)2 + 32 = 49 = 7
So the length of AG is 7.
Alternative solution. More elegantly (again using Pythago-
ras’ Theorem several times), we have
|AG|2 = |F G|2 + |AF |2
= |F G|2 + |EF |2 + |AE|2
..
.
= |F G|2 + |EF |2 + |DE|2 + |CD|2 + |BC|2 + |AB|2
= 5 × 32 + 22
= 49
∴ |AG| = 7
So again the length of AG is 7. [2 marks]
(4) 7200 = 2 × 3600 = 2 × 602 . From this we see that the square
of any divisor of 60 is a square divisor of 7200, and vice versa.
So we need only count the divisors of 60. These are 1, 2, 3, 4,
5, 6, 10, 12, 15, 20 and 60. So 12 perfect squares divide 7200
exactly. [2 marks]

(5) There is 1 number of the given shape that contains no 2, 5


which contain one 2, 6 which contain two 2s, and 1 (21212)
which contains three 2s. Altogether there are 1 + 5 + 6 + 1 = 13
such numbers. [2 marks]

(6) Triangle OBC is isosceles, so angle BOC is 15◦ . Angle ABO is


an external angle of the triangle OBC and so equals 2 × 15◦ =
30◦ . Triangle AOB is also isosceles; so angle BAO also equals
30◦ .
C
B

A
O

Now angle AOD is an external angle to triangle OAC, and so


∠AOD = ∠BAO + ∠BCO = 30◦ + 15◦ = 45◦ . [3 marks]

(7) The main question is to decide whether the points shown as


solid dots are √
inside or outside
√ the circle. Their distance from
the centre 2 2
√ √ is 3 + 3 = 18, by Pythagoras’ Theorem, and
18 > 16 which is the radius of the circle; so the points are
outside the circle.

t t
× ×
× × ×
× × ×

t t
Hence the squares in the first quadrant of the circle that are
marked × all lie inside the circle, and so in all there are 4 × 8 =
32 squares inside the circle. [3 marks]

(8) Let her age be x and the cube be y 3 . (Both x and y are integers.)
Then
2 2
3
x = y3.
Hence
2x2 = 3y 3 (1)
Now 2 divides the LHS . . . so 2 divides the RHS and hence 2
divides y. Thus 8 divides the RHS. So 2 divides x.
Now try a similar idea with 3: 3 divides the RHS . . . so 3 divides
the LHS and hence 3 divides x. Thus 9 divides the LHS (and
hence the RHS). So 3 divides y. So 81 divides the RHS and
hence 9 divides x.
Thus lcm(2, 9) = 18 divides x and lcm(2, 3) = 6 divides y. Now
let x = 18α and y = 6β. Then substituting in (1) we get:
2 × 182 α2 = 3 × 63 β 3
which reduces to
α2 = β 3
Suppose for a prime p, pi is the largest power of p that divides
the LHS. Then i is a multiple of 2. Also since pi is the largest
power of p that divides the RHS, i is a multiple of 3. Thus i is
a multiple of 6. Either i is 0 for every prime p or i is at least 6
for some prime p. If i is at least 6 for some p then at least p3
divides α in which case α ≥ 23 = 8 and x ≥ 8 × 18 = 144 and
by today’s standards Chris would not be a young lady and a
candidate for the Guinness Book of Records. Thus α = 1 and
x = 18. So Chris is 18. [3 marks]

(9) The question is essentially asking for the least number that can
be written as the product of two numbers in 5 different ways
without regard to order. The answer is 36. He can then make
rectangles with dimensions 1 × 36, 2 × 18, 3 × 12, 4 × 9 and
6 × 6. One way that we could have found this is by observing
that if the number of tiles n has prime factorisation
p11 p22 · · · pnn
then each factor of n has the form
pi11 pi22 · · · pinn
where 0 ≤ ij ≤ j , for j = 1, . . . , n. Hence n has
(1 + 1)(2 + 1) · · · (n + 1)
distinct factors. So we require (1 + 1)(2 + 1) · · · (n + 1) to be
either 2 × 5 − 1 = 9 if n is a perfect square, or 2 × 5 = 10 if n
is not a perfect square.
Now 10 = 2 × 5 suggests 1 = 4, 2 = 1; the least n = p41 p2
occurs for p1 = 2, p2 = 3, namely n = 48. However, if n is a
perfect square, in which case the j must all be even, we see
that 9 = 3 × 3 suggests 1 = 2, 2 = 2, and the least n = p21 p22
again occurs for p1 = 2, p2 = 3, and is n = 36. [4 marks]

(10) Let v be the jeep’s speed and u be the column’s speed (in
km/hr), let T be the total time of Tom’s round trip (in hrs),
and let x be the total distance Tom travelled (in km). Then
3 3
T = + , uT = 4, vT = x
v+u v−u
Rearranging the last two equations we obtain u = 4/T , v =
x/T (noting that this is allowed since T cannot be zero). Now
substitute those expressions for u and v in our first equation:
3 3 3T 3T
T = + = +
x 4 x 4 x+4 x−4
+ −
T T T T
Dividing both sides by T (which is non zero), followed by rear-
ranging we get:
3 3
1= +
x+4 x−4 
(x + 4)(x − 4) = 3 (x − 4) + (x + 4)
x2 − 16 = 6x
x2 − 6x − 16 = 0
(x − 8)(x + 2) = 0.

So x is 8 or −2 . . . but x cannot be negative. Hence Tom drove


8 km.
Alternative solution. Let x, u, v, T be as defined above. Fur-
ther, let y be the distance Tom travelled to the rear of the
column and t the time needed for that. Then vt = y and
ut = 3 − y, so v/u = y/(3 − y). On the other hand, for the
total distances vT = 2y + 4 and uT = 4, so v/u = (2y + 4)/4.
Thus, y/(3 − y) = (2y + 4)/4, which gives y 2 + y − 6 = 0, i.e.
y = 2 (we disregard the possibility y = −3 since y > 0). The
total distance is then 2y + 4 = 8 km. [4 marks]
Solutions to Team Questions
(A) After 8 clicks the dots are in positions 3 and 8 so the right dot is
OK. A further 9 clicks leaves the right dot at 8 but the left is now
at 2. After 9 more the dots are at positions 1 and 8 as required.
So we need 8 + 9 + 9 = 26 clicks. [5 marks]
(B) After 7 clicks the dots are at positions 4 and 4. 9 more takes us
to positions 3 and 4, 9 more to 2 and 4, and another 9 to 1 and 4.
So we need 7 + 9 + 9 + 9 = 34 clicks. [10 marks]
(C) 5 clicks move us to positions 7 and 2. 24 more take us to 14 and
2. Then after 15 more sets of 24 clicks we arrive at 0 and 2. The
total number of clicks required is therefore 15 + 24 × 16 = 399.
[10 marks]
(D) We first get the two right hand dials correct. 5 clicks put the dials
in positions 2, 3, 4 so that the third dial is right. 7 more clicks
move us to positions 4, 4, 4 with dial 3 still OK and it stays OK
with each set of 7 clicks. After a total of 5 + 5 × 7 clicks we get
positions 2, 2, 4 with dials 2 and 3 OK. Now a set of 42 more clicks
will leave these dials in the correct position and advances the first
dial 2 positions. We find that after 4 sets of 42 clicks the first
dial is in position 0 and we are done. The total number of clicks
needed is 5 + 5 × 7 + 4 × 42 = 208. [10 marks]
(E) Say that L is the number of times Roxanne clicks the left button
and R is the number of times she clicks the right button. Then
if L = 3 and R = 0 we get to positions 0, 1, 6 which puts the left
hand button in the correct position. Then L = 3 and R = 5 moves
us to 0, 0, 4 with the left and right dials in the correct positions.
Increasing L by multiples of 5 and R by multiples of 7 leaves these
dials in the correct positions, but moves the middle dot. If we
perform both these operations we move the middle dot 12 positions
which is the same as leaving it alone so we should be adding a
multiple of 5 to L or a multiple of 7 to R but not both. Trying
both schemes we find we can get to 0, 2, 4 using (L, R) = (23, 5)
or (L, R) = (3, 19). The second is the most efficient so she should
click the left button 3 times and the right 19 times. [10 marks]
Western Australian
Junior Mathematics Olympiad 2005
Individual Questions 100 minutes

General instructions: Each solution in this part is a positive integer


less than 100. No working is needed for Questions 1 to 9. Calculators
are not permitted. Write your answers on the answer sheet provided.

(1) ABCD is a trapezium with AB parallel to DC, AB = 4 cm,


DC = 11 cm and the area of triangle ABP is 12 cm2 . What is
the area of the trapezium ABCD in square centimetres?

A 4 cm B
S c
 S  cc
 S  c
 S 12 cm2  c
 S  c
 S  c
c
 S  c
D S cC
P
 11 cm -

[1 mark]

(2) A library has 6 floors. There are 10 000 more books on the
second floor than the first. The number of books on the third
floor is the same as the number on the second. There are 10 000
fewer books on the fourth floor than the third and twice as many
books on the fifth floor as there are on the fourth. On the sixth
floor there are 4 000 fewer books than on the fifth. Coinciden-
tally the number of books on the sixth floor is the same as the
number on the first. Altogether, how many thousands of books
are there in the library?
[1 mark]

(3) XY Z is a three-digit number. Given that


XXXX + Y Y Y Y + ZZZZ = Y XXXZ,
what is X + Y + Z?
[2 marks]
(4) Mathilda and her friends have 20 1-metre sticks out on the
basketball court to make triangles. Below is a diagram of a
triangle using all 20 sticks. It has sides of 5 m, 7 m and 8 m. All
triangles with sides of 5 m, 7 m and 8 m (even mirror images as
shown) are considered to be the same.

5m 7m 7m 5m

8m 8m

Altogether, how many different triangles could they make, one


at a time, each time using all 20 sticks?
[2 marks]

(5) What is the size of the angle, in degrees, between the hands of
a clock when the time is ten past eleven?
[2 marks]

(6) A plane is due to leave Perth at midnight and arrive at Tokyo


at 12:29 pm, but it departs 49 minutes late. What percentage
increase in speed is required in order that it will arrive exactly
on time?
[3 marks]

(7) Esther has 20 coins in her purse. They are 10c, 20c and 50c
coins and the total value is $5. If she has more 50c coins than
20c coins, how many 10c coins has she?
[3 marks]

(8) Alice spent all her money in five shops. In each shop, she spent
$1 more than half of what she had when she entered that shop.
How many dollars did Alice have when she entered the first
shop?
[3 marks]
(9) Given a square ABCD, circular arcs centred at B and D are
drawn from A to C. Now draw diagonal BD √ to cut these arcs
at X and Y , respectively. If XY = 12 − 6 2, what is the area
of the square ABCD?
A B

D C
[4 marks]

(10) Farmer Brown runs a dairy farm with cows, sheep and goats.
Dabbling in mathematics in his spare time, he noticed that the
numbers of each animal were different prime numbers. He also
observed that if he multiplied the number of cows by the to-
tal number of cows and sheep, he obtained a number just 120
greater than the number of goats. How many goats are there?

For full marks, explain how you found your solution.


[4 marks]
Western Australian
Junior Mathematics Olympiad 2005
Team Questions 45 minutes

General instructions: Calculators are (still) not permitted.

A number appears on your computer screen. If you push the A key


the number is decreased by 3, if you push the B key it’s increased by
3 and if you press the C key the number is halved. You want to use a
sequence of key strokes to change the number to 1.
For example, there are three ways to change 16 to 1:
• Use CCCC giving the number sequence 16, 8, 4, 2, 1.
• Use AAAAA giving 16, 13, 10, 7, 4, 1.
• Use CCA giving 16, 8, 4, 1.
So the shortest path from 16 to 1 requires 3 key strokes.

A. Find a starting number between 30 and 40 whose shortest path is


4, and show the path.
B. Find a starting number between 50 and 60 whose shortest path is
5, and show the path.
C. Find a starting number between 30 and 40 whose shortest path is
6, and show the path.
D. Which starting number between 40 and 50 has the longest “shortest
path”?
E. Find a number which has two shortest paths, one beginning with
the A key and one with the B key.
F. Find a starting number where the “add 3” operation must be used
to get the shortest path.
G. What starting numbers can never get to 1?
H. What starting number between 500 and 1000 would have the short-
est path?
I. In this question, the numbers on the screen don’t have to be inte-
gers. If we start with any number then these two sequences of key
strokes always produce the same final number: ACCB and BCBC.
Explain why.
Solutions
Solutions to Individual Questions
(1) Let h be the height of the trapezium. This is also the height
of triangle ABP . Thus 12 = 12 × 4h so h = 6. The area of the
trapezium is then
AB + DC 4 + 11
Areatrap = h × =6× = 45.
2 2
(2) Let there be x thousand books on the first floor.
On the second floor there are x + 10 thousand books.
On the third floor there are x + 10 thousand books.
On the fourth floor there are x thousand books.
On the fifth floor there are 2x thousand books.
On the sixth floor there are 2x − 4 thousand books.
Thus we have 2x − 4 = x and so x = 4. The total number of
books, in thousands, is therefore
x + x + 10 + x + 10 + x + 2x + x = 7x + 20
= 7 × 4 + 20 = 48.

(3) Writing the sum in the traditional way,


XXXX
YYYY
ZZZZ
Y XXX Z
we observe from the last column that X + Y + Z = Z + 10k,
where k is the carry to the second column, and so X +Y = 10k.
Now X + Y can’t be as big as 20, and, since XY Z is a 3-digit
number, X 6= 0. Thus
X + Y = 10 (1)
and the carry k = 1. Looking at the second column now, we
have X + Y + Z + 1 = X + 10m, i.e. Y + Z + 1 = 10m, where
m is the carry to the third column. Again, Y + Z + 1 can’t be
as big as 20, so
Y + Z + 1 = 10 (2)
and m = 1. We see that this pattern is repeated for the third
and fourth columns, and so the carry to the fifth column which
is just Y , implies Y = m = 1. Adding equations (1) and (2) we
have
X + Y + Y + Z + 1 = 20
X + Y + Z = 20 − 1 − Y
= 18
(Of course, we had enough information to evaluate that X = 9
and Z = 8.)
(4) We’re looking for sets of three positive integers x, y, z satisfying
x + y + z = 20 and (so that we don’t count the same set twice)
x ≥ y ≥ z. With x the length of the hypotenuse, we also
need x > y + z; otherwise we can’t form a triangle. Listing
systematically we get:
9,9,2
9,8,3
9,7,4
9,6,5
8,8,4
8,7,5
8,6,6
7,7,6.
So there are 8 possible triangles.
(5) The minute hand is 1/6 of a revolution past the 12, which is 60
degrees and the hour hand is 5/6 of the angle between 11 and
12 which is 5/6 = 1 − 1/6 of 1/12 of a revolution before the 12,
i.e. 25 degrees, so the angle between the hands is 85 degrees.
(6) The time usually taken for the trip is 12×60+29 = 749 minutes.
In order to arrive on times the pilot must do the trip in 749 −
49 = 700 minutes. Let the distance between Perth and Tokyo be
d kilometres. The usual speed is therefore d/749, the required
speed after the late departure is d/700. The required percentage
increase in speed is therefore,
d/700 − d/749  1 1 
× 100% = 749 − × 100%
d/749 700 749
 749 749 
= − × 100%
700 749
= (1.07 − 1) × 100%
= 7%.
So the required answer is 7.
(7) Let the number of 10c, 20c and 50c coins be x, y and z respec-
tively. We have,
x + y + z = 20 (3)
10x + 20y + 50z = 500 (4)
and z > y. Dividing (4) by 10 and subtracting (3) gives
y + 4z = 30. (5)
Since z > y this means 5z > 30 and so z ≥ 7. If z ≥ 8 the left
hand side of (5) is at least 32, which is impossible, so z = 7 and
we quickly find y = 2 and x = 11. So the number of ten cent
coins is 11.
(8) In each shop she spends 2 more then she has when she leaves.
So if she leaves with y dollars then she spent y + 2 and entered
with 2y + 2 dollars.
She leaves the last shop with $0, so (using y = 0), she entered
shop 5 with $2.
She left shop 4 with $2, so entered it with 2 × 2 + 2 = 6 dollars.
She left shop 3 with $6, so entered it with 2 × 6 +2 = 14 dollars.
She left shop 2 with $14, so entered with 2×14+2 = 30 dollars.
She left shop 1 with $30, so entered with 2×30+2 = 62 dollars.
So Alice had 62 dollars when she entered the first shop.
(9) Let the side-length of the square be s.√Now BX = DY = s,
and, by Pythagoras’ Theorem, BD = s 2. Therefore

XY = BX + DY − BD = (2 − 2)s.
√ √
But we’re told XY = 12 − 6 2 = 6(2 − 2). Thus s = 6 and
the square has area 36.
(10) Let the number of cows, sheep and goats be c, s and g respec-
tively. The farmer notices that

c(c + s) = g + 120.

We know c, s and g are primes. If they were all odd then the
left hand side of the above equation would be even and the right
odd which is impossible. So one of them is even and therefore
equals 2.
If c = 2 the left hand side of the equation is still even and the
right hand side odd.
If g = 2 the equation becomes c(c + s) = 122 = 2 × 61. This is
impossible with c odd.
Therefore we must have s = 2 and get

c2 + 2c = g + 120
c2 + 2c + 1 = g + 121
(c + 1)2 = g + 112
g = (c + 1)2 − 112
= (c + 1 + 11)(c + 1 − 11)

Since g is a prime the first of the factors here must equal g and
the other equal 1:

g = c + 1 + 11 (6)
1 = c + 1 − 11 (7)

From (7), we obtain c = 11; whence from (6) we deduce g = 23.


Solutions to Team Questions

(A) There is just one path with a starting number between 30 and 40
such that the shortest path from that starting number has length
4:
32, 16, 8, 4, 1
The sequence of operations giving this path is CCCA. An impor-
tant observation here is that C effects a more rapid descent for
numbers x that are even and greater than 4, but A requires just
one step to convert 4 to 1 (compared with performing CC).
(B) There is just one path with a starting number between 50 and 60
such that the shortest path from that starting number has length
5:
56, 28, 14, 7, 4, 1
The sequence of operations giving this path is CCCCA.
(C) There are three paths with a starting number between 30 and 40
such that the shortest path from that starting number has length
6:
37, 34, 17, 14, 7, 4, 1 (by sequence: ACACAA) or
37, 40, 20, 10, 7, 4, 1 (by sequence: BCCAAA) or
37, 40, 20, 10, 5, 2, 1 (by sequence: BCCCAC)
Thus all such paths start at 37. Thus we can see that a shortest
path can involve the B operation.
(D) The starting number between 40 and 50 that has the longest
“shortest path” is 49, with paths
49, 46, 23, 20, 10, 7, 4, 1 (ACACAAA) or
49, 46, 23, 20, 10, 5, 2, 1 (ACACCAC) or
49, 52, 26, 13, 10, 7, 4, 1 (BCCAAAA) or
49, 52, 26, 13, 10, 5, 2, 1 (BCCACAC) or
49, 52, 26, 13, 16, 8, 4, 1 (BCCBCCA)
all of length 7. The path is not required.
(E) Starting numbers that have two shortest paths, one beginning with
the A key and one with the B key, that are less than 100 are 13,
25, 37, 41, 49, 65, 73, 89, 97.
(F) Starting numbers where the “add 3” operation must be used to
get the shortest path, that are less than 100 are 29, 53, 58, 61, 77,
85.
(G) The starting numbers that can never get to 1 are multiples of 3.
(The way to see this is that if a number starts as a multiple of 3,
then after any of the A, B or C operations is performed, the result
is still a multiple of 3, and of course 1 is not a multiple of 3.)
(H) The starting number between 500 and 1000 that has the shortest
path is 512.
(I) We need to show that ACCB and BCBC always produces the
same result. Let x be the starting number. Then ACCB produces
 x−3
(x − 3)/2 /2 + 3 = +3
4
x − 3 + 12 x+9
= =
4 4
On the other hand, BCBC produces
 x + 3 
(x + 3)/2 + 3 /2 = + 3 /2
2
x + 3 + 6
= /2
2
x+9
=
4
Thus irrespective of what starting number x is chosen. The result
after each of the sequences ACCB and BCBC is (x + 9)/4.
Western Australian
Junior Mathematics Olympiad 2006

Individual Questions 100 minutes

General instructions: Each solution in this part is a positive integer


less than 100. No working is needed for Questions 1 to 9. Calculators
are not permitted. Write your answers on the answer sheet provided.

1. The diagram below shows the train line from the outer suburb of A
to the inner city station E, with the distance between stations shown
in kilometres. After leaving a station the train travels at an average
speed of 30 km/h for the first kilometre, then at 60 km/h until it
reaches the next station. It spends 2 minutes at each station. How
many minutes will elapse between the train leaving A and arriving
at E?
3 km 4 km 6 km 1 km
A B C D E

[1 mark]
Solution. 60 km/h = 1 km/minute. So when the train travels at
60 km/h, it takes 1 minute to cover a km, and when it travels at
30 km/h, it takes 2 minutes to cover a km. So the minutes that will
elapse between the train leaving A and arriving at E is given by:
(2 + 2) + 2 + (2 + 3) + 2 + (2 + 5) + 2 + (2 + 0) = 24
where the first number (2) in each bracket is the time spent at
30 km/h, the second number in each bracket is the number of kilo-
metres travelled at 60 km/h (which equals the time in minutes to
cover that distance) and each unbracketed 2 is the waiting time at
a station. Answer: 24.

2. Find the two-digit prime number that is 2 less than a perfect square
that is 2 less than a prime. [1 mark]
Solution. Let the prime be p. Then p has 2 digits and hence is
odd (2 is the only even prime). It is 2 less than a square. So the
square is odd; possibilities are: 25, 49, 81. Of these only 81 has
a prime both 2 less than it and 2 more than it. So p must be 79.
Answer: 79.
3. For how many positive integers (whole numbers) n are n, 3n and
n/3 all three-digit integers? [2 marks]
Solution. The least that n/3 can be is 100 (the smallest 3-digit
positive integer) and the most that 3n can be is 999 (the largest
3-digit positive integer). If 3n = 999 then n/3 = 111. So n/3 can
be any integer from 100 to 111 inclusive, a total of 12 possibilities,
leading to 12 possibilities for n (300, 303, . . . , 333). Answer: 12.

4. A point E lies outside a square ABCD so that ABE is an equilateral


triangle. What is the measure in degrees of the angle CED?
E

A B

D C
[2 marks]
Solution. Since 4ABE is equilateral, its angles are all 60◦ . Hence
∠EAD = 60◦ + 90◦ . Now 4ADE is isoceles. Thus
1
∠AED = (180◦ − ∠EAD)
2
= 15◦
Similarly, ∠BEC = 15◦ . So finally we have
∠CED = 60◦ − 2 × 15◦ = 30◦ .
Answer: 30.

5. A video store has a choice of 920 films to rent. You can rent some
on DVD, some on video cassette and some on both. If the store
owns a total of 1000 DVDs and video cassettes, how many films are
available on both video cassette and DVD? [2 marks]
Solution. By the pigeon hole principle, 80 films are available on
both video cassette and DVD (the films are the pigeon holes; put
920 of the 1000 DVDs and video cassettes in the 920 available pigeon
holes; the remaining 80 DVDs or video cassettes must go in a pigeon
hole that already contains a DVD or video cassette). Answer: 80.
6. Brenda was sick on the day of the maths test so she had to sit for
it the next day. Her score of 96 raised the class average from 71 to
72. How many students (including Brenda) took the test?
[3 marks]
Solution. Let n be the number of students in the class. Without
Brenda the average is 71 and hence the total marks without Brenda
is 71(n − 1). The average with Brenda’s mark of 96 is 72. So

71(n − 1) + 96
72 =
n
72n = 71(n − 1) + 96 = 71n − 71 + 96
n = 25

So there are 25 students in the class. Answer: 25.

7. A plane flies in still air at an average speed of 810 km/h for the
duration of its flights. When flying from Perth to Sydney it takes
four hours while from Sydney to Perth it takes five hours. Assuming
that the wind is at a constant speed and from the west (i.e. in the
direction from Perth to Sydney) for both flights, what is the speed
of the wind? [3 marks]
Solution. Let w be the speed of the wind (from west to east). Then
travelling east the plane’s speed is (810 + w) km/h and travelling
west its speed is (810 − w) km/h. In general velocity v, distance d
and time t are related by v = d/t. Let d now denote the distance
from Perth to Sydney. Then
d
810 + w = (1)
4
d
810 − w = (2)
5
Eliminating d by dividing (1) by (2) we have
810 + w 5
=
810 − w 4
4(810 + w) = 5(810 − w)
4w = 810 − 5w
9w = 810
w = 90

Therefore the wind speed w is 90 km/h. Answer: 90.


8. The five faces of a right square pyramid all have 6
the same area. The height of the pyramid is 3 m.
What is its total surface area in square metres?
3m

? 


[3 marks]
Solution.
Let a be the sidelength of the square base, so that the 6
area of the base is a2 which is also the area of each
isosceles triangular face. So each triangular face has 3m
height 2a, and the total surface area is 5a2 . Using
Pythagoras’ Theorem and the pyramid height of 3 m, 
we have ? 

(2a)2 − (a/2)2 = 32
(4 − 14 )a2 = 15 2
4 a = 3
4 · 5a2 = 9
5a2 = 4
3 · 9 = 12
So the total surface area of the pyramid is 12 m2 .
Answer: 12.

9. Jasper glues together 504 cubes with 1 cm edges to make a solid


rectangular-faced brick. If the perimeter of the base of the brick is
64 cm, what is the height of the brick? [4 marks]
Solution. 504 = 9 × 56 = 9 × 7 × 8 = 23 × 32 × 7. The volume
of the brick L × B × H = 504 cm3 , whereas the perimeter of the
base 2(L + B) = 64 cm. So we need to write 504 as the product of
three positive integers L, B, H such that L + B = 32. We see that
18 × 14 × 2 = 504, so that (in cm) L = 18, B = 14, H = 2 is a
solution.
To see the solution H = 2 is unique (not required for the competi-
tion), observe first that L must satisfy 16 ≤ L < 32 (taking L ≥ B).
Now L and B must both be even or both odd since there sum is
even. If L and B are both odd then their product is 63 leading to
L = 21 (to satisfy the inequality) and B = 3 which do not sum to
32. Hence L and B must both be even,√whence H = 1 or 2. If
H = 1 then the least L + B could be is 2 504 > 2 × 22 = 44 > 32.
Thus H = 2 cm. Answer: 2.
10. In the AFL Grand Final between the Knockers and the Beagles the
Knockers won by 3 points. This was despite the Beagles kicking
16 scoring shots compared to the Knockers 14. It was also noticed
that the number of behinds kicked by the Beagles was greater than
the number of goals kicked by the Knockers, and the number of
goals kicked by the Beagles was greater than the number of behinds
kicked by the Knockers. How many points did the Beagles score?

Note: in the AFL game there are two ways to score: behind s score
1 point each, and goal s score 6 points each.

For full marks, explain how you found your solution. [4 marks]
Solution.
Let . . . B = the number of goals kicked by the Beagles,
b = the number of behinds kicked by the Beagles,
K = the number of goals kicked by the Knockers, and
k = the number of behinds kicked by the Knockers.
From the given information, we have:
B + b = 16 (3)
K + k = 14 (4)
6K + k = 6B + b + 3 (5)
B>k (6)
b>K (7)
Since B and b are integers, from (6) and (7) we have
B ≥k+1 (8)
b≥K +1 (9)
Adding the inequalities (8) and (9) and rearranging we get
(B + b) − (K + k) ≥ 2.
But subtracting (3) and (4) gives
(B + b) − (K + k) = 2.
So the inequalities (8) and (9) must actually be equalities:
B =k+1 (10)
b=K +1 (11)
Substituting (10) in (3) and rearranging we have
B = 15 − K (12)
Rearranging (4) we have
k = 14 − K (13)
Substituting (11), (12) and (13) in (5) we have
6K + 14 − K = 6(15 − K) + K + 1 + 3
10K = 80
K=8 (14)
Substituting (14) in (12) and (11), we have:
B=7 (15)
b=9 (16)
so that the Beagles scored
6B + b = 51 points
Note the problem can also be solved by systematically listing the
possible scores for the Beagles (17 of them) and the possible scores
for the Knockers (15 of them) and then doing a careful elimination
. . . and there are many other ways. The more satisfying solutions
are ones that set up some inequality at the beginning that reduce
the possibilities to a small list prior to doing an elimination.
Answer: 51.
Western Australian
Junior Mathematics Olympiad 2006
Team Questions 45 minutes

General instructions: Calculators are (still) not permitted.

"
Consider a garden table made of 15 square tiles "
"
in a 5 × 3 arrangement.
"
"
"
The table has a straight crack along a diagonal. "
"
Seven of the individual tiles are broken. "
"





Now consider a 6 × 4 rectangle. 

This time eight tiles are broken. 






Try some other sizes. Use the squared paper provided.

A. How many tiles get broken when an 8 × 6 table is cracked along a


diagonal?
Solution. 12 tiles get broken.

B. Give the dimensions of two different rectangular tables that get


nine tiles broken when they are cracked along a diagonal.
Note. Remember that a square is also a rectangle. Also, note that
for this and subsequent questions, an 8 × 6 table, for example, is
considered the same as a 6 × 8 table.
Solution. The possible dimensions are: 9 × 1, 7 × 3, 9 × 3, 9 × 9.

C. How many different rectangular tables can you find that get ten
tiles broken when they are cracked along a diagonal? Write down
their dimensions.
Solution. The possible dimensions are: 10 × 1, 10 × 2, 10 × 5,
10 × 10, 9 × 2, 8 × 3, 7 × 4, 6 × 5.

D. Try some square tables. Describe what happens.


Solution. The number of broken tiles is the same as the length of
the side of the square.
E. What happens when the shorter dimension of the table is 1?
Solution. Every tile is broken, i.e. the number of tiles is broken
is the length of the rectangle.

F. For what sort of dimensions does the crack go through corners of


tiles inside the rectangle?
Solution. For dimensions that have a common factor (larger than
1).

G. How many tiles are cracked when the diagonal does not go through
any corner of a tile inside the rectangle? Explain your reasoning.
Solution. If the diagonal does not go through any corner of a
tile inside the rectangle, a tile is cracked when and only when the
diagonal enters a new column or enters a new row.
Say the table has m rows and n columns. The first tile broken is
in the first row and the first column. Then there are m − 1 further
rows and n − 1 further columns. Therefore the number of tiles
cracked is 1 + (m − 1) + (n − 1) = n + m − 1.

H. Predict the number of broken tiles in a 56 × 32 rectangle.


Solution. Remove the highest common factor 8. Then consider a
7×4 rectangle which has 7+4−1 = 10 broken tiles. Reintroduce the
removed common factor and the number of broken tiles is 10 × 8 =
80.
Alternatively: In general, the number of cracked tiles in a table
with dimensions m × n is m + n − hcf(m, n) where hcf(m, n) counts
the number of times the diagonal enters a new column and new
row at the same time, which, if we think of the ‘first’ tile broken
as being the bottom lefthand tile, is the number of times the crack
goes through the bottom lefthand corner of a tile. So for a 56 × 32
rectangle, 56 + 32 − hcf(56, 32) = 56 + 32 − 8 = 80 tiles are broken.

I. Explain how you can predict the number of broken tiles in any size
of table.
Solution. If the two dimensions have no common factor, add the
two numbers and subtract 1. If the two dimensions do have a
common factor, remove the highest common factor), consider the
reduced table as above, then multiply the result by the removed
HCF. Algebraically, the number of tiles broken is
 m n 
+ − 1 hcf(m, n) = m + n − hcf(m, n).
hcf(m, n) hcf(m, n)
Alternatively: use the general argument in H to obtain the gen-
eral formula m + n − hcf(m, n).
Western Australian
Junior Mathematics Olympiad 2007
Individual Questions 100 minutes

General instructions: Each solution in this part is a positive integer


less than 100. No working is needed for Questions 1 to 9. Calculators
are not permitted. Write your answers on the answer sheet provided.

1. I’m a two digit number. I’m one less than a multiple of 8 and three
less than a multiple of seven. What is the least number I could be?
[1 mark]

2. The diagram, which is not drawn to scale,


6 9
shows a rectangle divided by a horizontal
and a vertical line into four rectangles. The
areas of three of them are shown. What is 8
the area of the whole rectangle?
[2 marks]

3. From a group of girls and boys, fifteen girls depart, leaving twice
as many boys as girls. Then 45 boys depart, leaving five times as
many girls as boys. How many girls were there originally?
[2 marks]

4. A regular pentagon has five diagonals and they are all of the one
length. A regular hexagon has nine diagonals and they are of two
different lengths.

If we consider all of the diagonals of a regular polygon which has


twenty sides, how many different lengths will there be? [2 marks]
5. There are three cards on a table, each marked with a positive whole
number. Alice says “The sum of these two is 54”. Bill points to
another pair of cards and says “The sum of these is 41”. Finally,
Cyril points to another pair and says “The sum of these two is 33”.
What is the sum of all three cards? [2 marks]

6. Given that a, b, c and d are positive integers with a < 2b, b < 3c,
c < 4d and d < 5, what is the largest possible value of a? [3 marks]

7. Gwen has four children, one is a teenager (13 to 19 years old) and
the product of their ages is 1848. How old is the teenager?
[3 marks]

8. An arithmetic progression is a sequence of numbers such that the


difference of any two successive numbers is a constant. For example,
3, 5, 7 is an arithmetic progression of three numbers, with common
difference 2. Also, 3, 5, 7 are prime. The sum of the numbers is
3 + 5 + 7 = 15, which is the lowest possible sum for an arithmetic
progression of primes of length 3.
Find an arithmetic progression of four numbers, all of which are
prime, which has the lowest sum of any arithmetic progression of
primes of length 4. What is the sum of your four numbers?
[3 marks]

9. The picture shows a board of nails on a 1 cm grid.


Jane wants to put rubber bands around some of
the nails to make squares. Two such squares are
shown in the diagram (the larger square has side
length 2 cm). How many square centimetres is the
total area of all the squares she can make?
[3 marks]

10. For this question you must show working to all parts.
Two snails, Alfa and Romeo, both set out at the same time to go
along the same road from X to Y . Alfa crawled at a constant speed
of 12 m/h (metres per hour) till he reached Y . Romeo started out
at 8 m/h but after two hours he realised he was falling behind so
hitched a ride on a passing turtle called Toyota, who was on her
way to Y at a constant speed of 20 m/h. Toyota and Romeo soon
caught up with Alfa and two hours after doing so reached Y .
(a) How many hours after leaving X did it take Romeo to catch
Alfa?
(b) How many hours after starting out from X did Romeo reach Y ?
(c) How many metres is it from X to Y ?
[4 marks]
Individual Questions Solutions
1. Answer: 39. The number has the form 8n − 1 and so belongs to
the set {7, 15, 23, 31, 39, 47, . . . }. It also has the form 7n − 3 and so
belongs to {4, 11, 18, 25, 32, 39, 46, . . . }. So it must be 39.
2. Answer: 35. The rectangles on the left have the same width, so
their areas are proportional to their heights, namely 8 to 6. The
rectangles on the right also have areas proportional to their heights,
so the unknown area is 9 × 8/6 = 12, and the area of the whole
rectangle is 6 + 9 + 8 + 12 = 35.
3. Answer: 40. Say there were g girls and b boys originally. Then
b = 2(g − 15), after the first departure (1)
g − 15 = 5(b − 45), after the second departure (2)

∴ g − 15 = 5 2(g − 15) − 45 , substituting for b from (1) in (2)
∴ 5 × 45 = 9(g − 15)
5 × 5 = g − 15
g = 40.
4. Answer: 9. If we think about the diagonals starting at vertex num-
ber 1 we see that they increase from the shortest diagonal, from
vertex 1 to 3, to the longest, from vertex 1 to vertex 11, then de-
crease again, in a symmetric fashion, down to the diagonal from
vertex 1 to vertex 18. Thus there are 9 different lengths.
5. Answer: 64. If the numbers on the cards are a, b and c then we get
the system of equations
a + b = 54 (3)
a + c = 41 (4)
b + c = 33 (5)
∴ 2(a + b + c) = 54 + 41 + 33 = 128, adding (3), (4) and (5)
∴ a + b + c = 64.
6. Answer: 87. The largest possible value of d is 4 = 5 − 1. So the
largest possible value of c is 15 = 4 × 4 − 1. So the largest possible
value of b is 44 = 3 × 15 − 1. Hence the largest possible value of a
is 87 = 2 × 44 − 1.
7. Answer: 14. We have the factorisation 1848 = 2 × 2 × 2 × 3 × 7 × 11.
The only combination of factors yielding a ‘teen’ is 2 × 7, so the
teenager is 14. (There is more than one possibility for the other
three ages!)
8. Answer: 56. The arithmetic progression 5, 11, 17, 23 consists of
primes, has sum 5 + 11 + 17 + 23 = 56 and has common difference
6. There is no arithmetic progression of length 4 starting at 3 and
with common difference 2, 4 or 6. There is none starting at 5 with
common difference 2 or 4. If there is one starting at 7 with smaller
sum it must have common difference 2 or 4, but no such progressions
exist. There are none starting at 11 with difference 2 or 4, nor 13.
Since 56/4 = 14, there can’t be an arithmetic progression with
smaller sum starting with a prime greater than 13. Thus 5, 11, 17,
23 has the smallest sum (56) of an arithmetic progression of primes
of length 4.
9. Answer: 52. There are nine 1 × 1 squares with area 1, four√ 2× √2
squares with area 4, one 3 ×√3 square√ with area 9, four 2 × 2
squares with area 2 and two 5 × 5 squares with area 5. So the
total area is 9 × 1 + 4 × 4 + 1 × 9 + 4 × 2 + 2 × 5 = 52.
10. (a) Romeo covers 16 m in the first two hours. Suppose Romeo
and Toyota overtake Alfa t hours later. Then they have travelled
16 + 20t m. meanwhile, Alfa has travelled 12(2 + t) m. Hence
16 + 20t = 24 + 12t.
So t = 1. Hence it took a total of 3 hours to catch Alfa.
(b) Romeo travelled 2 hours on his own and 3 hours on Toyota’s
back, a total of 5 hours.
(c) Romeo travelled 16 m on his own and 60 m on Toyota’s back, a
total of 76 m.
Western Australian
Junior Mathematics Olympiad 2007
Team Questions 45 minutes

General instructions: Calculators are (still) not permitted.

A teacher has a class of 100 students whom he numbers from 1 to 100.


He has given each a t-shirt showing his or her number, and taken them
to a very long corridor with 100 doors. The doors are also numbered
from 1 to 100. When a student goes down the corridor for each door
whose number is divisible by the number on his/her t-shirt, he/she
either closes the door if it is open or opens the door if it is closed.
Students don’t touch doors with numbers that are not divisible by
their t-shirt numbers.
For example, if the doors are all closed to start with then when
student 40 goes down the corridor he will open doors 40 and 80. If
student 80 follows him she will close door 80.

A. If all the doors are closed, and students 7, 28 and 84 go down the
corridor, which doors will be open?
B. All the doors are closed except number 42. Which students should
he send down to get all the doors closed?
C. Suppose all the doors are closed and all the students go down the
corridor. Which of the doors 49, 51 and 53 will be open?
D. If all the doors are closed, and students 1, 2, 4, 8, 16, 32 and 64 go
down the corridor, explain how you can predict which doors will
be closed?
E. If all the doors are closed, and all the students go down the corridor,
which doors will be open? Explain your answer – not just by saying
which doors are open and which are closed.
F. The doors from 1 to 49 are closed but somebody has left all the
others open. Which students should he send down to ensure all the
doors are closed?
G. Now the doors from 1 to 49 are open and the rest are closed.Which
students should he send down to get all the doors closed?
H. All the doors were closed but the Number 1 student has just walked
down the corridor and gone home. The teacher now has no way
of closing door number 1. But is it possible to use the remaining
students to close all the other doors? Explain your reasoning.
Team Questions Solutions
A. 7, 14, 21, 35, 42, 49, 63, 70, 77, 84, 91, 98. [4 marks]
B. 42 and 84. [4 marks]
C. Only 49 will be open, because 49 has 3 factors, i.e. it is moved by
students 1, 7, 49; and 51 and 53 will be closed because they have 4
and 2 factors, respectively. [6 marks]
D. Doors 2, 6, 8, 10, 14, 18, 22, 24, 26, 30, 32, 34, 38, 40, 42, 46, 50,
54, 56, 58, 62, 66, 70, 72, 74, 78, 82, 86, 88, 90, 94, 96, 98 will be
closed. To decide whether a given door is closed or not find the
highest power of 2 which divides the door number. If this is an odd
power of 2 (2, 8 or 32) then the door will be closed. Otherwise it
will be open. [7 marks]
E. The doors 1, 4, 9, 16, 25, 36, 49, 64, 81 and 100 be open because
these are perfect squares and such numbers have an odd number
of factors, and so will be opened or closed by an odd number of
students. [6 marks]
F. 50, 51, 52, . . . , 99 (not 100). [6 marks]
G. 1 and 50, 51, 52, . . . , 99. [4 marks]
H. Yes, send down the student whose number is that of the first open
door, and continue doing this till all doors other than 1 are closed.
This is the same as sending all students except those with square
numbers on their shirts. [8 marks]
Western Australian
Junior Mathematics Olympiad 2008
Individual Questions 100 minutes

General instructions: Each solution in this part is a positive integer


less than 100. No working is needed for Questions 1 to 9. Calculators
are not permitted. Write your answers on the answer sheet provided.

1. Below are three different views of a child’s building block and a sin-
gle view of a different block.

1 2 3 4
Which is the different block? [1 mark]

2. Some horses and some jockeys are in a stable. In all, there are 71
heads and 228 legs. How many jockeys are in the stable? [1 mark]

3. Four friends go fishing and catch a total of 11 fish. Each person


caught at least one fish. The following five statements each have a
label from 1 to 16. What is the sum of the labels of all the statements
which must be true?
1: One person caught exactly 2 fish.
2: One person caught exactly 3 fish.
4: At least one person caught fewer than 3 fish.
8: At least one person caught more than 3 fish.
16: Two people each caught more than 1 fish. [2 marks]

4. There are four throwers in the shot put final at the Olympic games.
The distance thrown by the second thrower is 2% less than the first
thrower while the third thrower achieves a distance 20% greater
than the first thrower. The fourth thrower throws it 10% further
than the third person. If the total distance of the four throwers is
90 m how many metres did the first thrower throw the shot put?
[2 marks]
5. A cube of side 7 cm is painted green all over, then cut into cubes of
side 1 cm.
How many of these small cubes have exactly 2 faces painted green?
[2 marks]

6. Ann is four times as old as Mary was when Ann was as old as Mary
is now. Furthermore, Ann is twice as old as Mary was when Ann
was six years older than Mary is now. How old is Ann? [3 marks]

7. A barrel contains a number of blue balls and a number of green balls


which you take out one by one. Each time you take out a blue ball
somebody puts 100 frogs into the barrel and each time you take out
a green ball the person puts in 72 frogs. Finally, when you have
removed all the balls, you find there are 2008 frogs in the barrel.
How many green balls were there in the barrel initially? [3 marks]

8. In the figure, B is the M A


mid-point of AD, C is the F E
mid-point of DE, A is the
mid-point of EF , and M
is the midpoint of AF . B C
If the area of 4AM B is
6 cm2 , how many cm2 is the
remaining area of 4DEF ? D [3 marks]

9. Five grandmothers go to a cafe to eat cake. The cafe sells 4 different


types of cake. Each grandmother chooses two different cakes. They
find their bills are for $6, $9, $11, $12 and $15.
The next day I go to the cafe and buy one of each type of cake.
How much do I pay? [4 marks]

10. For full marks, explain how you found your solution.
A square is divided into three x
pieces of equal area as shown. The
distance between the parallel lines is
1 cm. What is the area of the square
in cm2 .
z
1

y [4 marks]
Western Australian
Junior Mathematics Olympiad 2008
Team Questions 45 minutes

General instructions: Calculators are (still) not permitted.

How to multiply on Titania


The Titans are an intelligent race who live on the planet Titania. They
use the same numerals as us for the integers and their addition and sub-
traction is the same as ours. However, they don’t use multiplication.
Instead of ×, they have an operation called ‘star’ which has the follow-
ing properties in common with ×:
For all integers a, b and c,
• a ∗ c = c ∗ a and
• (a ∗ b) ∗ c = a ∗ (b ∗ c).
However the other properties of star are completely different:
(i) For all integers a, a ∗ 0 = a, and
(ii) For all integers a and b, a ∗ (b + 1) = (a ∗ b) + (1 − a), e.g.
7 ∗ 2 = (7 ∗ 1) + (−6).

A. Copy and complete the table shown. ∗ 0 1 2 3 4


The entry in the row labelled by a
0 0 1 2 3 4
and the column labelled by b should
1 1
be a ∗ b, where each of a, b range over
0, 1, . . . , 4. 2
Some entries have been filled in to get 3
you started. 4

B. Show that for every integer a, a ∗ 1 = 1.

C. Find (−4) ∗ (−3).

D. Show that for any integer a, 4 ∗ a = 4 − 3a.

E. Show that for all integers a and positive integers b, a∗b = a+b−ab.
Hint. Try a = 5 and try to generalise your argument.
F. Show how to express a ∗ b, for any (positive, negative or zero)
integers a, b, in terms of our addition and multiplication.

G. The Titans also use powers by defining a∗n to be


a ∗ a ∗ · · · ∗ a, where a appears n times,
for n a positive integer, e.g. a∗1 = a, a∗2 = a ∗ a, etc.
Find (2∗3 ) ∗ (3∗2 ) and (3∗3 )∗3 .

H. Write the Titans’ power operation a∗n in terms of our operations,


where n is a positive integer and a is any integer.
Hint. To start, try a∗1 , a∗2 , a∗3 and look for a pattern.
Individual Questions Solutions
1. Answer: 3. The views 1, 2 and 4 can be explained thus: the star
face is opposite the square face, and the pentagon face is opposite
the triangle (the face opposite the octagon is unknown). Then 2 is
obtained from 1 by rotating in an axis perpendicular to the centre
of the octagon face so that the triangle face becomes the front face.
And 4 is obtained by flipping 1 upside down and then rotating in
the vertical axis so that the octagon is the front face. Flipping 2 so
that the octagon is on the top and the star on the right, the triangle
face should be behind and the pentagon at the front. So 3 is the
different block. [1 mark]

2. Answer: 28. Let j be the number of jockeys and let h be the number
of horses. Then
j + h = 71 =⇒ 2j + 2h = 142
2j + 4h = 228 =⇒ j + 2h = 114
=⇒ j = 142 − 114
= 28
Thus the number of jockeys is 28. [1 mark]

3. Answer: 4. The example: 3 of the friends caught 1 fish and 1 caught


8 fish meets the criteria, so that the statements labelled 1, 2 and 16
need not be true.
Also, the example: 3 of the friends caught 3 fish and 1 caught 2 fish,
shows that the statement labelled 8 need not be true.
Now, suppose for a contradiction that the statement labelled 4 is
false, then all 4 friends caught 3 or more fish, which implies there
are 12 or more fish, but there are only 11 fish (contradiction). Thus,
at least one friend caught fewer than 3 fish. Hence the statement
labelled 4 must be true, and since this is the only statement that
must be true, the sum of such labels is 4. [2 marks]

4. Answer: 20. Let x be the distance thrown by the first person in


metres. Then the second throws it 0.98x, the third, 1.2x and the
fourth 1.32x. Thus 4.5x = 90 and so x = 20. [2 marks]

5. Answer: 60. The cube has 12 edges. Along each of those edges of
the 7 cm cube, 5 will have exactly two faces painted green. So there
are 12 × 5 = 60 such cubes. [2 marks]
6. Answer: 24. Let Ann’s age be a and Mary’s age be m, and write
a1 , a2 and m1 , m2 be their ages at the two other times. Write
a1 = a+k (so that m1 = m+k), and a2 = a+` (so that m2 = m+`.
Then rewriting the statements with their ages, we have:
Ann [is a and] is four times as old as Mary was [when she
was m1 and] when Ann [was a1 and] was as old as Mary is
now [namely m]. Furthermore, Ann [is a and] is twice as
old as Mary [was when she was m2 which] was when Ann
[was a2 and] was six years older than Mary is now [namely
m].

Thus from the given information we have:

a = 4m1 =⇒ a = 4(m + k) =⇒ a − 4m − 4k = 0 (1)


a1 = m =⇒ a + k = m =⇒ 4a − 4m + 4k = 0 (2)
a = 2m2 =⇒ a = 2(m + `) =⇒ a − 2m − 2` = 0 (3)
a2 = 6 + m =⇒ a + ` = 6 + m =⇒ 2a − 2m + 2` = 12 (4)

Firstly, we eliminate k and `:

5a − 8m = 0, (1) + (2) (5)


3a − 4m = 12, (3) + (4) (6)
∴ a = 24, 2 · (6) − (5)
5
Hence, Ann is 24 (and Mary is 8
· 24 = 15). [3 marks]

7. Answer: 14. Say we started with b blue balls and g green balls. So
we must find integer solutions to the equation

100b + 72g = 2008.

Dividing through by 4 and rearranging gives

18g = 502 − 25b.

We now try values of b until we find one that makes the right hand
side divisible by 18. This happens when b = 10 since 502−10×25 =
252 = 14 × 18. So he had 14 green balls. [3 marks]

8. Answer: 42. Write (4XY Z) for the ‘area of 4XY Z’ and let S =
(4ABC). Then 4ABC and 4DBC have a common altitude to C.
Hence
(4DBC) = (4ABC) = S
Similarly, (4ECA) = (4DCA) = 2S
(4F AD) = (4EAD) = 4S
AM 1 AB
= =
AF 2 AD
∠M AB = ∠F AD
∴ 4M AB ∼ 4F AD, by the Similar 4s SAS Rule
(4F AD)
∴ (4M AB) = =S
22
Now S = 6 and (4DEF ) − (4AM B) = 7S = 42. [3 marks]

9. Answer: 21. Say the cakes have prices a, b, c and


 d. The possible
totals for the price of two cakes are the 6 = 42 sums a + b, c + d,
a + c, b + d, a + d, b + c. Since the grandmothers all paid different
amounts the amounts they paid are 5 of these 6 sums. Notice that
the sum of the first and second pairs is a + b + c + d, so is the sum
of the third and fourth, so is the sum of the fifth and sixth. This
means that two pairs of grandmothers’ bills have the same total.
The totals are:
6 + 9 = 15
6 + 11 = 17
6 + 12 = 18
6 + 15 = 21
9 + 11 = 20
9 + 12 = 21
9 + 15 = 24
11 + 12 = 23
11 + 15 = 26
12 + 15 = 27

We see that there is only one pair of equal sums: 6 + 15 = 21 and


9 + 12 = 21 so a + b + c + d = 21, which is the amount I pay for my
4 cakes. [4 marks]
10. Answer: 13. With x, y and z as shown, the middle strip is a paral-
lelogram made up of two congruent triangles of height x and base
y, and this is a third of the total area, i.e.
xy = 13 x2
∴ y = 13 x
By Pythagoras’ Theorem,
z 2 = x2 + (x − y)2
= x2 + ( 23 x)2
13 2
= 9
x
1

∴z= 3
13 x.
Looking at the two congruent triangles making up the parallelogram
another way: they have height 1 and base z, i.e. the area of the
parallelogram is also given by:

1 × z = 1 × 13 13 x = 13 x2

∴ 13 = x
Thus, the area of the square is x2 = 13 cm2 . [4 marks]
Team Questions Solutions
How to multiply on Titania

A.
∗ 0 1 2 3 4
0 0 1 2 3 4
1 1 1 1 1 1
2 2 1 0 −1 −2
3 3 1 −1 −3 −5
4 4 1 −2 −5 −8 [8 marks]

B. a ∗ 1 = a ∗ 0 + 1 − a, by (ii)
= a + 1 − a, by (i)
=1 [3 marks]

C. Answer: −19.
Rearranging (ii) we have
a ∗ b = a ∗ (b + 1) − (1 − a)
Hence
(−4) ∗ (−3) = (−4) ∗ (−2) − (1 − −4)
= (−4) ∗ (−1) − (1 − −4) − (1 − −4)
= (−4) ∗ (−1) − 2(1 − −4)
= −4 ∗ 0 − 3(1 − −4)
= −4 − 15, by (i)
= −19. [4 marks]

D. 4∗a=a∗4
=a∗3+1−a
= a ∗ 2 + 2(1 − a)
= a ∗ 1 + 3(1 − a)
= a ∗ 0 + 4(1 − a)
= a + 4 − 4a
= 4 − 3a. [5 marks]
E.
a ∗ b = a ∗ (b − 1) + (1 − a)
= a ∗ (b − 2) + (1 − a) + (1 − a)
..
.
= a ∗ (b − k) + k(1 − a)
= a ∗ 0 + b(1 − a)
= a + b − ab.
[6 marks]

F. Observe that

a∗0=a
= a + 0 − a · 0.

i.e. for b = 0, the formula

a ∗ b = a + b − ab

still works. Rearranging, a ∗ (b + 1) = (a ∗ b) + (1 − a), we have

a ∗ b = a ∗ (b + 1) − (1 − a).

Suppose b = −β, where β > 0. Then

a ∗ −β = a ∗ (−β + 1) − (1 − a)
= a ∗ (−β + 2) − (1 − a) − (1 − a)
..
.
= a ∗ (−β + k) − k(1 − a)
= a ∗ 0 − β(1 − a)
= a + −β − a(−β)
= a + b − ab, again, since in this case b = −β.

Thus, since we already showed a ∗ b = a + b − ab, for positive b,

a ∗ b = a + b − ab, ∀a, b ∈ Z.
[6 marks]
G. Answer: (2∗3 ) ∗ (3∗2 ) = 5, (3∗3 )∗3 = 513.

2∗3 = (2 ∗ 2) ∗ 2
= 0 ∗ 2 = 2, from table in A
3∗2 = 3 ∗ 3 = −3, from table in A
(2∗3 ) ∗ (3∗2 ) = 2 ∗ (−3)
= 2 + −3 − 2 · −3, from rule in E
=5
3∗3 = (3 ∗ 3) ∗ 3
= (−3) ∗ 3
= −3 + 3 − −3 · 3, from rule in E
=9
(3∗3 )∗3 = (9 ∗ 9) ∗ 9
= (9 + 9 − 9 · 9) ∗ 9
= (−63) ∗ 9
= −63 + 9 − −63 · 9
= 9(−7 + 1 + 63)
= 9 · 57 = 513
[6 marks]

H. Method 1.Using the a ∗ b = a + b − ab rule, again and again.

a∗1 = a = 1 − (1 − a)
a∗2 = a + a − a · a
= 2a − a2 = 1 − (1 − a)2
a∗3 = (a + a − a · a) + a − (a + a − a · a) · a
= 4a − 6a2 + 4a3 − a4
..
.
   
n 2 k+1 n
a∗n = na − a + · · · + (−1) ak + · · · + (−1)n+1 an
2 k
= 1 − (1 − a)n
Method 2.
a∗1 = a = 1 − (1 − a)
a ∗ b = a + b − ab
= 1 − (1 + ab − a − b)
= 1 − (1 − a)(1 − b)
∴ a∗2 = a ∗ a
= 1 − (1 − a)(1 − a) = 1 − (1 − a)2
a∗3 = (a ∗ a) ∗ a
 
2
= 1 − 1 − 1 − (1 − a) (1 − a)
= 1 − (1 − a)2 (1 − a) = 1 − (1 − a)3
..
.
a∗(k+1) = a∗k ∗ a
 
= 1 − 1 − 1 − (1 − a)k (1 − a)
= 1 − (1 − a)k (1 − a) = 1 − (1 − a)k+1
∴ a∗n = 1 − (1 − a)n [7 marks]
Western Australian
Junior Mathematics Olympiad 2009
Individual Questions 100 minutes

General instructions: Each solution in this part is a positive integer


less than 100. No working is needed for Questions 1 to 9. Calculators
are not permitted. Write your answers on the answer sheet provided.

1. Evaluate
55 − 52
√ .
55 − 54
[1 mark]

2. From each vertex of a cube, we remove a small cube whose side


length is one-quarter of the side length of the original cube.
How many edges does the resulting solid have? [1 mark]

3. A certain 2-digit number x has the property that if we put a 2 before


it and a 9 afterwards we get a 4-digit number equal to 59 times x.
What is x? [2 marks]

4. What is the units digit of 22009 × 32009 × 62009 ? [2 marks]

5. At a pharmacy, you can get disinfectant at different concentrations


of alcohol. For instance, a concentration of 60% alcohol means it
has 60% pure alcohol and 40% pure water. The pharmacist makes
a mix with 53 litres of alcohol at 90% and 51 litres of alcohol at 50%.
How many percent is the concentration of that mix? [2 marks]

6. If we arrange the 5 letters A, B, C, D and E in different ways we


can make 120 different “words”. Suppose we list these words in
alphabetical order and number them from 1 to 120. So ABCDE
gets number 1 and EDCBA gets number 120.
What is the number for DECAB? [3 marks]
7. Every station on the Metropolis railway sells tickets to every other
station. Each station has one set of tickets for each other station.
When it added some (more than one) new stations, 46 additional
sets of tickets had to be printed.
How many stations were there initially? [3 marks]

8. At a shop, Alice bought a hat for $32 and a certain number of hair
clips at $4 each. The average price of Alice’s purchases (in dollars)
is an integer.
What is the maximum number of hair clips that Alice could have
bought? [3 marks]

9. The interior angles of a convex polygon form an arithmetic sequence:


143◦ , 145◦ , 147◦ , . . . .
How many sides does the polygon have? [4 marks]

10. For full marks, explain how you found your solution.
A square ABCD has area 64 cm2 . Let M be the midpoint of BC,
let d be the perpendicular bisector of AM , and let d meet CD at F .
How many cm2 is the area of the triangle AM F ?
D F C

A B
[4 marks]
Western Australian
Junior Mathematics Olympiad 2009
Team Questions 45 minutes

General instructions: Calculators are (still) not permitted.

Crazy Computers
Rebecca has an old Lemon brand computer which has a defective key-
board. When she types L the letters LOM appear on the screen, and
when she types M she gets OL and when she types O she gets M . We
will abbreviate this to:
Lemon Computer: L → LOM , M → OL, O → M.
So if she types OLO, she gets M LOM M .

A. What will she get on the screen if she presses the Enter key to get
to a new line and then types M LOM M ?

B. Tom has a more modern Raincoat computer which also has a broken
keyboard. Typing R puts RS on the screen and S puts R.
Raincoat: R → RS, S → R.
If Tom types R to give one line of the screen, types in this line to
get a second line and so on until he has 5 lines, what will the last
line be? (Be careful – the first line on the screen is RS not R.)

C. If Tom kept going till he had 12 lines, how many letters would there
be in the final line? Try to calculate this without writing down the
final line of letters (which is quite long).

D. Sarah’s computer uses software produced by the giant Megafloppy


Corporation, and is as defective as the others. If she types in H to
get a first line on the screen, then types that line in to get a second
line, then types that to get a third line, she finds the third line is
HGGHGHHG.
Assuming that only the G and H keys are faulty, what would she
get if she deleted everything and then typed G?

In the next question, Rebecca again uses her Lemon computer.


E. If Rebecca starts by typing O on her Lemon and continues until
there are 6 lines on the screen, how many Os will there be in the
last line?
For full marks you must show how to calculate this without actually
writing down the final line.

F. Ben has a Super-Useless Lap Bottom computer, which has mysteri-


ous problems with the letters U , X and Z of its keyboard. You will
need to discover its rule by describing what should replace each box
in answering this question (the size of the boxes does not indicate
the number of replacement letters):
Lap Bottom: U → ,X→ ,Z→ .
Ben started by entering the letter U , and then like the others en-
tered what he saw on the screen onto the next line; doing this 4
times he finally obtained:
U ZU U XU ZU U ZU ZXU ZU U XU ZU U ZU U XU ZU U XZXU ZU U XU ZU U ZU ZXU ZU U XU ZU
What letter(s) replace each box above? Explain how you got your
answer.
Below, we repeat the above sequence of letters several times, in the
hope it might be helpful in your scratchwork for this problem.

U ZU U XU ZU U ZU ZXU ZU U XU ZU U ZU U XU ZU U XZXU ZU U XU ZU U ZU ZXU ZU U XU ZU

U ZU U XU ZU U ZU ZXU ZU U XU ZU U ZU U XU ZU U XZXU ZU U XU ZU U ZU ZXU ZU U XU ZU

U ZU U XU ZU U ZU ZXU ZU U XU ZU U ZU U XU ZU U XZXU ZU U XU ZU U ZU ZXU ZU U XU ZU

U ZU U XU ZU U ZU ZXU ZU U XU ZU U ZU U XU ZU U XZXU ZU U XU ZU U ZU ZXU ZU U XU ZU

U ZU U XU ZU U ZU ZXU ZU U XU ZU U ZU U XU ZU U XZXU ZU U XU ZU U ZU ZXU ZU U XU ZU

U ZU U XU ZU U ZU ZXU ZU U XU ZU U ZU U XU ZU U XZXU ZU U XU ZU U ZU ZXU ZU U XU ZU

U ZU U XU ZU U ZU ZXU ZU U XU ZU U ZU U XU ZU U XZXU ZU U XU ZU U ZU ZXU ZU U XU ZU

U ZU U XU ZU U ZU ZXU ZU U XU ZU U ZU U XU ZU U XZXU ZU U XU ZU U ZU ZXU ZU U XU ZU


Individual Questions Solutions
1. Answer: 62.
55 − 52 52 (53 − 1)
√ = 2√
55 − 54 5 5−1
124
= = 62
2
[1 mark]

2. Answer: 84. Initially there are 2 · 4 + 4 = 12 edges. By removing


a small cube from a vertex (of which there are 8), we increase the
number of edges by 12 − 3 = 9. Hence, the resulting solid has
12 + 8 · 9 = 84 edges.
[1 mark]

3. Answer: 41. Represent the 2-digit number x by ∗#. Putting 2


before it and a 9 after it, we get
2∗#9 = 2009 + ∗#0
= 2009 + 10x.
We are told that this number is 59x. Thus we have
2009 + 10x = 59x
2009 = 49x
41 = x.
[2 marks]

4. Answer: 6. Since
22009 × 32009 × 62009 = 62009·2
is just a power of 6 and 6 × 6 = 36 also ends in 6, any power of 6
ends in 6. So the answer is 6.
Alternative. Observe that
62 = 36 ≡ 6 (mod 10)
n
∴6 ≡6 (mod 10) for any integer n ≥ 1
2009 2009 2009 2009·2
∴2 ×3 ×6 =6
≡6 (mod 10)
Hence, the last digit of 22009 × 32009 × 62009 is 6. [2 marks]
5. Answer: 80. The new concentration is the the total volume of alco-
hol over the total volume of liquid expressed as a percentage:
3 90 1 50
total volume of alcohol · + ·
= 5 100 5 100
total volume 3 1
+
5 5
90 50
3· +1·
= 100 100
3+1
10(3 · 9 + 1 · 5)
=
4 · 100
10(27 + 5)
=
4 · 100
80
= = 80%
100
So the number of percent of the new concentration is 80. [2 marks]

6. Answer: 95. There are 120/5 = 24 words beginning with A, 24


beginning with B and 24 beginning with C. These all come before
DECAB. Of those beginning with D there are 24/4 = 6 beginning
with DA, 6 beginning with DB and 6 beginning with DC. These
also come before DECAB. Those beginning with DE go DEABC,
DEACB, DEBAC, DEBCA and DECAB. There are 5 of these.
So DECAB’s number is 3 × 24 + 3 × 6 + 5 = 95.
Alternative. There’s less counting if one starts from the other end.
There are 24 words beginning with E. Then DECBA is the last word
beginning with D, and the one before it is DECAB. So DECAB’s
number is 120 minus the number that follow it, i.e. 120 − (24 + 1) =
95. [3 marks]

7. Answer: 11. If y stations are added to x already existing stations,


each new station will require (x + y − 1) sets of tickets; for y new
stations this is y(x + y − 1) sets. Each old station needs y sets. So:
y(x + y − 1) + xy = 46
∴ y(2x + y − 1) = 46.
Thus y must be a positive integer which is a factor of 46, i.e. it is
1, 2, 23, or 46. But y > 1, and y = 23 or y = 46 imply x < 0.
∴ y = 2, x = 11. Therefore there were 11 old stations. [3 marks]
8. Answer: 27. Let x be the number of hair clips Alice bought. Then
the total of her purchases is:
4x + 32
so that the average price of her purchases is
4x + 32 4(x + 1) + 28 28
= =4+ ,
x+1 x+1 x+1
which is an integer, if 28 is divisible by x + 1. Hence, x + 1 must be
one of 1, 2, 4, 7, 14 or 28 (the divisors of 28), i.e. x must be one of
0, 1, 3, 6, 13 or 27. The largest of these is 27. [3 marks]

9. Answer: 18. Let n be the number of sides of the polygon. Then,


n 
(n − 2) · 180 = 2 · 143 + (n − 1) · 2
2
180(n − 2) = n(143 + n − 1)
= n(142 + n)
n2 − 38n + 360 = 0
(n − 18)(n − 20) = 0.
So n = 18 or n = 20. Since the n-gon is convex, all its angles, in
particular, the largest, must be less than 180◦ . Now, for n = 20, the
largest angle is
143 + 19 · 2 = 181 > 180.
So, n 6= 20. On the other hand, for n = 18, the largest angle
143 + 17 · 2 = 177 < 180,
which is ok. So n = 18. [4 marks]

10. Answer: 30. Let y = F D. Since d is the perpendicular bisector of


AM , it is the locus of points equidistant from A and M .
So AF = M F .
Since the area of the square is 64 cm2 , its side length is 8 cm. Hence
applying Pythagoras’ Theorem to 4F DA and 4CF M , we have
82 + y 2 = (8 − y)2 + 42
= 82 + y 2 − 16y + 16
∴ 16y = 16
y=1
Take the parenthesising of the vertices of a figure, as a convenient
shorthand for the figure’s area, so that (XY Z) means “the area of
figure XY Z”. Then
(AM F ) = (ABCD) − (ABM ) − (F DA) − (CF M )
= 64 − 21 (8 · 4 + 8 · 1 + 4 · 7)
= 64 − 12 · 68
= 64 − 34 = 30.
So 4AM F has area 30 cm2 .
Alternative 1. Instead, let x = F C. As before, deduce AF =
M F , and that square has side length 8 cm. Applying Pythagoras’
Theorem to 4F DA and 4CF M , we have
82 + (8 − x)2 = 42 + x2
82 + 82 − 2 · 8x + x2 = 42 + x2
2 · 82 − 42 = 2 · 8x
2 · 82 − 42
∴x=
2·8
= 8 − 1 = 7.
The rest of the solution proceeds like the first solution.
Alternative 2. Since the area of the square is 64 cm2 , its side length
is 8 cm. Since M is the midpoint of BC, M B = 4 cm. Applying
Pythagoras’ Theorem to 4ABM , we have

AM = 82 + 42
√ √
= 4 22 + 12 = 4 5
Let the midpoint of AM be X, i.e. XM = XA. Then
F M 2 = F X 2 + XM 2
= F X 2 + XA2
= F A2
So, F M = F A. Now deduce x or y as above and hence deduce that
F M 2 = F A2 = 65
∴ F X 2 = F A2 − XA2

= 65 − (2 5)2
= 65 − 20 = 45
∴ (AM F ) = 12 AM · F X
√ √
= 12 · 4 5 · 45
√ √
=2 5·3 5
= 6 · 5 = 30
[4 marks]
Team Questions Solutions
Crazy Computers

A. Answer: OLLOM M OLOL. Putting a little space between the


replacement letters, we have M LOM M → OL LOM M OL OL.
[4 marks]

B. Answer: RSRRSRSRRSRRS.
R → RS (1st line)
→ RSR (2nd line)
→ RSRRS (3nd line)
→ RSRRSRSR (4th line)
→ RSRRSRSRRSRRS (5th line)
[5 marks]

C. 377. The numbers of letters increases by the number of Rs in the


line, which is the same as the number of letters in the previous line.
Let `n be the length of the nth line and let `0 = 1 be the length of
the first entered line (namely R). Then
`n+1 = `n + `n−1 , n ≥ 0,
where `0 = 1, `1 = 2. So we have:
`2 = `1 + `0 = 2 + 1 = 3
`3 = `2 + `1 = 3 + 2 = 5
`4 = `3 + `2 = 5 + 3 = 8
`5 = `4 + `3 = 8 + 5 = 13
`6 = `5 + `4 = 13 + 8 = 21
`7 = `6 + `5 = 21 + 13 = 34
`8 = `7 + `6 = 34 + 21 = 55
`9 = `8 + `7 = 55 + 34 = 89
`10 = `9 + `8 = 89 + 55 = 144
`11 = `10 + `9 = 144 + 89 = 233
`12 = `11 + `10 = 233 + 144 = 377.
So the 12th line has 377 letters.
It helps to recognise that
`n = Fn+1 ,
where Fn is the nth term of the Fibonacci sequence. [8 marks]

D. Answer: GH. One can show that HGGHGHHG arises from the
replacements H → HG, G → GH. [6 marks]

E. Answer: 11. Let `n , mn , on be the number of Ls, M s and Os,


respectively on line n, or initially input in the case when n = 0.
Then `0 = m0 = 0, o0 = 1 and for n ≥ 1,
`n = `n−1 + mn−1
mn = `n−1 + on−1
on = `n−1 + mn−1 = `n .
Representing this in a table we have
n `n = `n−1 + mn−1 mn = `n−1 + on−1 on = `n−1 + mn−1
0 0 0 1
1 0 1 0
2 1 0 1
3 1 2 1
4 3 2 3
5 5 6 5
6 11 10 11
So there are 11 Os in the sixth line. [12 marks]

F. Answer: U → U ZU, X → ZX, Z → U X. Looking at the final


line we see the following repetitions.
U ZU U XU ZU U ZU ZX U ZU U XU ZU U ZU U XU ZU U X ZX U ZU U XU ZU U ZU ZX U ZU U XU ZU
If U → U ZU then passing from the third to fourth line, we require
Z → U X. So the remaining letters must be the result of Z, i.e. we
must have X → ZX. Confirming this
U → U ZU
→ U ZU U XU ZU
→ U ZU U XU ZU U ZU ZXU ZU U XU ZU
→ U ZU U XU ZU U ZU ZXU ZU U XU ZU U ZU U XU ZU U XZXU ZU U XU ZU U ZU ZXU ZU U XU ZU
[10 marks]
Western Australian
Junior Mathematics Olympiad 2010
Individual Questions 100 minutes

General instructions: Each solution in this part is a positive integer


less than 100. No working is needed for Questions 1 to 9. Calculators
are not permitted. Write your answers on the answer sheet provided.

1. What is the sum of the prime factors of 2010? [1 mark]

2. Among the 150 students at a school, 98 play tennis, 53 play football


and 39 play both sports.
How many students play neither of the two sports? [1 mark]

3. On the line segment ABCDE, D is the midpoint of AE. The length


of BD is two-thirds the length of AB, and BC = CD.
How many percent of AE is AC? [2 marks]

4. A solid cube of side 3 cm has two 1 cm


square holes drilled through its centre,
each hole being drilled through to the op-
posite side of the cube.
How many cm2 is the total surface area of
the solid remaining?

[2 marks]

5. In the diagram, A is the centre of the circle


and ABCD is a rectangle. C
If BE = 2 cm and AD = 18 cm, how many D
centimetres is the radius of the circle?
A BE

[3 marks]
6. In a company 5 years ago, the ratio of the number of male employees
to the number of female employees was 76:100. Today, the total
number of employees has not changed but the number of female
employees has increased by 10%.
If the ratio of the number of male employees to the number of female
employees is now x : 100, what is x? [3 marks]

7. How many isosceles triangles of perimeter 113 have integer length


sides? [3 marks]

8. A big jar contains 100 balls, some red and the rest black.
Pierre and Heidi take turns removing a ball.
Each time Pierre takes out a red ball, he gets 1 dollar, and when he
takes out a black ball, he gets 4 dollars.
On the other hand, Heidi gets 2 dollars for a red ball, and 3 dollars
for a black ball.
When all the balls are gone, Pierre has $110 and Heidi has $121.
How many black balls were in the jar originally? [3 marks]

9. Now, Mark is twice as old as Naomi was when Mark was as old as
Naomi is now.
When Naomi is as old as Mark is now, together their ages will total
63 years.
What is the sum of Mark’s and Naomi’s ages now? [3 marks]

10. For full marks, explain how you found your solution.
The area of the circumcircle of an equilateral triangle is 12π.
(The circumcircle of a triangle is the circle that passes through each
vertex of the triangle.)
What is the perimeter of the triangle? [4 marks]
Western Australian
Junior Mathematics Olympiad 2010
Team Questions 45 minutes

General instructions: Calculators are (still) not permitted.

Look-and-Say Sequences
This is a Look-and-Say sequence:
1
11
21
1211
111221
The first row is called the seed. It is allowed to be any positive integer.
The numbers in the following rows each say what the previous row
looks like. In the example above, the first row has “one 1”; the second
row is: 11. So the third row looks at “two 1s” and says: 21. Then
the fourth row looks at “one 2 and one 1” and says: 1211. Thus all
the rows are completely determined by the seed. If a row contains
more than nine consecutive copies of the same digit, then the sequence
terminates.
The second row is called the daughter of the seed, and all the rows
after the seed are called descendants.
Note. A consequence of the given termination condition is that, for
example, the row 1111111111 has no descendant.

A. Start with the seed 2 and write the first eight rows of the resulting
Look-and-Say sequence.

B. What is the seed if the fourth row is 1213?

C. Show that if a digit appears in a row, then it also appears in all


the following rows.

D. Explain why no descendant can contain four consecutive copies of


the same digit.
E. Show that if the seed has a daughter, then the sequence never
terminates.
Hint. See the introduction for an explanation of how a Look-and-
Say sequence can terminate.

F. Find a seed that is equal to its daughter.

G. Find a seed whose daughter is its triple, where, for example, the
triple of 14 is 42.

H. What do you observe about the last digit of every row?


Prove your observation.

I. Explain why no seed has a daughter which is its double.


Individual Questions Solutions
1. Answer: 77. Factorising, 2010 we have:
2010 = 10 · 201
= 2 · 5 · 3 · 67.
So the prime factors of 2010 are 2, 3, 5 and 67, and their sum is:
2 + 3 + 5 + 67 = 77.
[1 mark]

2. Answer: 38. Let T, F be the sets of students who play tennis and
football, respectively. Then

|T ∪ F | = |T | + |F | − |T ∩ F |
= 98 + 53 − 39 .................. ........
... ..........
... ...
.....
So the number of students who play 98 .
....
.
.
... 39 53
...
.......
................ .............

neither tennis nor football, i.e. the


.........................
number in the complement of T ∪ F , 150.............
.. T F
|(T ∪ F )0 | = 150 − (98 + 53 − 39)
= 38.
So 38 students play neither sport. [1 mark]

3. Answer: 40. Let x = AB. Then BD = 32 x, so that


BC = CD = 12 BD = 31 x and
DE = AD = AB + BD = x + 23 x = 53 x
Hence, we have the following picture
1 1 5
x 3
x 3
x 3
x
z }| {z }| {z }| {z }| {
A B C D E

AC x + 13 x
∴ =
AE x + 31 x + 31 x + 53 x
4
3
x
= 10
3
x
4
=
10
= 40%.
[2 marks]
4. Answer: 68.
Area (hole-less faces) = 2 × 32 = 18
Area (holed faces) = 4 × (32 − 1) = 32
Area (holes to 1 cm in) = 4 × 4 × 12 = 16
2
Area (holes at centre) = 2 × 1 =2
∴ Total Surface Area = 68.
[2 marks]

5. Answer: 82. Let the radius be r. Then AB = r − BE = r − 2.


Using Pythagoras’ Theorem on 4ABC, we have

(r − 2)2 + 182 = r2
C
182 = r2 − (r − 2)2 D
r
= 2(r + r − 2)
A BE
= 4(r − 1)
2
9 =r−1
∴ r = 82.
[3 marks]

6. Answer: 60. Let m1 , m2 be the numbers of male employees, 5 years


ago, and now, respectively, and similarly, let f1 , f2 be the numbers
of female employees, 5 years ago, and now, respectively. Then
m1
= 0.76 (1)
f1
m1 + f1 = m2 + f2 (2)
f2 = 1.1f1 (3)
∴ m1 = 0.76f1 , by (1)
m2 = m1 + f1 − f2 , by (2)
= 0.76f1 + f1 − 1.1f1
= 0.66f1
x m2 0.66f1
∴ = =
100 f2 1.1f1
= 0.60
60
=
100
∴ x = 60
[3 marks]
7. Answer: 28. Let x be the length of the equal length sides and y the
length of the remaining side. Then x and y satisfy

1 ≤ y < 2x and 2x + y = 113


∴ 1 ≤ 113 − 2x < 2x x x
∴ 2x ≤ 112 and 113 < 4x
∴ 28 < x ≤ 56 y
So x can be any of the 28 integers between 29 and 56 (inclusive).
So there are 28 such isosceles triangles.
Note that y = 113 − 2x is odd, and each x between 29 and 56
(inclusive) does indeed give a triangle satisfying the required condi-
tions. Also note that if one had eliminated x to find bounds on y
instead (namely, 1 ≤ y < 57), one would need to take care to count
only the odd integers between 1 and 55 (inclusive). [3 marks]

8. Answer: 41. Say Pierre took out x red balls and y black balls and
Heidi took out u red balls and v black balls. We need to find y + v.
Since each took out 50 balls we have 4 simultaneous equations:
x + y = 50 (4)
u + v = 50 (5)
x + 4y = 110 (6)
2u + 3v = 121 (7)
(6) − (4) : 3y = 60
∴ y = 20
(7) − 2 · (5) : v = 21
∴ y + v = 41.
So there were 41 black balls. [3 marks]

9. Answer: 49. Let m, n be Mark’s and Naomi’s ages, now, respec-


tively. We can summarise the given information in the following
table:
Now Mark as old as When Naomi is as old
Naomi is now as Mark is now
Mark m n 63 − m
1
Naomi n 2
m m
Difference m − n n − 12 m 63 − 2m
Equating the differences we have,

m
m−n=n− = 63 − 2m
2
∴ 3m = 4n and 3m − n = 63
∴ 3n = 63
n = 21
m = 34 · 21 = 28
∴ m + n = 49.

So the sum of Mark’s and Naomi’s ages now is 49.


Alternatively, let Naomi’s age, now, be n as above, and let d be
the difference in Naomi’s and Mark’s ages, so that Mark’s age now
is n + d. Then Mark was as old as Naomi is now, d years ago, and
Naomi will be as old as Mark is now, d years from now.

Age now Age d years ago Age d years from now


Naomi n n−d n+d
Mark n+d n n + 2d

Note that the sum of Naomi’s and Mark’s ages is n + n + d = 2n + d.


From the given information, we have

n + d = 2(n − d) and
n + d + n + 2d = 63
∴ n = 3d and 2n + 3d = 63
∴ 9d = 63
d=7
∴ 2n + d = (2n + 3d) − 2d
= 63 − 14
= 49.

So again the sum of Mark’s and Naomi’s ages now is 49. [3 marks]
10. Answer: 18. Let the triangle be ABC,
with circumcentre O and circumradius B
R, and denote the foot of the altitude
dropped from B by X. Then
12π = πR2
√ √ O
R = 12 = 2 3
The medians of an equilateral triangle
are also its altitudes and concur at O C X A
(which is also the centroid and ortho-
centre of the triangle). We have

2 3 = R = OB
= 23 XB

∴ XB = 3 3

Now AX : XB : BA = 1 : 3 : 2. So
AX = 3, and hence
AB + BC + CA = 6 · 3 = 18.
A correct diagram with up to two steps of the solution is also worth
a point. [4 marks]
Team Questions Solutions
Look-and-Say Sequences

A. The sequence starting at the seed (first row) is as follows:


2
12
1112
3112
132112
1113122112
311311222112
13211321322112
[6 marks]

B. Answer: 333. If the fourth row is 1213, then there is ‘one 2 and
one 3’ in the third row, i.e. the third row is 23.
Hence the second row has ‘two 3s’, i.e. the second row is 33.
Hence the first row (the seed) has ‘three 3s’, i.e. the seed is 333.
[4 marks]

C. When you see a digit X in a row, you say ‘m Xs’, and write mX, in
the next row. In particular, if X appears in a row, then X appears
in the next row, and so on (by induction). [4 marks]

D. Suppose, for a contradiction, that XXXX appears in a descen-


dant. Then either the first X is an even positioned digit in which
case some digit precedes X, or the second X is an even positioned
digit. Either way, the descendant has a sequence Y XXX where
the Y is in an odd position (and may actually be X), saying that
the previous row has Y Xs followed by X Xs, but then the descen-
dant would actually have ZX where Z is at least X + Y (it may
be greater, if there are other adjacent Xs. So we have a contradic-
tion, and so a sequence of four Xs cannot occur in a descendant.
[6 marks]
E. The only way a sequence can terminate is if it has more than 9
consecutive copies of a digit, but D shows this cannot happen except
for the seed. [5 marks]

F. Answer: 22. If a seed consists of one digit X then its daughter is


1X, so that a 1-digit seed is necessarily different from its daughter.
So an example if it exists must have at least two digits. A seed and
its daughter must finish with the same digit. If a 2-digit seed has
different digits XY then its daughter is 1X1Y again different from
the seed. So for a 2-digit seed to be equal to its daughter it must
have equal digits XX in which case the daughter is 2X, so that we
need X = 2, and hence we have found 22 is a seed that is equal to
its daughter. [4 marks]

G. Answer: 5. First we note that the seed and its daughter end in the
same digit. If the seed consists of a single digit then the seed is X
and the daughter 1X, and we have
3 · X = 10 + X
2 · X = 10
X=5
Thus 5 is an example of such a seed. [4 marks]

H. Answer: the last digit of every row is the same. If a row finishes
with m Xs, then the next row finishes with mX. Thus, in particu-
lar, if one row finishes with X, so will the next row. Applying this
repeatedly we see every row finishes with the last digit of the seed.
[4 marks]

I. Suppose, for a contradiction, that the daughter is twice the seed.


By H, the last digit of the seed and the daughter are the same. Let
that digit be X. Then we require 2 · X to have last digit X. This
can only occur if X = 0. (Alternatively, 2 · X ≡ X (mod 10), in
which case, X ≡ 0 (mod 10).)
The last two digits of the seed cannot be 00, otherwise the last two
digits of the daughter would be X0 for a positive X, so it would
not be twice the seed
So the last two digits of the daughter are 10, which implies the seed
ends in 5 not 0, contradicting H.
So the daughter cannot be double the seed. [8 marks]
Western Australian
Junior Mathematics Olympiad 2011
Individual Questions 100 minutes

General instructions: Each solution in this part is a positive integer


less than 100. No working is needed for Questions 1 to 9. Calculators
are not permitted. Write your answers on the answer sheet provided.
In general, diagrams are provided to clarify wording only, and are not
to scale.

1. How many numbers between 1 and 100 have 5 as their smallest


prime factor? [1 mark]

2. A rhombus has sides of length 5 cm and one diagonal has size 8 cm.
How many cm2 is the area of the rhombus? [1 mark]

3. The number whose 9-digit representation is 20102011x, is divisible


by 3 and 8.
What is the digit x? [1 mark]

4. A square whose side length is 148 cm, is


divided into 4 rectangles and a smaller
square, as shown. The rectangles and
the smaller square all have the same
perimeter.
How many centimetres is the side
length of the smaller square?

[2 marks]

5. A rectangular water tank with a base of


100 cm × 50 cm originally has water in
it to a depth of 40 cm. A metal block,
of size 60 cm × 40 cm × 25 cm is then
submerged in the water, as shown.
How many cm is the water depth now?

[2 marks]
1
6. How many pairs of integers (x, y) satisfy 1 ≤ 2
x + 1 ≤ y ≤ 6?
[3 marks]

7. A tunnel is cut through a hill-


side, with a semi-circular cross- P
section. A truck of height 6 m
can just drive with its nearside
6m
wheels 1 m from the point where
the curved roof meets the hori-
zontal road surface. A 1m B
How many metres wide is the
tunnel?
[3 marks]

8. Two friends are training at the same circular track, running in op-
posite directions. Chris takes 112 seconds to run a complete lap,
but finds that he is meeting Sophie every 48 seconds.
How many seconds is Sophie taking for each lap? [3 marks]

9. On holiday in the Pacific, Julie is about to send postcards to friends


in Australia, New Zealand and New Caledonia. In the local money,
stamps for Australia cost 50c, for New Zealand 60c and for New
Caledonia 80c. Half the postcards are going to Australia and the
total cost of stamps will be $14.
How many postcards is she sending? [4 marks]

10. For full marks explain how you found your solution.
A Mexican triangle is made up of an equilateral
triangle of shaded and unshaded discs, with all
the shaded discs making up a smaller equilateral
triangle in one corner. The diagram shows a
Mexican triangle with 3 shaded discs and 12 un-
shaded discs. Find a Mexican triangle in which
there is more than 1 shaded disc, twice as many
unshaded discs as shaded discs, and fewer than
100 discs overall.
How many discs are there altogether, in the
Mexican triangle you have found?
[5 marks]
Western Australian
Junior Mathematics Olympiad 2011
Team Questions 45 minutes

General instructions: Calculators are (still) not permitted.

Fossils
To fossilise a number N , one first multiplies its digits together. Then
one does the same with the resulting number, repeating the process
until the number remaining has only one digit.
The final one-digit number is the fossil of N , except that if the final
one-digit number is zero, we say that the number N leaves no fossil.
Example.
489 gives 4 × 8 × 9 = 288
288 gives 2 × 8 × 8 = 128
128 gives 1 × 2 × 8 = 16
16 gives 1 × 6 = 6.
So, we say that 489 leaves a fossil of 6.

A. For the following numbers, find their fossil or show that they leave
no fossil.

(i) 273 (ii) 619 (iii) 333 (iv) 513

B. What is the largest 2-digit number that leaves a fossil of 4?

C. (i) What is the largest 3-digit number that leaves a fossil of 3?


(ii) What is the largest 3-digit number that leaves a fossil of 2?

D. Find the largest 3-digit number, all of whose digits are different,
that leaves a fossil (non zero).

E. Find the largest 3-digit number, all of whose digits are different,
that leaves an odd fossil.
F. Find the largest number, whose digits are all different, that leaves
an odd number for a fossil.

G. What proportion of 2-digit numbers leave no fossil? Try to find an


argument other than checking all the 2-digit numbers.

H. Among the 2-digit numbers, which fossil is the rarest. Explain


why.

I. Show that for every n the rarest fossil given by n-digit numbers is
the same as in H.
Individual Questions Solutions
1. Answer: 7. The question is equivalent to asking: how many numbers
between 1 and 100 are divisible by 5, but not by 2 or 3?
Firstly, the numbers between 1 and 100 that are divisible by 5 are
of form 5k, where k = 1, 2, . . . , 19. Writing these numbers k down
and knocking out the ones with a factor 2, we have
1, 3, 5, 7, 9, 11, 13, 15, 17, 19
and of these knocking out those that have a factor of 3, we have
1, 5, 7, 11, 13, 17, 19
(7 possibilities).
So there are 7 numbers between 1 and 100 that have 5 as their
smallest prime factor. [1 mark]

2. Answer: 24. The diagonals of a rhombus meet at right-angles. Half


the given diagonal is 4 cm. Then this half-diagonal, half the other
diagonal, x say, and a side of the rhombus form a Pythagorean triad,
i.e. 4 : x : 5 is a Pythagorean triad. Hence x = 3. Hence the area of
the rhombus is
4 · 21 · 3 · 4 = 24 cm2 .
[1 mark]

3. Answer: 2. A number is divisible by 3 if its digit sum is divisible by


3, i.e. we require
2 × (2 + 0 + 1) + 1 + x
to be divisible by 3. Hence x must be 2 more than a multiple of 3.
So x is one of 2, 5, 8.
For a number to be divisible by 8, the number formed by its last
three digits must be divisible by 8 (since 1000 = 8 × 125). However,
since a number that is divisible by 8, must at least be even, we can
exclude x being 5. So x is either 2 or 8.
Now 112 is divisible by 8, and 118 is not. So x = 2. [1 mark]

4. Answer: 74. If the smaller square has side length a and the rectan-
gles have smaller side length b, then the condition that the rectangles
and the smaller square have the same perimeter gives 2a + 4b = 4a
and hence a = 2b, so that the larger square has side length 2a = 148.
[2 marks]
5. Answer: 52.
Volume of water is 100 × 50 × 40.
Volume of block is 60 × 40 × 25.
Together,
100 × 50 × 40 + 60 × 40 × 25 = 100 × 50 × 40 + 100 × 10 × 60
= 100 × 50 × 40 + 100 × 50 × 12
= 100 × 50 × (40 + 12).

So the new depth is 40 + 12 = 52 cm. [2 marks]

6. Answer: 36. Firstly, the conditions imply 1 ≤ y ≤ 6. Also,


1 ≤ 21 x + 1 ≤ y =⇒ 0 ≤ 21 x ≤ y − 1
=⇒ 0 ≤ x ≤ 2(y − 1),
i.e. if y = n then x can be any one of 0, 1, . . . 2n − 2, which is 2n − 1
possibilities.
As y ranges over the values of 1, 2, . . . , 6 we have a total number of
6
1 + 3 + · · · + 2 · 6 − 1 = (1 + 2 · 6 − 1) · 2
= 36
possible pairs (x, y), since it is an arithmetic series (with common
difference 2). [3 marks]

7. Answer: 37. Since the tunnel is semi-circular, ∠AP B = 90◦ . Drop


a perpendicular from P to AB to meet AB at C.
Then 4ACP , 4AP B and 4P CB
are similar by the AA Rule, since P
each has a right angle (at ∠C, ∠P ×

and ∠C, respectively), 4ACP and
6
4AP B have a common angle at
∠A, and 4AP B and 4P CB have ◦ ×
a common angle at ∠B. A O C 1 B
So we have,
AC PC
=
CP CB
(P C)2
∴ AC = = 62
CB
∴ AB = AC + CB
= 62 + 1 = 37 m.
Note. There are at least two other ways to get the answer.
√ √
Alternative 1. In 4P CB we see P B = 12 + 62 = 37. Then,
since 4AP B ∼ 4P CB,
PB CB
=
AB PB
(P B)2
∴ AB = = 37.
CB
Alternative 2. Let O be the centre of the semi-circle, and note
that OP = OB = R, the radius of the semi-circle, and AB = 2R.
Now considering 4OCP , and noting OC = R − 1 we have,
(R − 1)2 + 62 = R2
R2 − 2R + 1 + 36 = R2
∴ AB = 2R = 37.
[3 marks]

8. Answer: 84. Chris completes laps in 112 seconds, and Sophie is


meeting him after only 48 seconds, then he is only completing 48/112
of a lap in the time that Sophie takes to complete the rest of the
lap, namely 64/112 of a lap. Hence, Sophie is running at a speed
equal to 64/48 = 4/3 of his own speed, and will run an entire lap in
3
4
of his time = 43 × 112 = 84 seconds. [3 marks]

9. Answer: 24. Say that Julie is sending c postcards to New Caledonia


and z to New Zealand. Then she is sending c + z to Australia. The
total cost will be 50(c + z) + 60z + 80c = 130c + 110z cents. This
must equal $14 = 1400 cents, so we have 130c + 110z = 1400, or
equivalently
13c + 11z = 140.
Observe that 13 − 11 = 2 and 10 · 13 = 130, so that we have
140 = 5(13 − 11) + 10 · 13
= 15 · 13 − 5 · 11
= (15 − 11t)13 + (−5 + 13t)11.
Hence, the general solution over the integers is
c = 15 − 11t, z = −5 + 13t,
where t ∈ Z, for which only t = 1 gives c and z positive. Thus, we
find the only solution is c = 4 and z = 8 so the total number of
stamps is 4 + 8 + 12 = 24. [4 marks]
10. Answer: 45. A triangle with n discs along the bottom contains
n(n + 1)/2 discs altogether. So if the outer triangle in a Mexican
triangle has n discs on each side and the inner triangle has m there
will be m(m + 1)/2 shaded discs and n(n + 1)/2 − m(m + 1)/2
unshaded discs. We therefore have the equation,
n(n + 1)/2 − m(m + 1)/2 = 2(m(m + 1)/2).
Expanding and simplifying gives
n2 + n = 3(m2 + m).
We calculate a few values of n2 + n till we find one which is 3 times
another (ignoring n = 1 since we’re told there is more then 1 shaded
disc, and we can stop at n = 13 since for n > 14, n(n + 1)/2 > 100):
n n2 + n
2 6
3 12
4 20
5 30
6 42
7 56
8 72
9 90
10 110
11 132
12 156
13 182
Since 90 = 3 × 30 we need m = 5 and n = 9, and the total number
of discs is 9 × (9 + 1)/2 = 45.
For larger n, either n(n + 1) is not divisible by 3 or n(n + 1)/3 is
not in the table.
Thus the solution is 45 (and it’s the only solution). [5 marks]
Team Questions Solutions
Fossils

A. Answers: 8, 0, 4, 5.
273 7−→ 2 · 7 · 3 = 42 7−→ 8,
619 7−→ 6 · 1 · 9 = 54 7−→ 20 7−→ 0,
333 7−→ 3 · 3 · 3 = 27 7−→ 14 7−→ 4,
513 7−→ 5 · 1 · 3 = 15 7−→ 5.

B. Answer: 98.
99 7−→ 9 · 9 = 81 7−→ 8,
98 7−→ 9 · 8 = 72 7−→ 14 7−→ 4.
So the largest 2-digit number that leaves a fossil of 4 is 98.
Alternatively, backtracking:
4 = 1 · 4 = 4 · 1 = 2 · 2.
14 = 7 · 2, 41 is prime, and 22 = 2 · 11 (11 is not a single digit).
Now 72 = 9 · 8 and 99 leaves a fossil of 8. So 98 is largest.

C. Answer: (i) 311 (ii) 999.


First let us discuss jargon and methodology. Thinking as in one’s
family tree, one can talk of the numbers at each stage of fossilisation
as descendants. So, for
273 7−→ 2 · 7 · 3 = 42 7−→ 8
we have 42 and 8 as descendants of 273, with the last descendant
(8 in this case) when it is non-zero being the fossil of the number.
Thinking the other way, a word that is the opposite of “descendant”
is antecedent. We could say that the fossil 8 comes from 273 and
42, or that 273 and 42 are antecedents of 273.
As we discovered in B., in terms of methodology, we may work
forwards toward the fossil, i.e. find descendants, or work backwards
from the fossil, i.e. find antecedents. In working backwards, we see
that a number can have no antecedents if it is a prime of more than
one digit, or has a prime factor of more than one digit (noting that:
if a number of more than one digit is not divisible by 2, 3, 5 or 7
then it must have a prime factor of more than one digit).
For 3: 3 = 3 · 1 = 3 · 1 · 1. (We work backwards.)
Each of the 2-digit numbers 13 and 31 is prime, and so have no
antecedents (let alone 3-digit ones).
Each of 113, 131 and 311 has no antecedents, since each is not
divisible by any of 2, 3, 5 or 7.
So 311 is the largest 3-digit number that leaves a fossil of 3.
For 2: 2 = 1 · 2 = 1 · 1 · 2. (Working forwards is best here!)
Observe that:
999 7−→ 9 · 9 · 9 = 729 7−→ 126 7−→ 12 7−→ 2.
Since 999 is the largest 3-digit number and it leaves a fossil of
2, we are done!
Alternatively (for 2), working backwards (a partial solution this
way is given, just to show it’s the wrong approach – it needs great
care to get right!).
12 = 3 · 4 = 2 · 6 = 1 · 3 · 4 = 1 · 2 · 6. So . . .
• The 2-digit antecedents of 12 are: 34 = 2 · 17, 43 (prime),
26 = 2 · 13 and 62 = 2 · 31, none of which has an antecedent.
• The 3-digit antecedents of 12 with digits 1, 3, 4 are:
134 (prime factor 67), 341 (prime factors 11, 31), 431 (prime),
none of which has an antecedent.
• The 3-digit antecedents of 12 with digits 1, 2, 6 are:
126, 162 both of which have lots!! of large antecedents, one of
which is 999. A complete discussion is omitted.
• 21 = 3 · 7 = 1 · 3 · 7. Each of 37, 73, 137, 173, 317, 371, 713, 731
is either prime or has a 2-digit prime factor (which for some
of the larger ones is determined by ruling out 2, 3, 5 and 7 as
factors), and so none has an antecedent.
• 112 = 4 · 4 · 7 = 8 · 2 · 7, 121 = 112 , 211 is prime.
Checking for possible antecedents of 447, 474, 744, 278, 287,
728, 782, 827, 872, there are none, since each has a prime factor
of more than one digit (or is in fact prime).

D. Answer: 986.
987 7−→ 9 · 8 · 7 = 504 7−→ 0.
986 7−→ 9 · 8 · 6 = 432 7−→ 24 7−→ 8.
So 986 is the largest 3-digit number with distinct digits that leaves
a fossil.

E. Answer: 975. To leave an odd fossil, we must avoid even numbers


at every stage of fossilisation, since “even” times “odd” gives an
“even”.
So the largest one conceivably possible is 975, and
975 7−→ 9 · 7 · 5 = 315 7−→ 15 7−→ 5 (non-zero).
F. Answer: 9751. We can use the digit 5, since we are not using any
even digits, so we are left, at best, possibly with 97531.
But 97531 7−→ 945 7−→ · · · 7−→ 0.
So we proceed by removing individual digits until we get a fossil
result. (Bracketing indicates removal of the digit.)
9753(1) behaves just like 97531.
975(3)1 7−→ 315 7−→ 15 7−→ 5, a fossil!
97(5)31 7−→ 189 7−→ 63 7−→ 18 7−→ 8, a fossil!
9(7)531 7−→ 135 7−→ 15 7−→ 5, again.
(9)7531 7−→ 105 7−→ 0.
So we have our answer:
We can’t get a fossil from a 5-digit number with distinct
digits, and the only 4-digit numbers with distinct digits
that leave an odd fossil are those that use the digits 9,7,5,1
or 9,5,3,1 – the largest of which is 9751.

G. Answer: 24/90 = 4/15.


The direct antecedents of 0 are 10, 20, . . . , 90 (9 of them).
Considering antecedents of 10, 20, . . . , 90:
10 ←−[ 25, 52.
20 ←−[ 45, 54.
30 ←−[ 65, 56.
40 ←−[ 85, 58.
50 to 90 have no 2-digit antecedent, since such a number would
have 5 as one digit, and at most 9 for its other digit, the product
of which is at most 45 < 50.
Thus there are 8 direct antecedents of 10, 20, . . . , 90.
Next consider the antecedents of 10, 20, . . . , 90.
25 ←−[ 55.
52 is divisible by 13, and so has no antecedent.
45 ←−[ 59, 95.
54 ←−[ 69, 96.
65 is divisible by 13, and so has no antecedent.
56 ←−[ 78, 87.
58 is divisible by 29, and so has no antecedent.
85 is divisible by 17, and so has no antecedent.
Thus we have 7 more antecedents.
Now we investigate the next level antecedents.
55 is divisible by 11, and so has no antecedent.
59 is prime, and so has no antecedent.
95 is bigger than 81, and so has no 2-digit antecedent.
69 is divisible by 23, and so has no antecedent.
96 is bigger than 81, and so has no 2-digit antecedent.
78 is divisible by 13, and so has no antecedent.
87 is bigger than 81, and so has no 2-digit antecedent.
So we have added no new 2-digit antecedents at this level and
hence we had already found all the 2-digit antecedents of 0, and in
all there are 9 + 8 + 7 = 24 out of 90.
The following is included for interest’s sake. It is not the style
of solution desired.
The table shows the fossil of the two-digit number mn (‘.’ in the
table represents “no fossil”).
mn m0 m1 m2 m3 m4 m5 m6 m7 m8 m9
1n . 1 2 3 4 5 6 7 8 9
2n . 2 4 6 8 . 2 4 6 8
3n . 3 6 9 2 5 8 2 8 4
4n . 4 8 2 6 . 8 6 6 4
5n . 5 . 5 . . . 5 . .
6n . 6 2 8 8 . 8 8 6 .
7n . 7 4 2 6 5 8 8 . 8
8n . 8 6 8 6 . 6 . 8 4
9n . 9 8 4 4 . . 8 4 8
So we have the following fossil frequencies (‘.’ in the table represents
“no fossil”).
fossil . 1 2 3 4 5 6 7 8 9
24 1 8 2 9 6 12 2 23 3
In particular, the no-fossil frequency is 24/90 = 4/15.

H. Answer: 1. The fossil 1 occurs exactly once. Observe that for any
2-digit number mn, the first product is ≤ 81 (a 2-digit number).
Since, 11 is prime and of 2 digits, it cannot occur at an intermediate
stage of the fossilisation of a number. The only way of representing
1 as the product of two 1-digit numbers is 1 = 1 · 1.
Thus, only 11 fossilises as 1.
Also, all other fossils x, (i.e. x > 1) occur at least twice, coming
from at least the 2-digit numbers 1x and x1.
Thus the rarest fossil is 1.

I. Assume n > 1. As in H., we observe that fossils k other than 1,


can be reached from at least two numbers, e.g. k1 . . . 1 and 1 . . . 1k.
Now we show that only numbers 1 . . . 1 have fossil 1.
Since representing 1 as the product of 1-digit numbers requires each
factor to be 1, this is equivalent to showing numbers of form 1 . . . 1
have no antecedents, which in turn is equivalent to showing 1 . . . 1
necessarily has a factor of more than 1 digit:
1 . . . 1 is odd and hence not divisible by 2, 4, 6 or 8.
It’s not divisible by 5 (it’s last digit is neither 5 nor 0).
If it’s divisible by 3, it’s divisible by 111 and so by the prime 37.
If it’s divisible by 9, it’s divisible by 3 and therefore 37.
Lastly, if it’s divisible by 7, it’s divisible by 111111 which is divisible
by 111 and therefore by 37.
Thus we see that 1 . . . 1 is prime of more than 1 digit or has 37 as
a factor, and hence can have no antecedents.
So only numbers 1 . . . 1 have fossil 1; hence among n-digit numbers,
n > 1, the fossil 1 is rarest, the only fossil occurring exactly once.
Western Australian
Junior Mathematics Olympiad 2012
Individual Questions 100 minutes

General instructions: Each solution in this part is a positive integer


less than 1000. No working is needed for Questions 1 to 11. Calculators
are not permitted. Write your answers on the answer sheet provided.
In general, diagrams are provided to clarify wording only, and are not
to scale.

1. A point X within a rectangle P QRS is such that XS = XR and


the area of the triangle QXR is 7 square centimetres.
How many square centimetres is the area of the rectangle? [1 mark]

2. A freeway is (in total) 16 m wide, and streetlamps are placed in


the middle of the road (along the median strip separating the two
directions of traffic). Each lamp lights a disc (i.e. the area within a
circle) with diameter 20 m.
What is the maximum number of metres between the lamps in order
that no part of the freeway is unlit? [1 mark]

3. In the picture of adjoining tiles below, tiles A to H are square. The


area of tile F is 16 square units, the area of tiles B and G are each
25 square units.
How many square metres is the total area of tile D and tile H com-
bined? [1 mark]

B
C
A
E
F

D
G

H
4. On Monday, the produce manager, Arthur Applegate, stacked the
display case with 80 lettuces. By the end of the day, some of the
lettuces had been sold. On Tuesday, Arthur surveyed the display
case and counted the lettuces that were left. He decided to add an
equal number of lettuces. (He doubled the leftovers.) By the end of
the day, he had sold the same number of lettuces as on Monday. On
Wednesday, Arthur decided to triple the number of lettuces that he
had left. At the end of the day there were no lettuces left, and it
turned out that he had sold the same number that day, as each of
the previous two days.
How many lettuces were sold each day? [2 marks]

5. A rhombus has diagonals of length 420 mm and 560 mm.


A circle is drawn inside this rhombus, touching all four sides.
How many millimetres is the circle’s radius? [2 marks]

6. Jo set off for a hike along a cross-country trail to Bluff Knoll and
returned along the same route. She started at 10.00 am and got
back at 4.00 pm, having been up and down hills and along some flat
ground too. Her speed along the flat was 4 km/h; and she managed
3 km/h up hills, and 6 km/h down hills.
What is the total number of kilometres that Jo walked? [2 marks]

7. On January 1st, Honi was given a large collection of scorpions by her


grandmother. During January, the scorpion population increases by
5%. From February 1st till the end of February, the population in-
creases by 10% of the population at the beginning of February, and
during March it increases by 20% of the population at the beginning
of March.
To the nearest whole number, by how many percent has the popu-
lation increased during the three months? [2 marks]

8. The first two digits of a certain three-digit number form a perfect


square and so do the last two digits.
If the number is divisible by 11, what is it? [2 marks]

9. A dingo starts chasing a rabbit which is 15 dingo leaps in front of


the dingo. He (the dingo) takes 5 leaps while she (the rabbit) takes
6, but he covers as much ground in two leaps as she does in three.
How many leaps will it take the dingo to catch the rabbit? [3 marks]
10. Five consecutive positive integers have the property that the sum of
the squares of the three smallest is equal to the sum of the squares
of the two largest.
What is this common sum? [3 marks]

11. Jasper and Mariko have to travel 12 km to get home but have only
one bicycle between them.
Their travel plan is such that they start and finish together.
Jasper sets out on the bicycle at 10 km/h, then leaves the bicycle
and walks on at 4 km/h. Mariko sets out walking at 5 km/h, reaches
the bicycle and rides home at 8 km/h.
For how many minutes was the bicycle not in motion? [3 marks]

12. For full marks explain how you found your solution.
Let ABCD be a rectangle, and let E be a point on BC and F
a point on CD such that BE = DF and AEF is an equilateral
triangle.
Prove that the area of triangle ECF equals the sum of the areas of
triangles ABE and AF D. [3 marks]
Western Australian
Junior Mathematics Olympiad 2012
Team Questions 45 minutes

General instructions: Calculators are (still) not permitted.

A compelling problem
Pell’s equation is an equation of the form
X 2 − N Y 2 = 1,
where N is a given positive integer and non-negative integer solutions
for X and Y are sought.
These equations were studied more than a millennium ago in India.

A. Give one solution valid for all N .

B. Find all the solutions if N is a perfect square K 2 .

C. Find one solution different from that in part A, for X 2 − 2Y 2 = 1


and one for X 2 − 3Y 2 = 1.

D. Show that if X = a, Y = b and X = c, Y = d are two solutions of


X 2 −N Y 2 = 1, then X = ac+N bd, Y = ad+bc is also a solution.

E. Find two more solutions (not the ones from parts A and C) for each
of the equations X 2 − 2Y 2 = 1 and X 2 − 3Y 2 = 1.

F. How many non-negative integer solutions are there for the equations
X 2 − 2Y 2 = 1 and X 2 − 3Y 2 = 1?


G. Explain how the Pell equation can be used to approximate N as
a fraction.
Individual Questions Solutions
1. Answer: 28. S B R
Draw AB through X parallel to
QR, with A on P Q and B on RS.
Then X

P A Q
∠XBS = ∠XBR = 90◦ , since AB k QR ⊥ QS
XS = XR
XB is common
∴ 4XBS ∼
= 4XBR by the RHS Rule
∴ SB = RB
So the areas of BSP A and AQRB are equal, and hence each is half
the area of P QRS. Also the area of triangle QXR is half the area
of AQRB, and hence the area of QXR is a quarter of the area of
P QRS, i.e.
Area of P QRS = 4 × 7 = 28.

2. Answer: 12.
Let O and P be the positions of two
adjacent streetlamps, and let the outer
vertical lines represent the sides of the
freeway. Thus AO as shown represents
the distance from the middle of the O
road to one side of the freeway; hence A
AO = 8 m.
Let the two circles centred at O and P X M
represent the circumferences of the ar-
eas lit by the two adjacent streetlamps.
Then for the lamps to be a maximum P
distance apart their light discs must in-
tersect at a point X on one side of the
freeway, and OX = 10 m, since each
lamp lights a disc with diameter 20 m.
Thus XAD is a Pythagorean triangle with hypotenuse 10 and one
side 8, and hence the other side AX is 6 m (to complete a 3 : 4 : 5
triangle.
Hence AX = OM = M P (the last equality, by symmetry).
So the lamps need to be at most OP = 12 m apart to completely
light the freeway.
3. Answer: 113. Since tile B is 5 × 5 and tile F is 4 × 4, tile E is 1 × 1.
Since tile B is 5 × 5 and tile E is 1 × 1, tile C is 6 × 6, and hence
D has side length the sum of the side lengths of C and E, namely
6 + 1 = 7. Thus, D has area 7 × 7.
Since B and F have side lengths 5 and 4, respectively, A has side
length 5 + 4 = 9.
Since G has area 25 square units, its side length is 5. Hence H has
side length the total of the side lengths of F and A minus the side
length of G, i.e. H has side length 4 + 9 − 5 = 8, and hence area 64
square units.
Hence the total area of tiles H and D is 64 + 49 = 113 square units.

4. Answer: 48. Let x be the number of lettuces sold each day.


At the end of Monday, the no. of lettuces is: 80 − x.
So by end of Tuesday, the no. of lettuces is: 2(80−x)−x = 160−3x.
And hence at end of Wednesday:
3(160 − 3x) − x = 0
480 = 10x
x = 48
Hence, 48 lettuces were sold each day.

5. Answer: 168. The diagonals of a rhombus bisect each other at right


angles. Scale the rhombus, by dividing its dimensions by 70 to ob-
tain ABCD with half-diagonals AO = 3 mm and BO = 4 mm.
Then 4AOB is a 3 : 4 : 5 Pythagorean B
triangle, and hence AB = 5 mm. Let r =
OE be the radius of the incircle of ABCD. 5
Now, 4AOE ∼ 4ABO (AA Rule). So, 4
E
r 4 O
= A C
3 5 3
∴ r = 2.4 mm
Scaling back to original size by multiplying
by 70, the incircle of the given rhombus has
radius 168 mm.
D

6. Answer: 24. Let the distances travelled on the way out, uphill,
downhill, and along level ground be: a, b and c, respectively.
Then on the way back, the uphill, downhill, and level ground dis-
tances are b, a and c, respectively.
The total time taken is
a b c b a c
6= ++ + + +
3 6 4 3 6 4
a + b a + b 2c
= + +
3 6 4
a+b+c
=
2
∴ a + b + c = 12
The total distance, out and back is 2(a + b + c) = 24 km.

7. Answer: 39. Say that Honi had s scorpions at the beginning of


January.
105
At the end of January she has 100 × s scorpions.
At the end of February she has 110
100
× 105
100
× s scorpions.
At the end of March she has 100 × 100 × 105
120 110
100
× s scorpions, which
equals
6 11 21 1386
× × ×s= s = 1.386s.
5 10 20 1000
So, to the nearest whole number per cent, the scorpion population
has increased by 39 per cent.

8. Answer: 649. The two digit squares are 16, 25, 36, 49, 64 and 81.
We need a three digit number whose first two digits come from this
list and so do the last two. The candidates are 164, 364, 649 and
816. Of these, only 649 is divisible by 11, since its alternating sum
6 − 4 + 9 = 11 is divisible by 11.

9. Answer: 75. The dingo takes 5 dingo leaps to the rabbit’s 6 rabbit
leaps, which equals 4 dingo leaps. So each 5 leaps, the dingo gains
1 leap on the rabbit. So after 5 × 15 = 75 leaps, he catches her.

10. Answer: 365. Let a be the second number of the sequence. Then
the sum of the squares of the three smallest numbers is
(a − 1)2 + a2 + (a + 1)2 = a2 − 2a + 1 + a2 + a2 + 2a + 1
= 3a2 + 2
and the sum of the two largest numbers is
(a + 2)2 + (a + 3)2 = a2 + 4a + 4 + a2 + 6a + 9
= 3a2 + 2, since equal to the previous sum
∴ 0 = a2 − 10a − 11
= (a − 11)(a + 1)
and hence a = 11 or −1. But the numbers are all positive. So
a = 11 and thus the common sum is 3 · 112 + 2 = 365.
11. Answer: 40. Let the bicycle be left after x km. Since, total times
taken are the same

x 12 − x x 12 − x
+ = +
8 5 4 10
12 − x x
=
10 8
96 − 8x = 10x
96 16
x= =
18 3

Hence,

x 16 1 2
= · = h = time after starting when bicycle is left, and
8 3 8 3
x 16 1 4
= · = h = time after starting when bicycle is picked up.
4 3 4 3

2
So the bicycle is not in motion for 3
h = 40 min.
12. Let x = DF and y = F C.

∠ABE = ∠ADF, since ABCD is a rectangle


AE = AF, since 4AEF is equilateral
BE = DF, (given)
∴ 4ABE ∼
= 4ADF
A B
∴ AB = AD
= DF + F C
=x+y E
∴ ABCD is a square
∴ EC = BC − BE
= DC − DF
= FC = y

∴ 4F CE is a 1 : 1 : 2 triangle
√ D F C
∴ EF =y 2
∴ 2y 2 = EF 2
= AF 2
= DF 2 + AD2
= x2 + (x + y)2
= x2 + x2 + 2xy + y 2
∴ y 2 = 2x2 + 2xy
= 2x(x + y)
Now observe that for this last equation, the left hand side is twice
the area of 4EF C and the righthand side is four times the area
of 4ADF , but since 4ABE and 4ADF are congruent, they have
equal area. Hence
2Area(4EF C) = 4Area(4ADF )

= 2 Area(4ADF ) + Area(4ABE)
Area(4EF C) = Area(4ADF ) + Area(4ABE)
Team Questions Solutions
A compelling problem

A. X = 1, Y = 0.

B. Then (X − KY )(X + KY ) = 1, and since both factors are integers,


we must have X + KY = X − KY = 1, so X = 1, Y = 0.

C. For X 2 − 2Y 2 = 1, i.e. N = 2: X = 3, Y = 2 is a solution.


For X 2 − 3Y 2 = 1, i.e. N = 3: X = 2, Y = 1 is a solution.

D. We assume a2 − N b2 = 1 and c2 − N d2 = 1. Now


(ac + N bd)2 − N (ad + bc)2
= a2 c2 + 2N abcd + N 2 b2 d2 − N a2 d2 − 2N abcd − N b2 c2
= a2 c2 − N a2 d2 + N 2 b2 d2 − N b2 c2
= a2 (c2 − N d2 ) − N b2 (c2 − N d2 )
= a2 − N b2
= 1.

E. N = 2: using part D, with X = 3, Y = 2 twice,


X = 3 · 3 + 2 · 2 · 2 = 17, Y = 3 · 2 + 2 · 3 = 12 is a solution.
Then with X = 3, Y = 2 and X = 17, Y = 12, we get
X = 3 · 17 + 2 · 2 · 12 = 99, Y = 3 · 12 + 2 · 17 = 70
is a solution.
Other solutions are possible. The following is a list of all possible
(X, Y ), for which 1 ≤ Y ≤ 100 000:
(3, 2), (17, 12), (99, 70), (577, 408), (3363, 2378),
(19601, 13860), (114243, 80782)
Similarly for N = 3: X = 2, Y = 1 twice gives
X = 4 + 3 = 7, Y = 2 + 2 = 4.
Then X = 2, Y = 1 and X = 7, Y = 4 gives
X = 14 + 3 · 4 = 26, Y = 8 + 7 = 15.
Other solutions are possible. The following is a list of all possible
(X, Y ), for which 1 ≤ Y ≤ 100 000:
(2, 1), (7, 4), (26, 15), (97, 56), (362, 209), (1351, 780),
(5042, 2911), (18817, 10864), (70226, 40545)
F. We can always use part D to find larger and larger solutions; so
there are infinitely many solutions.

G. We have that X 2 = N Y 2 + 1, so (X/Y √


)2 = N + 1/Y 2 . So if Y is
large, X/Y is a good approximation of N .
Individual Questions Answers
1. 28

2. 12

3. 113

4. 48

5. 168

6. 24

7. 39

8. 649

9. 75

10. 365

11. 40

12.
Team Questions Answers
A compelling problem

A. X = 1, Y = 0.

B. X = 1, Y = 0.

C. For X 2 − 2Y 2 = 1: X = 3, Y = 2.
For X 2 − 3Y 2 = 1: X = 2, Y = 1.

D. We assume a2 − N b2 = 1 and c2 − N d2 = 1. Now


(ac + N bd)2 − N (ad + bc)2
= a2 c2 + 2N abcd + N 2 b2 d2 − N a2 d2 − 2N abcd − N b2 c2
= a2 c2 − N a2 d2 + N 2 b2 d2 − N b2 c2
= a2 (c2 − N d2 ) − N b2 (c2 − N d2 )
= a2 − N b2
= 1.

E. For N = 2: X = 17, Y = 12 is another solution, and X = 99, Y =


70 is yet another solution.
For N = 3: X = 7, Y = 4 is another solution, and X = 26, Y = 15
is yet another solution.

F. There are infinitely many solutions.


G. X/Y is a good approximation of N , for Y large.
Western Australian
Junior Mathematics Olympiad 2013
Individual Questions 100 minutes

General instructions: Except possibly for Question 12, each answer in


this part, is a positive integer less than 1000. No working is needed for
Questions 1 to 11. Calculators are not permitted. In general, diagrams
are provided to clarify wording only, and are not to scale.
Write your answers for Questions 1–11, and solution for Question 12 on
the front and back, respectively, of the Answer Sheet provided.

1. A 40 m by 70 m rectangular piece of land is divided into 10 m by 10 m


small plots by fences. There is also a fence around the whole piece of
land.
How many metres is the total length of fencing? [1 mark]

2. The 3-digit numbers ab4 and 4ab satisfy the property


400 − ab4 = 4ab − 400
What is the 2-digit number ab? [1 mark]

3. Two middle-distance runners are training for the 1500 m. Jo can usually
run 1500 m in 3 minutes 57 seconds, while her younger sister Pat can run
1500 m in 4 minutes 10 seconds. To run a fair race against each other,
Pat will start ahead of the starting line so that both may be expected
to reach the finish line at the same time.
How many metres ahead should Pat stand? [1 mark]

4. On the island Exotica 15% of inhabitants are red heads and 30% have
blue eyes. Among the red heads 2 out of 5 have blue eyes.
What number of percent is the proportion of inhabitants that have blue
eyes but not red hair? [1 mark]

5. The year 2013 has digits whose values are four consecutive integers.
How many years ago was the last time when the digits of the year were
four consecutive integers? [2 marks]
6. A rectangle is cut into two areas by
a single line from a corner, as shown
a
with a > b. The areas are in the
ratio of 8 : 3 and the side length
a + b = 132 cm. b

How many cm is the value of a? [2 marks]

7. A metal cube, of side 9 cm, is melted down and recast into two cylinders.
These cylinders are (geometrically) similar, with their diameters being
5 cm and 10 cm, respectively.
If no metal is lost in the process, how many cm3 is the volume of the
smaller cylinder? [2 marks]

8. The diagram shows a “billiard” table with dimensions 4 × 12.


28 27 26 25 24 23 22 21 20 19 18 17 16

29 15

30 14

31 13

0 1 2 3 4 5 6 7 8 9 10 11 12

A ball initially at position 11 is struck firmly at an angle of 45◦ to the


edge of the table, to move to the right side at position 13. From here it
bounces off and moves to position 19. It continues in this way forever.
At what position is the ball after 2013 moves? [2 marks]

9. The sequence of numbers


1, 2, 3, 4, 5, 6, 8, 9, 10, 11, 12, 13, 15, 16, . . .
is made by removing multiples of 7 from the list of positive integers.
What is the 99th term? [3 marks]
10. ABC is an example of an isosceles B
(but not equilateral) triangle with
integer side lengths. 2 2

A 3 C

How many such triangles can be made using the integers from 1 to 9
inclusive? [3 marks]

11. An empty 4 km long iron ore train is travelling north along a flat plain
at 15 km/h. At the same time a fully laden iron ore train that is 3 km
long, is travelling south at a speed of 5 km/h, along a parallel track,
towards the northbound train. The two tracks are a negligible distance
apart. At 12 noon, when the two trains are 12 km apart, the drivers of
both trains catch sight of one another.
How many minutes after 12 noon will the guards at the rear of each
train finally lose sight of one another, given that they will do so when
the rears of both trains are 12 km apart? [3 marks]

12. For full marks explain how you found your solution.
P QRS is a trapezium with P Q = 6 cm, SR = 10 cm, and P S = QR.
The distance between P Q and SR is 12 cm.
What is the radius of the smallest circle that can cover P QRS? √
Note. An exact number is required. For instance, if the answer is 3,
then give that answer, not 1.732 (or any other decimal). [4 marks]
Western Australian
Junior Mathematics Olympiad 2013

Team Questions 45 minutes

General instructions: Calculators are (still) not permitted.

Special numbers
We say that a positive integer is k-special, where k is a positive integer, if
the sum of the digits of the number plus k times the product of the digits
of the number equals the original number.
Example. 29 is 1-special, since

2 + 9 + 2 × 9 = 29.

Convention. To be able to distinguish a number with digits a, b, . . . from


its product, we write it with a line over the top. So, ab is the same as
10a + b, e.g. if a = 2, b = 9 then

ab = 10 × 2 + 9 = 29
whereas ab = 2 × 9 = 18.

A. List all the two-digit numbers that are 1-special.

B. Show why the 1-special numbers you found in A all end with the same
digit. (An algebraic proof is expected.)

C. Show there is no two-digit number that is 2-special.

D. For which integers k, are there two-digit numbers which are k-special?
Note. For each k you find, you should give an example of such a k-
special number, to show that there really are k-special numbers, for the
number k.
Let abc be k-special for all that follows.
Note. If you get stuck on F or G, you can continue to H and I, and use
the results of F and G.

E. What is the largest integer k such that a 3-digit number can be k-


special?

F. Show that if a = b then a is one of 1, 2, 3, 4, 6, 9.

G. Show that if a 6= b, then the possible a, b are as in the following table.


a b
1 3
2 6
3 9
1 9
3 1
6 2
9 3
9 1

H. Show there is no 8-special 3-digit number.

I. Find the unique 5-special 3-digit number.


Individual Questions Solutions
1. Answer: 670. The total fencing is the total length of the lines in the
diagram,

i.e. the total fencing required is


5 × 70 + 8 × 40 = 350 + 320 = 670 m.

2. Answer: 36. Using the convention abc to represent the 3-digit number,
with digits (in order) a, b, c.
Whatever a and b are,
4ab − 400 = ab
∴ 400 − ab4 = ab.
So the units digit b = 6 and a < 4. But a = 1 or 2 would give a 3-digit
answer, so a = 3.
Alternatively, let ab = N . Then
400 − (N × 10 + 4) = 400 + N − 400
396 = 11N
36 = N.

Hence, the 2-digit number N = ab = 36.

3. Answer: 78. Pat’s average speed for the 1500 m is 1500/250 = 6 m/s,
since 4 min. 10 s = 250 s. If Jo runs the full 1500 m, then the race will
take 3 minutes 57 seconds. So Pat needs what for her is a 13 second
start; in 13 seconds, Pat can run a distance of d = vt = 6 × 13 = 78 m.
So she will need a 78 m start in order to finish level with Jo.
4. Answer: 24. We may as well assume that there are 100 people on
Exotica. Then the percentages simply become numbers. Let R be the
set of red heads, and B be the set of blue-eyed inhabitants. Then the
n(R) = 15 red heads are distributed
2
5 · 15 = 6 have blue eyes, and 9 do not have blue eyes.
Hence, the number of blue-eyed inhabitants without red hair is
n(B) − n(B ∩ R) = 30 − 6 = 24.
Hence 24% of inhabitants have blue eyes but not red hair.

5. Answer: 581. The last time this happened was 1432, which is 581 years
ago.
Student’s probable solution: No year beginning 2 and using consecutive
digits occurs before 2013. So we want the most recent year beginning
1 and using consecutive digits 0, 1, 2, 3 or 1, 2, 3, 4. The most recent
such year begins 14 and of those beginning 14, the most recent is 1432.

6. Answer: 72. Adding an extra line, we split the lower area into an area
that is congruent to the upper area and a rectangle.

3 3 a

5 b

And, so we see,
a` : b` = (3 + 3) : 5
where ` is the length of the rectangle. Hence,
6
a= × 132 = 72.
6+5

7. Answer: 81. The fact that the cylinders are ‘similar’ means that their
volumes are in the ratio of the cube of their diameters, i.e. 53 : 103 =
125 : 1000 or 1 : 8.
This means the smaller cylinder has a volume equal to 91 of the original
cube, i.e. the smaller has a volume of 91 × 93 = 81 cm3 .
8. Answer: 29. By tracing out the path of the ball we find it returns to
position 11 after 8 moves; 2008 is a multiple of 8 so after 2008 moves
the ball will be at position 11. Tracing out its path shows that it is at
position 29 after 5 more moves.

9. Answer: 115. The number of multiples of 7 ≤ n is bn/7c. Thus n must


satisfy 99 = n − bn/7c. Since b99/7c = 14, n ≥ 99 + 14. However,
between 99 and 99 + 14 there are another two multiples of 7. Testing
n = 115, we have 115 − b115/7c = 115 − 16 = 99. So the 99th term is
115.

10. Answer: 52. If the base is of length b, then the two equal legs ` must
be such that 2` > b, in order to form a triangle. Thus we have:
for b = 1, ` 6= 1, leaving 8 choices;
for b = 2, ` 6= 1, 2, leaving 7 choices;
for b = 3, ` 6= 1, 3, leaving 7 choices;
for b = 4, ` 6= 1, 2, 4, leaving 6 choices;
for b = 5, ` 6= 1, 2, 5, leaving 6 choices;
for b = 6, ` 6= 1, 2, 3, 6, leaving 5 choices;
for b = 7, ` 6= 1, 2, 3, 7, leaving 5 choices;
for b = 8, ` 6= 1, 2, 3, 4, 8, leaving 4 choices;
for b = 9, ` 6= 1, 2, 3, 4, 9, leaving 4 choices.
Thus, in all there are 2 × (4 + 5 + 6 + 7) + 8 = 52 such triangles.

11. Answer: 93. The problem is equivalent to one train being stationary
and the other train moving at (15 + 5) km/h.
After becoming visible to the other train, the ‘moving’ train will have
to travel
(2·12+3+4) km = (twice the visibility distance+sum of two train lengths)
to become invisible again, i.e. the time in minutes until one train will
become invisible to the other is:
2 · 12 + 3 + 4
· 60 = 93 minutes.
15 + 5

12. Answer: 31 481 cm. The smallest circle cov-
P 12 Q
ering P QRS is the circumcircle of P QRS.
Locate the midpoint of SR at the origin.
By symmetry the circumcentre of P QRS, is r
located at a point (0, y) that is equidistant
from Q and R. Hence, the circumradius r (0, y)
of P QRS satisfies,
r2 = (12 − y)2 + 32 r
= y 2 − 24y + 122 + 32 (1) S R
−5 0 3 5
r 2 = y 2 + 52 (2)
∴ 0 = −24y + 122 + 32 − 52 , by (1) − (2)
24y = 122 − 16
2 16
y =6− 3 = 3
∴ r2 = ( 16 2
3) +5
2

= ( 13 )2 (162 + 152 )
= ( 13 )2 (256 + 225)
= ( 13 )2 · 481
1

∴ r= 3 481 √
Hence, the smallest circle covering P QRS has radius 13 481 cm.
Alternatively, locate the midpoint of (−3, 12) (3, 12)
SR at the origin, and assign coordinates P Q
to the vertices as shown in the diagram.
By symmetry the circumcentre of P QRS,
is located where the perpendicular bisec-
tor ` of QR meets the y axis (which is ` M
X
the axis of symmetry of P QRS). QR has
slope
12 − 0 S R
= −6. (−5, 0) O (5, 0)
3−5
Hence, ` is the straight line through the midpoint

M = (3 + 5)/2, 12/2) = (4, 6)
1
with slope 6 (the negative reciprocal of the slope of QR), i.e.

` : y − 6 = 61 (x − 4).
The circumcentre X of P QRS is the point on ` where x = 0. Substi-
tuting, x = 0 in the equation for `, we have
y − 6 = 16 (0 − 4)
2 16
y =6− 3 = 3
16
∴ X = (0, 3)
Hence, the circumradius r of P QRS satisfies
r2 = XR2
= OX 2 + OR2
2 2
= ( 16
3) +5
= ( 31 )2 (162 + 152 )
= ( 31 )2 (256 + 225)
= ( 31 )2 · 481
1

∴ r = 3 481
1

Hence, the smallest circle covering P QRS has radius 3 481 cm.
Team Questions Solutions
Special numbers

A. Answer: 19, 29, 39, 49, 59, 69, 79, 89, 99. Let the number be ab. Then
for ab to be 1-special, we require
(a + b) + 1 · ab = ab = 10a + b
ab = 9a
b = 9.
So the two-digit 1-special numbers are those that have 9 as the units
digit, i.e. they are: 19, 29, 39, 49, 59, 69, 79, 89, 99.

B. Let the two-digit 1-special number be 10a + b. Then


a + b + ab = 10a + b
∴ ab − 9a = 0
a(b − 9) = 0
∴ a = 0 or b = 9.
But for a = 0, the number is not of 2 digits. So we must have b = 9.
Hence all two-digit 1-special numbers end with 9.

C. Suppose for a contradiction that a two-digit 2-special number exists.


Then there are integers a, b ∈ {0, 1, 2, . . . , 9}, but a 6= 0 such that
(a + b) + 2ab = ab = 10a + b
2ab = 9a
2b = 9, dividing through by a, since a 6= 0.
So we have our contradiction, since 2b is necessarily even, but 9 is odd.
Therefore, no 2-special numbers exist.

D. Answer: k is 1, 3 or 9, with examples 19, 13, 11, respectively. Suppose


ab is k-special. Then a, b ∈ {0, 1, 2, . . . , 9}, but a 6= 0 such that
(a + b) + kab = ab = 10a + b
kab = 9a
kb = 9, dividing through by a, since a 6= 0.
So k is a divisor of 9, i.e. k ∈ {1, 3, 9}.
If k = 1, then 19 is an example of a 1-special number.
If k = 3, then any 2-digit number ending in 3 is k-special, e.g. 13 is
3-special, since
(1 + 3) + 3 · 1 · 3 = 13.
If k = 9, then any 2-digit number ending in 1 is k-special, e.g. 11 is
9-special, since
(1 + 1) + 9 · 1 · 1 = 11.

E. Answer: 108. We have for general abc,


(a + b + c) + kabc = abc = 100a + 10b + c
kabc = 99a + 9b.
First observe that the righthand side is nonzero, since a 6= 0. Hence,
the lefthand side is nonzero, and so none of k, a, b or c is zero. Dividing
through both sides by abc we have,
1  99 9 
k= +
c b a
≤ 1 · (99 + 9) = 108,
where that bound can be attained if a = b = c = 1, i.e. 111 is a
108-special number:
(1 + 1 + 1) + 108 · 1 · 1 · 1 = 111.
Hence 108 is the largest integer k for which a 3-digit number can be
k-special.

F. As in E, for general abc, we have the condition


kabc = 99a + 9b.
With the additional condition, a = b,
ka2 c = 99a + 9a = 108a
kac = 108, since a 6= 0.

Thus, a must divide 108 = 22 ·33 , leading to the possibilities 1, 2, 3, 4, 6, 9.


The above is all that was required for solution of this part. In the fol-
lowing table, we include what kac = 108 reduces to, and an example,
verifying that the condition can be satisfied and hence that a is indeed
possible.
a Condition Example
1 kc = 108 119 is 12-special
2 kc = 54 229 is 6-special
3 kc = 36 339 is 4-special
4 kc = 27 449 is 3-special
6 kc = 18 669 is 2-special
9 kc = 12 993 is 4-special

G. As in E the k-special condition for 3-digit numbers abc reduces to


kabc = 99a + 9b.
Hence, we must have a 9b and b 99a. So ax = 9b and by = 99a for
some integers x, y. Multiplying the two equalities, and cancelling ab
we get xy = 11 · 34 . From the first equality (and using that a, b ≤ 9)
we get x ≤ 81 and also 116 x. So x ∈ {1, 3, 9, 27, 81}. The case x = 9
yields a = b, and the other four cases yield that one of a, b is a multiple
of 3 or 9 of the other, leading to the possibilities,
(1, 3), (3, 1), (2, 6), (6, 2), (9, 3), (3, 9), (1, 9), (9, 1).
Alternatively, as before, we have that the k-special condition for 3-
digit numbers abc reduces to
kabc = 99a + 9b.
Let d = gcd(a, b), and write a = da0 , b = db0 . Then after cancelling d
we have
ka0 b0 dc = 99a0 + 9b0 .
Now a0 divides the lefthand side and hence also the righthand side, and
so divides 9b0 , but gcd(a0 , b0 ) = 1. So a0 9, and hence a0 is 1, 3 or 9.
Arguing similarly, we have b0 99 but 0 ≤ b0 ≤ 9 implies 11 is not a
divisor of b0 . So b0 9, and hence b0 is 1, 3 or 9.
Now (a0 , b0 ) = (1, 1), implies a = b contrary to assumption. Also,
either a0 or b0 is 1, since otherwise they are have a gcd of at least 3,
contradicting their being coprime. Since a, b ≤ 9, if the larger of a0 or
b0 is 3, then d is 1, 2 or 3; but if the larger of a0 or b0 is 9, then d = 1.
Thus we have the possibilities,
(1, 3), (3, 1), (2, 6), (6, 2), (9, 3), (3, 9), (1, 9), (9, 1).
The above is all that is required. In the following table, we include for
each pair (a, b), what ka0 b0 dc = 99a0 + 9b0 reduces to, and an example,
verifying that the condition can be satisfied and hence that the pair
(a, b) is indeed possible.
a0 b0 d a b Condition Example
1 3 1 1 3 kc = 42 137 is 6-special
1 3 2 2 6 kc = 21 267 is 3-special
1 3 3 3 9 kc = 14 397 is 2-special
1 9 1 1 9 kc = 20 195 is 4-special
3 1 1 3 1 kc = 102 316 is 17-special
3 1 2 6 2 kc = 51 623 is 17-special
3 1 3 9 3 kc = 34 932 is 17-special
9 1 1 9 1 kc = 100 915 is 20-special

H. From F we have, with k = 8,


8abc = 99a + 9b.
In the case, a = b, we have
8a2 c = 99a + 9a = 108a
8ac = 108
which is impossible to satisfy, since 86 108.
And, if a 6= b, we must check for each pair (a, b) found in G whether
8abc = 99a + 9b
can be satisfied. We do so in the following table.
a b 8abc = 99a + 9b Why not satisfied
1 3 8 · 3 · c = 126 86 126
2 6 8 · 12 · c = 252 (8 · 4)6 252
3 9 8 · 27 · c = 378 86 378
1 9 8 · 9 · c = 180 86 180
3 1 8 · 3 · c = 306 86 306
6 2 8 · 12 · c = 612 86 612
9 3 8 · 27 · c = 918 86 918
9 1 8 · 9 · c = 900 86 900
Hence, since 8abc = 99a + 9b cannot be satisfied, no 8-special 3-digit
numbers with a 6= b exist.
∴ no 8-special 3-digit numbers exist.
Alternatively, if one has found the table given in G, then one has kc ∈
{42, 21, 14, 20, 102, 51, 34, 100}, and since none of these possibilities for
kc is divisible by 8, we deduce k cannot be 8, and hence no 8-special
3-digit numbers exist.
I. Answer: 194. Starting with the k-special condition for 3-digit numbers
abc from E,
kabc = 99a + 9b,
we have
5abc = 99a + 9b.
First observe that if a = b, this last condition reduces to
5ac = 108,
which is impossible to satisfy, since 5 does not divide 108. So we have
a 6= b and hence (a, b) must be one of the pairs in F. Now,
99a + 9b ≡ −a − b (mod 5)
So, we need the pair (a, b) to satisfy its sum is divisible by 5. Hence
(a, b) is (1, 9) or (9, 1).
For (a, b) = (9, 1) we have
5·1·9·c≤5·1·9·9
= 405
< 900 = 99 · 9 + 9 · 1.
So we cannot obtain a 5-special 3-digit number with (a, b) = (9, 1).
For (a, b) = (1, 9), we require
5 · 1 · 9 · c = 99 · 1 + 9 · 9
5c = 11 + 9 = 20
c = 4,
and so we discover the 5-special 3-digit number
194 = (1 + 9 + 4) + 5 · 1 · 9 · 4,
and we see that it is unique.
Western Australian
Junior Mathematics Olympiad 2014
Individual Questions 100 minutes

General instructions: Each solution in this part, except for Question


12, is a positive integer less than 1000. No working is needed for Ques-
tions 1 to 11. Calculators are not permitted. Write your answers on
the answer sheet provided. In general, diagrams are provided to clarify
wording only, and are not to scale.

1. A palindrome is a positive integer that reads the same forwards and


backwards, for example 3113.
What is the smallest x for which x + 2014 is a palindrome? [1 mark]

2. What is the maximum number of intersection points of a rectangle


and a circle? [1 mark]

3. In the diagram, AB and AC are equal, A


as are BC and BD.
If ∠BAC = 36◦ , how many degrees is D
∠DCA?

B C
[1 mark]

4. Two parallel chords in a circle lie on the same side of the centre and
have lengths 40 and 48 cm. The distance between them is 8 cm.
How many centimetres would the distance between the chords be,
if they were on opposite sides of the centre? [1 mark]

5. If each edge of a cube is extended by 40%, by how many percent


has the surface area of the cube been extended? [2 marks]
6. Stefan is driving 150 km from Perth to Australind. For the first
30 km he averages 40 km/h. Then for the rest of the journey, the
speed limit is 100 km/h, and he’d like his average speed for the whole
journey to be 75 km/h.
At how many km/h does Stefan need to travel over the last 120
kilometres? [2 marks]

7. Suppose
(2x + 5)6 a0 + a1 x + a2 x 2 + a3 x 3 + a4 x 4 + a5 x 5 + a6 x 6
= ,
(x + 4)5 b0 + b 1 x + b2 x 2 + b3 x 3 + b 4 x 4 + b5 x 5
for all x 6= −4.
a0 − a1 + a2 − a3 + a4 − a5 + a6
What is ? [2 marks]
b0 − b1 + b2 − b3 + b4 − b5

8. Let R be a rectangle of area 360 m2 . If the length of R is increased


by 10 m and its width is decreased by 6 m, this new rectangle has
the same area as R.
How many metres is the perimeter of the original rectangle R?
[2 marks]

9. The 3-digit numbers acb, a79, b0c and bb1 are increasing consecutive
terms of an arithmetic progression, that is
a79 − acb = b0c − a79 = bb1 − b0c.
What is the number abc? [3 marks]

10. Henrietta writes down all the two digit numbers and for each number
she calculates the product of the two digits. She then adds all the
products together and divides the total by 25.
What is her answer? [3 marks]

11. Given u, v, w, x, y are integers such that


u − 1 < v − 2 < w − 3 < x − 4 < y − 5,
and
(u − 1)(v − 2)(w − 3)(x − 4)(y − 5) = 2014,
find the largest possible value of
u + v + w + x + y.
[3 marks]
12. For full marks explain how you found your solution.
A lune is a crescent shape formed by two intersecting circular arcs;
it originates from the Latin word Luna which translates to “moon”.
The diagram below shows a right-angled triangle with sides of length
a, b and c and two lunes whose boundaries are formed by three semi-
circles.
Consider the sum of the areas of the two lunes, and the area of the
triangle. Which is larger? Your answer could depend on the values
of a, b, c. [4 marks]

a
b

c
Western Australian
Junior Mathematics Olympiad 2014
Team Questions 45 minutes

General instructions: Calculators are (still) not permitted.

Walking the Grid


The 5 × 5 grid below represents streets in a city. An m × n grid means
there are m blocks north-south and n blocks east-west, where a block
is the distance between two adjacent intersections. Observe that for a
5 × 5 grid there are 6 streets north-south and 6 streets east-west. As
indicated by the north sign on the diagram, north is at the top of the
page.
B
N

A. At each intersection, write the number of ways of getting there


from A if you are always travelling away from A (so you are always
moving north or east, never west or south). There is a grid on the
back of the Team Cover Sheet that you should use for this purpose.
Explain your method.

B. If you only travel north or east, how many ways are there to go
from A to B? List all the different lengths of these paths, in blocks,
where a block, as previously mentioned, is the distance between two
adjacent intersections.
C. We now consider an 8 × 6 grid, where we call A the SW corner and
B the N E corner, as with the initial 5 × 5 grid. How many paths
are there from A to B in this grid?
What about an m × n grid? Try to find a general formula.

D. On the 8 × 6 grid (i.e. 8 blocks north by 6 blocks east), described in


C., there is a shop at a point C two blocks north and three blocks
east of A.
How many paths go from A to B via C? Explain your method.

E. Imagine we have a large grid and an interior street intersection P .


On a diagram, mark all the intersections that you can get to after
walking 5 blocks from P , always moving in a compass direction
away from P .

F. In the large grid of E., describe the shape made by the locations
of all the intersections you can get to after walking 20 blocks from
P , always moving in a compass direction away from P .

G. Returning to the 8×6 grid with the shop at C, if you start at A (SW
corner) and walk only north or east for 5 blocks, choosing randomly
with probability 50% at each intersection whether to continue north
or east, what is the probability you will end up at C?

H. In the centre of Future City there


are 16 four-storey buildings set
out in a square array. On each of
the four floors of each building,
there are aerial walkways to
the same floor of each adjacent
building. There are also lifts
in each building. The lowest
floor of the buildings is set off
the ground so that savage and
ferocious animals can roam in
beautiful parkland beneath the
buildings. The view shown is
from an aircraft passing over the
buildings.

How many different paths are there which start in the lowest floor
of the most southwest building and finish in the top floor of the
most northeast building? Each path is always moving north, east
or up, never south, west or down.
Individual Questions Solutions
1. Answer: 98. The least palindrome greater than 2014 is 2112, and
2112 − 2014 = 98.

2. Answer: 8. The maximum number


occurs in the configuration shown.

3. Answer: 18. Since AB = AC, triangle ABC is isosceles with


∠ABC = ∠ACB. Hence ∠ABC = 21 (180 − 36)◦ = 72◦ .
Since BC = BD, triangle BCD is isosceles with ∠BDC = ∠BCD.
Hence ∠BCD = 12 (180 − 72)◦ = 54◦ .
Since ∠ACB = 72◦ , ∠DCA = (72 − 54)◦ = 18◦ .

4. Answer: 22. Let the radius of the circle be


r, and the distance from the centre of the
circle to the longer chord be x. Then by 40
Pythagoras’ Theorem, r 8 48
x + 24 = r
2 2 2 x
= (x + 8)2 + 202
= x2 + 16x + 82 + 202
∴ 242 = 16x + 82 + 202
62 = x + 22 + 52
∴ x = 36 − 25 − 4
= 7.
Hence when either the short or the long chord is shifted to the other
side of the centre, the distance between them is
8 + 7 + 7 = 22 cm.

5. Answer: 96. If length is scaled by 1.4 (100% + 40%), then area is


scaled by a factor of 1.42 = 1.96 = 100% + 96%, i.e. the surface area
increases by 96%.
6. Answer: 96. The whole journey must take
150 km
= 2 h.
75 km/h
The first part took
30 km
= 34 h.
40 km/h
5
So Stefan must do the remaining 120 km in 4
h. This requires a
speed of
120 km
5 = 120 × 54 km/h
4
h
= 96 km/h.

7. Answer: 3. The expression is obtained by replacing x by −1 in the


right hand side. Hence it is also equal to
(2(−1) + 5)6 36
= = 3.
(−1 + 4)5 35

8. Answer: 76. Let the length and width of R by ` and w, respectively.


Then
`w = 360 (1)
= (` + 10)(w − 6)
= `w + 10w − 6` − 60
∴ 0 = 10w − 6` − 60
6` = 10w − 60
` = 35 w − 10 (2)
∴ ( 53 w − 10)w = 360, substituting (2) in (1)
3
(w − 6)w = 216, multiplying both sides by 5
∴ 0 = w2 − 6w − 216
= (w + 12)(w − 18)
So either w = −12 or 18. Since w is a distance, it cannot be negative.
Thus w = 18, whence ` = 20, and the perimeter is 2(w + `) = 76.
9. Answer: 235. To avoid ambiguity, we write digit representations
with an overline; otherwise, numbers are written fully expanded in
base ten.
The common difference is a 2-digit number, say tu (for tens and
units).
The first difference yields 79 − 10c − b = 10t + u. Since u + b cannot
reach 19,
u + b = 9 and c + t = 7. (3)
The third difference yields 10b + 1 − c = 10t + u. So, either
c + u = 1 ( =⇒ b = t) or c + u = 11 ( =⇒ b = t + 1) (4)
The difference between bb1 and a79 is twice tu. So
100b + 10b + 1 − 100a − 70 − 9 = 20t + 2u. (5)
Hence 9 + 2u has units digit 1, so u = 1 or 6.
Suppose u = 1, then c = 0 by (4), b = 9 and t = 7 by (3), but this
contradicts (4). Hence u = 6.
So b = 8 by (3), c = 5 by (4), t = 7 by (3).
Substituting into (5) we get 252 − 100a = 52, so that a = 2.
Thus, finally we get abc = 235.

10. Answer: 81. The products of the numbers with 1 as the first digit
are 1 × 1, 1 × 2, . . . , 1 × 9. The sum of these is
1 × (1 + 2 + · · · + 9) = 1 × 45.
Similarly the sum of the numbers with 2 as the first digit is 2 × 45,
and so on. The grand total is therefore
(1 + 2 + · · · + 9) × 45 = 45 × 45 = 2025.
Dividing this by 25 gives 81.

11. Answer: 85. Let U = u − 1, V = v − 2, W = w − 3, X = x − 4,


Y = y − 5.
Then
U < V < W < X < Y,
so that, in particular, U, V, W, X, Y are distinct and
U V W XY = 2014 = 2 · 19 · 53.
We can write 2014 as the product of five positive integers, with at
most two equal only as 1 · 1 · 2 · 19 · 53, and, since at most two
of U, V, W, X, Y can have the same absolute value, (otherwise they
cannot be distinct), |U |, |V |, |W |, |X|, |Y | in some order must be 1,
1, 2, 19, 53, which means two of U, V, W, X, Y must be 1 and −1,
i.e. there must be at least one other of U, V, W, X, Y that’s negative,
since their product is positive. Now
u + v + w + x + y = U + V + W + X + Y + 15.
So maximising U + V + W + X + Y , maximises u + v + w + x + y,
and this occurs when
(U, V, W, X, Y ) = (−2, −1, 1, 19, 53).
Thus the maximum value of u + v + w + x + y is
−2 − 1 + 1 + 19 + 53 + 15 = 85.

12. Answer: Neither; the two areas are equal. Since a triangle in a
semicircle is right-angled, the area of the triangle (striped) is
1 ab
×a×b= .
2 2
The total area of the lunes (grey) is the area of the semi-circle of
diameter a (unshaded plus grey) plus the area of the semi-circle
of diameter b (unshaded plus grey) plus the area of the triangle
(striped) minus the area of the semicircle of diameter c (unshaded
plus striped), that is
 a 2 1  2
1 b 1 1  c 2
×π× + ×π× + ×a×b− ×π×
2 2 2 2 2 2 2
π 2  ab
= a + b2 − c 2 +
8 2
ab
= ,
2
since a + b = c by Pythagoras’ Theorem.
2 2 2

Therefore the total area of the two lunes is equal to the area of the
right-angled triangle.
Team Questions Solutions
Walking the Grid

A.

1 6 21 56 126 B
252

1 5 15 35 70 126

1 4 10 20 35 56

1 3 6 10 15 21

1 2 3 4 5 6

1 1 1 1 1
A

Explanation: first we fill in 1s going north and east of A; there is


only one way to get to these points, namely by proceeding from
the point immediately to the south (resp. west) of the point. For
all other points, fill in by summing the numbers to the immediate
south and west.

B. All path lengths are 10 blocks, since one must, in some order, go 5
blocks north of A and 5 blocks east.
C. Answer: 3003 paths. Proceeding in the same manner as for A., the
total at B is 3003, and so there are 3003 paths from A to B.
1 9 45 165 495 1287 B
3003

1 8 36 120 330 792 1716

1 7 28 84 210 462 924

1 6 21 56 126 252 462

1 5 15 35 70 126 210

1 4 10 20 35 56 84

1 3 6 10 15 21 28

1 2 3 4 5 6 7

1 1 1 1 1 1
A
For an m × n grid, observe that the summation we do at each
intersection, is a defining property of Pascal’s triangle, or better,
the number N (m, n) of paths from A(0, 0) to B(n, m) is the number
of ways of choosing n easterly blocks (or the number of ways of
choosing m northerly blocks) from m + n blocks, i.e.
m+n m+n
   
N (m, n) = = .
m n
D. Answer: 840 paths. We may obtain this by filling in the number
of paths to C from A, and then to B from C as per the diagram
below. Alternatively, the number of paths from A to B via C, is
the number of paths from A to C times the number of paths from
C to B, i.e.
(Number of paths from A to B via C)
= (Number of paths from A to C) × (Number of paths from C to B)
= N (2, 3) × N (6, 3)
   
5 9
= ×
2 3
5·4 9·8·7
= ×
1·2 1·2·3
= 10 × 84
= 840.
10 70 280 B
840

10 60 210 560

10 50 150 350

10 40 100 200

10 30 60 100

10 20 30 40

1 3 6 10 10 10 10
C

1 2 3 4

1 1 1
A
E. The intersections 5 blocks from P , always moving in a compass
direction away from P form a diamond (square) of dots centred at
P as per the diagram.


• •
• •
• •
• •
• P •
• •
• •
• •
• •

F. Intersections form a diamond (square) whose diagonals intersect at


P and whose vertices are 20 blocks in each compass direction from
P.

G. Answer: 165
. There are 25 = 32 paths emanating from A of length
5 blocks, of which 10 end at C. So the probability a 5 block walk
will end at C is 10
32
5
= 16 .

H. A path consists of three northward moves, three eastward moves


and three upward moves, which can be arranged in any order.
9

There are 3
= 84 ways to put the upward moves on the path,
6

then 3 = 20 ways to put on the northward moves, leaving the
three eastward moves determined.
Altogether there are 84 × 20 = 1680 paths.
Western Australian
Junior Mathematics Olympiad 2015
Individual Questions 100 minutes

General instructions: Except possibly for Question 12, each answer in this part is a
positive integer less than 1000. No working is needed for Questions 1 to 11. Calculators
are not permitted. In general, diagrams are provided to clarify wording only, and are not
to scale.
Write your answers for Questions 1–11, and solution for Question 12 on the front and
back, respectively, of the Answer Sheet provided.

1. The sum of two prime numbers is the prime number 883.


What is the larger of the two primes (whose sum is 883)? [1 mark]

2. The floor bac of a real number a is the largest integer not greater than a, and the
ceiling dae of a is the smallest integer not less than a.
For example, b2.7c = 2 and d2.7e = 3.
 
Calculate bπc × (bπc × bπc + dπe) × bπcdπe − dπebπc + bπc. [1 mark]

3. In a 36 item multiple choice algebra test, a correct answer earns 3 points, an incorrect
answer causes a one point deduction, and there is no penalty for leaving a blank.
Adam left 6 questions blank and got 6 wrong, while Eve answered all the questions
and got 23 as many marks as Adam.
How many questions did Eve get wrong? [1 mark]

4. How many 3-digit positive integers have no repeated digits?


Note. The number 123 has no repeated digit, but 343 has a repeated digit, namely 3.
[2 marks]

5. What is the mean of all positive two-digit multiples of 4? [2 marks]

6. When the three-digit numbers 8a4 and 2b3 are added together, the answer is a number
divisible by 9.
What is the largest possible value of a + b? [2 marks]

7. A circle is inscribed in a square as in the diagram. We draw


a rectangle with horizontal length 12 cm and vertical height
6 cm, whose bottom left corner is the corner point A of the
square and whose top right corner is on the circle but not
on the square.
How many cm is the length of a side of the square?

A [2 marks]
8. Gertrude is going to make 100 bets of $1 each, on a football game between the Beagles
and the Rockers. If she bets $1 on the Beagles and they win she will get $1.70 back. If
she bets $1 on the Rockers and they win she will get $2.50 back. She works out that
she will be certain to make a profit if she makes a certain number of her bets on the
Beagles and the rest of her bets on the Rockers.
How many bets should Gertrude make on the Beagles so that she makes a profit,
whatever the outcome of the football game, given that a draw cannot occur? [2 marks]

9. Rhombus EF GH is inscribed in E
A B
rectangle ABCD as shown; HQ and
SF are parallel to AB, and P G
and ER are parallel to AD. Also,
AE = 48, AH = 14, BF = 30. P
H Q
What is the length of the perimeter
of rectangle P QRS? S F
R

D G C
[2 marks]

10. For a positive integer n, “n factorial ”, written n!, is the product of all the positive
integers less than or equal to n.
For example, 10! means 1 × 2 × 3 × 4 × 5 × 6 × 7 × 8 × 9 × 10 = 3628800 and has 2
zeros on the end.
With how many zeros does the number 2015! end? [3 marks]

11. Every day Sahria rides her bicycle to school. One day, when she has ridden 23 of the
distance to school, her bicycle suddenly snaps in two. She is obliged to walk the rest
of the distance, at 31 the speed she normally cycles, and arrives at school 5 minutes
later than usual. She calculates that if she had ridden 400 metres further before the
awful calamity occurred, she would have arrived only 1 minute later than usual.
How many metres per minute is Sahria’s speed while riding her bicycle? [3 marks]

12. For full marks explain how you found your solution.
Three wheels can be arranged on a horizontal surface
so that all are touching each other and all are tangent
to the horizontal surface. The radius of the largest
wheel is 16 cm while the medium wheel has a radius
of 9 cm. The radius of the smallest wheel is x cm.

(a) Find the value of x. [2 marks]


(b) Suppose the two larger wheels have radii a and b cm.
Find the value of x in terms of a and b. [2 marks]
Western Australian
Junior Mathematics Olympiad 2015
Team Question 45 minutes

General instructions: Calculators are (still) not permitted.

A Knave’s Tale
Philip Philpot is a scientist studying lying and truth-telling. He visits a distant island in
which each inhabitant is either a Knight or a Knave. Knights always tell the truth and
Knaves always lie. It is impossible to distinguish between Knights and Knaves except by
asking them questions.
Notice that it would be no use asking someone ‘Are you a Knight?’ because both Knights
and Knaves would answer ‘Yes’. Also, it is no use asking ‘Are you a Knave’, because both
would answer ‘No.’
Your answers to the following question parts require explanations written in clear English.

A. Philip meets 2 islanders: Ivor and John (I and J for short).


I says: ‘Both of us are Knaves’.
Is J a Knight or a Knave?

B. This time, Philip meets Kenneth and Lancelot (K and L for short).
K says: ‘At least one of us is a Knave’.
Is L a Knight or a Knave?

C. According to another version of this story, Philip meets Maurice and Norris (M and
N for short), and M says: ‘We are both the same type’.
Is N a Knight or a Knave?

D. At another time, Philip meets Percy and Quentin (P and Q for short).
Philip asks P whether Q is a Knight. He then asks Q whether P is a Knight.
Would their answers be the same?

E. This time, Philip meets 3 islanders: Ross, Scott and Terence (R, S and T for short).
He asks R: ‘Are you a Knight or a Knave?’ and R answers, but Philip does not hear
him.
He asks S: ‘What did R say?’ and S replies: ‘He said he is a Knave.’
T says: ‘Don’t believe S, that is a lie!’
Is T a Knight or a Knave?
F. Philip then meets another 3 islanders: Umberto, Valdemar and Warwick (U , V and
W , for short).
He asks U how many of them are Knaves.
Once again he doesn’t hear the reply, so he asks V what U had said.
V replies that U said exactly 2 of them are Knaves.
W claims that V is lying.
Is W a Knight or a Knave?

G. Again, Philip comes upon 3 islanders: Xerxes, Yorrick and Zachary (X, Y and Z, for
short), who make the following statements:
X: ‘Exactly one of us is a Knave.’
Y : ‘Exactly two of us are Knaves.’
Z: ‘All of us are Knaves.’
What type is each?

H. Philip knows that the island has a chief and tries to find him. He does not know
whether the chief is a Knight or a Knave.
He narrows the search down to two brothers, Og and Bog. They make the following
statements:
Og: ‘Bog is the chief and he is a Knave.’
Bog: ‘Og is not the chief and he is a Knight.’
Who is the chief? Is he a Knight or a Knave?
Individual Questions Solutions
1. Answer: 881, since the only way two primes add to an odd number is for one of them
to be 2.

2. Answer: 666.
 
bπc × (bπc × bπc + dπe) × bπcdπe − dπebπc + bπc
= 3 × (3 × 3 + 4) × (34 − 43 ) + 3

= 3 × (9 + 4) × (81 − 64) + 3

= 3 × 13 × 17 + 3

= 663 + 3 = 666. [This was a devil of a question!]

3. Answer: 16. Adam got 24 · 3 − 6 = 66 points; so Eve got 44 points. Let x be the
number Eve got wrong. Then she got 36 − x right, giving her 108 − 3x points, and she
was penalised x points, so her final score is 44 = 108 − 4x. Hence 4x = 64 or x = 16.

4. Answer: 648. The leading digit can be any digit except 0 (9 possibilities); then the
middle digit can be any digit except what was chosen for the first digit (9 possibilities),
and the units digit can be any digit except what was chosen for the first two digits
(8 possibilities), i.e. there are 9 · 9 · 8 = 648 3-digit positive integers with no repeated
digits.

5. Answer: 54. The 2-digit multiples of 4 are: 12, 16, 20, 24, . . . , 96, a total of 22 numbers.
As they increase by the same amount, the sum of the first and the last, the second and
the second-last, etc. is always 108, which implies that their mean will be 11 × 108/22 =
54.
Alternative 1. The 2-digit multiples of 4 are: 12, 16, . . . , 96. Observe that since they
are in arithmetic progression, the means of first and last, second and second-last, etc.
are all the same, which implies the overall mean is given by the mean of first and last,
namely:
12 + 96
= 54.
2
Alternative 2. The total is
12 + 16 + · · · + 96 = 4(3 + 4 + · · · + 24), a total of 22 numbers
= 4(1 + 2 + 3 + · · · + 24 − 3)
 24 · 25 
=4 −3
2
= 4 · 297
4 · 297
∴ mean =
22
2 · 297
=
11
= 2 · 27 = 54.

6. Answer: 10. The digit sum of the total is 8 + a + 4 + 2 + b + 3 = 17 + (a + b). So


a + b = 1 or 10, larger is 10.

7. Answer: 60. Let x be the radius of the circle, and hence


half the side length of the square. Let O be the centre of
the circle and square. Let B be the top right corner of
the rectangle. Since the length of the rectangle is double O
the height, B must lie on the straight line between A
and the middle of the right-hand side of the square, and
so B lies at the intersection of this line and the circle. B
X
Extend the top of the rectangle from B to a point X A
on the vertical diameter of the circle. Then BXO is a
right-angled triangle with sides x − 12, x − 6 and x.
Pythagoras’ Theorem gives
(x − 12)2 + (x − 6)2 = x2
0 = x2 − 36x + 180
= (x − 30)(x − 6).
Hence, the solutions are x = 6 and x = 30, of which x = 6 gives a degenerate triangle
with B on both the circle and square, contrary to the given information. So the circle
has radius 30 and the square has side length 60 cm.

8. Answer: 59. Let n be the number of bets Gertrude makes on the Beagles. To make a
profit when the Beagles win she needs n×1.70 > 100 which means n > 100/1.7 ≈ 58.8;
so she’ll need to make at least 59 bets on the Beagles. On the other hand, if the Rockers
win, she needs (100 − n) × 2.50 > 100, so that n < 60. Hence the only solution is
n = 59.

9. Answer: 48. HE = 50, since in triangle AEH, 2(7, 24, 25) is a Pythagorean triple.
Therefore EF = 50, since EF GH is a rhombus, implying EB = 40, since in triangle
EBF , 10(3, 4, 5) is also a Pythagorean triple.
∴ P Q = 48 − 40 = 8
P S = 30 − 14 = 16
∴ Perimeter(P QRS) = 2(8 + 16) = 48.
10. Answer: 502. To give a zero on the end of a product, we need multiples of 5 and 2
as our inputs. There are clearly a surplus of multiples of 2 compared to the relatively
scarce multiples of 5, as we examine the numbers from 1 to 2015, so we are really
looking for how many multiples of 5 we have available to pair up with multiples of 2
and yield our zeros.
The number of 5s in 2015! is
       
2015 2015 2015 2015
+ + + = 403 + 80 + 16 + 3
5 25 125 625
= 502.

So 2015! ends in 502 zeros.

11. Answer: 200. Let v be Sahria’s bicycle speed. Essentially, at speed v/3 she takes
4 minutes longer to cover 400 metres, i.e.

400 400
4= −
v/3 v
∴ v = 300 − 100
= 200

So Sahria rides her bicycle at 200 m/min.


Alternatively, say the distance to school is d metres and that Sahria rides her bike
at v metres per minute. So she normally takes d/v minutes to get to school. On the
day of the calamity she goes 2d/3 metres by bike, which takes 2d/(3v) minutes, then
d/3 metres at v/3 metres per minute which takes d/v minutes. The total time taken
is therefore
2d d
+ minutes,
3v v

which is 5 minutes longer than usual. So we have

2d d d
+ = + 5,
3v v v

which implies 2d/3 = 5v. If she had travelled 400 metres further by bike she would
have spent

2d/3 + 400
minutes,
v

riding and

d − (2d/3 + 400)
minutes,
v/3
walking. She would then have arrived 1 minute later than usual. So we have
d 2d/3 + 400 d − (2d/3 + 400)
+1= +
v v v/3
2d/3 + 400 3(d/3 − 400)
= +
v v
2d
∴d+v = + 400 + d − 1200
3
2d
v + 800 =
3
= 5v
∴ 800 = 4v
v = 200.

So Sahria rides her bike at 200 m/min.

√ √
12. Answer: (a) 144/49, (b) ab/( a + b)2 .

(a) The horizontal distance between the centres of the larger two wheels can be rep-
resented as
p √
(16 + 9)2 − (16 − 9)2 = 32 · 18 = 24.
Similarly, the horizontal distance between the largest and smallest wheel centres is
p √ √
(16 + x)2 − (16 − x)2 = 32 · 2x = 8 x,

and the horizontal distance between the medium and smallest wheel centres is
p √ √
(9 + x)2 − (9 − x)2 = 18 · 2x = 6 x.

But the first horizontal distance is the sum of the other two. So,
√ √
24 = 8 x + 6 x

= 14 x

12 = 7 x
144
= x.
49
R
(b) In general, if two wheels of radii R and r are R−r r
on a plane and touching, then the horizontal
distance between the centres is
p
d(R, r) = (R + r)2 − (R − r)2
q 
= (R + r + R − r) R + r − (R − r)

= 2R · 2r

= 2 Rr
So, if the radii of the larger wheels are a and b, the radius x of the smallest wheel
satisfies,
d(a, b) = d(a, x) + d(x, b)
√ √ √
∴ 2 ab = 2 ax + 2 bx
√ √ √
ab = ax + bx
√ √ √
= x( a + b)

√ ab
x= √ √
a+ b
ab
∴x= √ √ .
( a + b)2
Team Question Solutions
A Knave’s Tale

A. Answer: J is a Knight.
Suppose that I is a Knight. Then, his statement that both are Knaves is a contradic-
tion. So I is a Knave, and hence the statement that both are Knaves is a lie. Since
we now know I is a Knave, J is not a Knave, and hence is a Knight.

B. Answer: L is a Knave.
Consider two cases:
Case 1: K is a Knight. Then his statement is true. So L is a Knave.
Case 2: K is a Knave. Then his statement that at least one is a Knave is true,
contradicting that as a Knave he would be lying. Hence this case is impossible.
So we deduce, L is a Knave.

C. Answer: N is a Knight.
Consider two cases:
Case 1: M is a Knight. Then they are both the same; so N is a Knight.
Case 2: M is a Knave. Then they are not the same, and again N is a Knight.
Hence (in both cases) N is a Knight.

D. Answer: Yes, their answers would be the same.


We have three cases:
Case 1: both are Knights. Then both would say yes.
Case 2: both are Knaves. Then both would say yes.
Case 3: one is a Knight and the other a Knave. Then both would say no.
So in all cases, the two islanders’ answers would be the same.

E. Answer: T is a Knight.
We saw in the preamble, that whether R is a Knight or a Knave, he would say he is
a Knight. So S’s reply is false, and hence T is telling the truth. Therefore, T is a
Knight.

F. Answer: W is a Knight.
Since W says that V is lying, W must be the opposite of V , because if V tells the
truth then W lies in saying V lied, and if V lies, then W tells the truth in saying that
V lied. So among V and W , there is one Knight and one Knave.
Now, consider two cases:
Case 1: U is a Knight. Then he would say there is one Knave.
Case 2: U is a Knave. Then there would be two Knaves, but being a Knave, he would
lie, and say that there are one or three Knaves.
Consequently, V lies and W tells the truth. So W is a Knight.
G. Answer: X and Z are Knaves, Y is a Knight.
Since all the statements are different, at most one is true. So at least two are Knaves.
If all three are Knaves, then Z’s statement would be true, which would make him
a Knight who claims to be a Knave, a contradiction. So Z’s statement is a lie, and
hence exactly two are Knaves. Thus Y ’s statement is true, and hence Y is a Knight,
and X and Z are Knaves.

H. Answer: Og is the chief and he is a Knave.


First suppose Og is a Knight. Then by his statement, Bog would be the chief and
a Knave. But then Og would not be the chief, and so Bog’s statement that Og is
not the chief but is a Knight would be true, which is a contradiction, since we had
deduced Bog is a Knave.
So Og is a Knave and consequently Bog’s statement is a lie. Hence Bog is also a
Knave.
If Bog were the chief, he would be the chief and a Knave; so Og’s statement would be
true. But this is a contradiction, since Og is a Knave.
Hence Og is the chief and he is a Knave.
Note. For a statement that is the and of two sub-statements to be false, only one of
the two sub-statements need be false. Og is a Knave, and so his statement is false;
this being due to Bog not being the chief, though Bog is a Knave. Similarly, since Bog
is a Knave, his statement is false; this being due to Og not being a Knight, though
he is the chief.
Western Australian
Junior Mathematics Olympiad 2016
Individual Questions 100 minutes

General instructions: Except possibly for Question 12, each answer in this part is a
positive integer less than 1000. No working is needed for Questions 1 to 11. Calculators
are not permitted. In general, diagrams are provided to clarify wording only, and are not
to scale.
Write your answers for Questions 1–11, and solution for Question 12 on the front and
back, respectively, of the Answer Sheet provided.

1. Find the value of x in the following diagram.


36◦

x◦
128◦
[1 mark]

2. An icecream dessert is made up of a cone and a scoop of icecream such that the scoop
of icecream is a perfect hemisphere placed atop the cone.
If the wafer biscuit which makes up the cone is 100 mm along the outside (slant edge)
from the top of the wafer to its tip and the radius of the hemisphere is 28 mm, how
many millimetres tall is the icecream dessert from the top of the icecream scoop to the
tip at the base? [1 mark]

3. I am thinking of two numbers between 1 and 20. The first is one less than a perfect
square and is not prime. The second is one more than a perfect square and is prime.
The sum of the two numbers is divisible by 4 but not by 8.
What is the product of the two numbers? [1 mark]

4. A jigsaw puzzle is 8 × 6, making 48 pieces altogether in a rectangular pattern. Each


corner piece is connected to two other pieces, each edge piece which is not on a corner
is connected to three other pieces and each of the remaining pieces is connected to four
other pieces.
How many connections are there altogether? Be careful not to count the connections
twice: if two pieces are connected to each other that’s only one connection. [1 mark]

5. A school bus has 45 bench seats. Each bench seat can take two children or one child
with their backpack.
If 23 of the children have backpacks, how many children can be seated on the bus?
[2 marks]
6. Mr Throttlebottom pays a bill using internet banking. The bill is for a two digit whole
number of dollars, say $ab. He accidentally inserts an extra digit after the b. Later he
is annoyed to discover that he has overpaid the bill by $647.
How many dollars should he have paid? [2 marks]

7. W XY Z is a square with P V ⊥ XY W X
and P W = P Z = P V = 10 cm.
How many cm2 is the area of W XY Z?

P V

Z Y
[2 marks]

8. How many positive integers less than 300 000 are there that have the digits 5, 6 and 7
next to each other and in that order? [2 marks]

9. In a right-angled triangle ABC, a point M on the hypotenuse BC is such that


AM ⊥ BC. Also, M C is 8 cm longer than BM , and the ratio AB : AC = 3 : 5.
How many centimetres is the length of the hypotenuse? [3 marks]

10. In a 5 000 m race, the winner finishes 200 m ahead of second and 600 m ahead of third.
Assuming the runners run at a constant speed, to the nearest integer how many metres
ahead of third will second finish? [3 marks]

11. Two numbers x and y satisfy three of the following equations


x
x + y = 63, x − y = 47, xy = 392, = 8,
y
but do not satisfy the remaining one.
What is the value of x? [3 marks]

12. For full marks explain how you found your solution.
Wilberforce walks 1 metre east from his initial position, then 2 metres north, then 3
metres west, then 4 metres south, then 5 metres east, then 6 metres north, and so on,
(so that he walks one metre further each time before turning 90◦ to his left), until he
finally walks 41 metres east.
How many metres in a straight line path is Wilberforce’s final position from his initial
position? [4 marks]
Western Australian
Junior Mathematics Olympiad 2016
Team Question 45 minutes

General instructions: Calculators are (still) not permitted.

Triangles with Integer Sides


This problem is about triangles all of whose sides have integer length, which we call
integer triangles.
Remember the triangle inequality:
There exists a triangle with sides of length a, b, c if and only if each length
is less than the sum of the other two.
We say that a triple (a, b, c) of positive integers is a good triple if there is an integer
triangle with sides of lengths a, b and c and a ≤ b ≤ c. The perimeter of a good triple
(a, b, c) is the perimeter of the triangle with sides a, b and c, that is a + b + c. We call a
good triple an odd triple if the perimeter is odd, and an even triple if the perimeter is
even. So (3, 4, 4) is an odd triple and (2, 2, 2) is an even triple.
If n is any positive integer, let T (n) be the number of good triples of perimeter n. For
example T (3) = 1 because the only side lengths of an integer triangle of perimeter 3 are
(1, 1, 1) and T (5) = 1 because every integer triangle with perimeter 5 is congruent to a
triangle with side lengths 1, 2, 2.

A. Explain why T (4) = 0.

B. Show that T (6) = 1.

C. Show that if (a, b, c) is a triple of positive integers satisfying a ≤ b ≤ c and a + b > c,


then (a, b, c) is a good triple.

D. Explain why a good triple (a, b, c) of perimeter n has a ≤ n/3.

E. Let (a, b, c) be an even triple. Explain why a ≥ 2.

F. Show that T (12) = 3.

G. Fill in the table on the back of the cover page listing all good triples and T (n) for
n = 1, 2, . . . , 14.
H. Let (a, b, c) be a good triple with perimeter n. Find the largest possible value for v
and smallest value for w such that
vn ≤ c < wn,
where v and w are fractions. Explain why this double inequality (with the values you
found for v and w) holds for all good triples.

I. Let (a, b, c) be an even triple. Explain why (a − 1, b − 1, c − 1) is a good triple which


is odd.

J. Let (a − 1, b − 1, c − 1) be an odd triple. Explain why (a, b, c) is a good triple that is


even.

K. Explain why if n is even then T (n) = T (n − 3).


Individual Questions Solutions
1. Answer: 88. Drawing in another parallel through the angle with value x◦ , we see that
x◦ is the sum of an alternate angle and a cointerior angle, i.e.
x = 36 + (180 − 128) = 88.

2. Answer: 124. The vertical height of the cone is


√ p
1002 − 282 = 42 (252 − 72 )
√ ......
..............................................................
..............
28 ......
...
.....
= 42 · 242 , using the Pythagorean ......
.............. .....
..............................................................
..
.

triple: 7, 24, 25
= 4 · 24
= 96. 100
Hence the total height is 96 + 28 = 124 mm.
Alternatively, the square of the vertical height of the cone is
1002 − 282 = (100 − 28)(100 + 28)
= 72 · 128
= 23 · 32 · 27
= (25 · 3)2

∴ 1002 − 282 = 25 · 3
= 96.
Hence the total height is 96 + 28 = 124 mm.

3. Answer: 75. The first number must be 8 or 15. The second must be 2, 5 or 17. The
possible sums are 8 + 2 = 10, 8 + 5 = 13, 8 + 17 = 25, 15 + 2 = 17, 15 + 5 = 20 and
15 + 17 = 32; of these sums, only 20 is divisible by 4 but not by 8.
So the numbers are 15 and 5 and their product is 75.

4. Answer: 82. We have columns of 6 pieces vertically and rows of 8 pieces horizontally.
Counting vertical connections, we have 7 columns of 6 connections, giving 42 connec-
tions. Counting horizontal connections, we have 5 rows of 8 connections, giving 40
more connections. Hence the total number of connections is 40 + 42 = 82.
Generally, for an x × y jigsaw we’ll get (x − 1)y + (y − 1)x connections.
Alternatively, the four corner pieces contribute 4 × 2 = 8 connections.
The pieces on the four sides contribute (4 + 4 + 6 + 6) · 3 = 60 connections.
There are 4 × 6 = 24 other pieces and these will contribute 4 × 24 = 96 connections.
Adding these up gives a total of 8 + 60 + 96 = 164 connections. However, in this
calculation we’ve counted each connection twice so we need to divide by 2. So the
answer is 164/2 = 82.
5. Answer: 54. Let the number of children be x. Then 2x/3 have backpacks and x/3 are
without backpacks. These x children need 2x/3 + x/6 = 5x/6 bench seats. Since there
are 45 seats,

5x
= 45
6
x = 65 · 45
= 54.

So 54 children are seated on the bus.

6. Answer: 71. Let the extra digit be denoted by c. Then he’s paid $(100a + 10b + c)
instead of $(10a + b), so

647 = (100a + 10b + c) − (10a + b)


= 9(10a + b) + c.

This means that c is the remainder when 647 is divided by 9, so c = 8, then

647 − 8 639
10a + b = = = 71.
9 9
So he should only have paid $71.

7. Answer: 256. Let s be the side length of the square. Then we want s2 where

(s − 10)2 + (s/2)2 = 102


5 2
4
s − 20s = 0
5
4
s = 20
s = ( 45 · 20)2
2

= 162 = 256.

8. Answer: 900. The numbers can be of the form x567yz, xy567z or xyz567, where in
each case, x is empty (i.e. 0), or 1 or 2, and each of y and z can be any digit, giving
3 × 10 × 10 choices for xyz. Hence there are 3 × 3 × 10 × 10 = 900 possibilities.
9. Answer: 17. Let x be the length of AM and y the B
y
length of BM . The three triangles ABC, BM A
M
and AM C are similar. So
x y+8 5
= = , x y+8
y x 3
and the length we are after is BC = 2y+8. Hence
5y 5x A C
x= and y + 8 =
3 3
25y
=
9
25y = 9y + 72
16y = 72
2y = 9
BC = 2y + 8 = 17.

10. Answer: 417. Let u, v and w be the speeds of the first, second and third places, and
let t1 be the time taken by first place. Then, since
distance
time =
speed
5000 5000 − 200 5000 − 600
(t1 =) = =
u v w
4800 24
∴ v= u= u
5000 25
4400 22
w= u= u
5000 25
Now let d be the distance between second and third at the time t2 when second place
crosses the finish line. Then
5000 5000 − d
(t2 =) =
v w
25 1 25 1
5000 · · = (5000 − d) · ·
24 u 22 u
22
5000 · = 5000 − d
24
 22  2 1250
d = 5000 1 − = 5000 · = = 416 23 m,
24 24 3
which rounds to 417 m.
Alternatively, let t1 , u, v and w be as above. Then, as before
5000 4800 4400
t1 = = =
u v w
4800 4400
∴ v= , w= .
t1 t1
To run 200 m, the second runner needs time
200 t1 t1
= 200 · = .
v 4800 24
In that time, the third runner will run a distance of

t1 4400 t1 4400 550


w· = · = = = 183 13 m,
24 t1 24 24 3

so that in the time the second runner runs the last 200 m, the third runner runs 183 31 m.
Before running those respective final distances the second and third runners were 400 m
apart. So when the second runner crosses the finish line, the second and third runners
are
400 + 200 − 183 13 = 416 23 ≈ 417 m

apart.

11. Answer: 56. Listing the equations in order to label them:

x + y = 63 (1)
x − y = 47 (2)
xy = 392 (3)
x
=8 (4)
y

Suppose both (1) and (2) are satisfied. Then

(1) + (2) : 2x = 110


x = 55
(1) : y = 63 − 55 = 8.

But then xy = 440 6= 392 and x/y = 55/8 6= 8, so that neither (3) nor (4) is satisfied.
So one of (1) and (2) is not satisfied, but both of (3) and (4) must be satisfied. Hence,

(3) · (4) : x2 = 392 · 8


= 142 · 2 · 8
= (14 · 4)2
x = ±56

If x = −56 then by (4), y = −7, so that

x + y = −63 and x − y = −49,

meaning neither (1) nor (2) is satisfied. Hence x = 56, and so by (4), y = 7 and (1) is
satisfied, but not (2).
12. Answer: 29. The net distance Wilberforce
walks eastwards is
1 − 3 + 5 − 7 + · · · + 41
N
= (1 − 3) + (5 − 7) + · · · + (37 − 39) + 41
= 10 × (−2) + 41
= 21 m S
The net distance Wilberforce walks north-
wards is
2 − 4 + 6 − 8 + · · · + 38 − 40 20 m
= (2 − 4) + (6 − 8) + · · · + (38 − 40)
= 10 × (−2)
21 m F
= −20 m
So he finishes 20 m south of his original po-
sition. His distance from his initial position
is therefore, using Pythagoras’ Theorem,
p √ √
212 + (−20)2 = 441 + 400 = 841 = 29.
Remark. Here we have an integer Pythagorean triple (a, b, c) with b = a + 1, so that
a2 + (a + 1)2 = c2
2a2 + 2a + 1 = c2
2a(a + 1) = c2 − 1
= (c − 1)(c + 1)
Since the left hand side of the last equation is even, so is the right hand side, and
hence at least one of the factors c − 1 and c + 1 is even. But c − 1 and c + 1 differ by
2, and so must be of the same parity, and hence are both even. Looking again at the
lefthand side, we see a and a + 1 are of opposite parity. So one is odd and the other
even. So the left hand side is in fact divisible by 4. Thus the last equation reduces to:
a(a + 1) c−1 c+1
= · ,
2 2 2
where the left hand side is a triangular number and the right hand side is the product
of consecutive integers. For the numbers in our problem, we have
20 · 21
T20 = = 14 · 15, and c = 29.
2
So a natural question to ask is:
How frequently does an integer Pythagorean triple of the form (a, a + 1, c)
happen?
The answer is that it’s rare. The following are the triples for c < 10 000:
(3, 4, 5), (20, 21, 29), (119, 120, 169), (696, 697, 985), (4059, 4060, 5741).
Team Question Solutions
Triangles with Integer Sides

A. To have integer sides that sum to 4, a triangle (a, b, c) would have to have sides of
lengths 1 or 2 and no such triangle has perimeter 4. That is, the only potential
triple satisfying a ≤ b ≤ c is (1, 1, 2) which violates the triangle inequality, since
1 + 1 = 2 6> 2.
So T (4) = 0.

B. Systematically listing integer partitions of 6 with a ≤ b ≤ c, we have


(1, 1, 4), (1, 2, 3), (2, 2, 2).
Of these only (2, 2, 2) satisfies the triangle inequality. ∴ T (6) = 1.
Alternatively, if c ≥ 3 then since a + b > c we have a + b + c > 6, a contradiction.
Hence, c is at most 2, but now if any of a, b or c < 2 then a+b+c < 6, a contradiction.
Hence, a = b = c = 2 and so (2, 2, 2) is the only good triple for perimeter 6.
∴ T (6) = 1.

C. In order to show that a triple with a ≤ b ≤ c is good, we need to show the triangle
inequality holds, i.e.
a < b + c (1)

b<a+c (2)

c<a+b (3)
Inequality (3) is assumed. However, the condition a ≤ b ≤ c with a, b, c positive
implies
b ≤ c < c + a,
so that (2) is automatically satisfied. Similarly,
a ≤ b < b + c,
so that (1) is automatically satisfied.

D. Suppose for a contradiction that a > n/3. Then since a is the smallest side, b > n/3
and c > n/3. Hence
a + b + c > n/3 + n/3 + n/3 = n,
i.e. the perimeter is larger than n, a contradiction.
∴ a ≤ n/3.

E. Suppose (a, b, c) is an even triple with a = 1. Since 1 + b + c is even, b and c have


opposite parity, so c − b ≥ 1 and hence 1 + b ≤ c, contradicting the triangle inequality.
Hence a ≥ 2.

F. By D. and E. we have 2 ≤ a ≤ 4. With this restriction on a, systematically listing


integer partitions of 12 with a ≤ b ≤ c, we have
 (2, 2, 8), (2, 3, 7), (2, 4, 6), (2, 5, 5),

(3, 3, 6), (3, 4, 5),



(4, 4, 4).
Of these only (2, 5, 5), (3, 4, 5), and (4, 4, 4) satisfy the triangle inequality.
∴ T (12) = 3.

G.
n T (n) Triples
1 0 —
2 0 —
3 1 (1, 1, 1)
4 0 —
5 1 (1, 2, 2)
6 1 (2, 2, 2)
7 2 (1, 3, 3), (2, 2, 3)
8 1 (2, 3, 3)
9 3 (1, 4, 4), (2, 3, 4), (3, 3, 3)
10 2 (2, 4, 4), (3, 3, 4)
11 4 (1, 5, 5), (2, 4, 5), (3, 3, 5), (3, 4, 4)
12 3 (2, 5, 5), (3, 4, 5), (4, 4, 4)
13 5 (1, 6, 6), (2, 5, 6), (3, 4, 6), (3, 5, 5), (4, 4, 5)
14 4 (2, 6, 6), (3, 5, 6), (4, 4, 6), (4, 5, 5)

H. Answer: n/3 ≤ c < n/2, i.e. v = 13 and w = 12 .


Checking the table in G., we see for instance that for n = 9, c can be 3 or 4, for
n = 12, c can be 4 or 5, and for n = 7, c can be only 3. From these data we guess the
above values for v and w.
Suppose for a contradiction that c ≥ n/2. Then since a + b > c, we have
a + b + c > 2c ≥ n,
i.e. the perimeter is larger than n, a contradiction.
∴ c < n/2, i.e. w = 21 .
Now suppose c < n/3. Then since c is the largest side we have a < n/3 and b < n/3
also, and so
a + b + c < n/3 + n/3 + n/3 = n,
i.e. the perimeter is less than n, again a contradiction.
∴ c ≥ n/3, i.e. v = 31 . Note that the example (2, 2, 2) shows that we cannot do better
than v = 13 .
∴ n/3 ≤ c < n/2.

I. By C., we need only show a − 1 ≤ b − 1 ≤ c − 1 and c − 1 < (a − 1) + (b − 1). The first


statement follows from a ≤ b ≤ c. Let us now prove the second inequality. Since a + b
and c sum to an even number, a + b and c have the same parity. Moreover, c < a + b
and so a+b ≥ c+2. Hence a−1+b−1 ≥ c > c−1, and so (a−1, b−1, c−1) is a good
triple which is odd because with a + b + c even, (a − 1) + (b − 1) + (c − 1) = a + b + c − 3
is necessarily odd.
J. By C., we need only show a ≤ b ≤ c and c < a + b. The first statement follows from
a − 1 ≤ b − 1 ≤ c − 1. Let us now prove the second inequality. If (a − 1, b − 1, c − 1)
is good then a + b − 2 > c − 1 so that a + b > c + 1 > c, and since it is an odd triple,
a + b + c − 3 is odd and hence a + b + c is even. Thus (a, b, c) is a good triple which
is even.

K. First we can check in table G. that the statement is true for n = 4, 6, 8, 10, 12, 14.
Since we only defined T (n) for positive integers n, we need n ≥ 4 in order that the
right hand side of T (n) = T (n − 3) be defined.
By I., if n is even, for every good triple (a, b, c) for perimeter n there is a good triple
(a − 1, b − 1, c − 1) of perimeter n − 3. Moreover, distinct triples (a, b, c) yield distinct
triples (a − 1, b − 1, c − 1). Thus we have T (n) ≤ T (n − 3).
If n is even, then n − 3 is odd, and so by J., for every good triple (a − 1, b − 1, c − 1)
of perimeter n − 3, there is a good triple (a, b, c) of perimeter n. Moreover, distinct
triples (a − 1, b − 1, c − 1) yield distinct triples (a, b, c). Thus we have T (n − 3) ≤ T (n).
Since we have both T (n) ≤ T (n − 3) and T (n − 3) ≤ T (n), we have
T (n) = T (n − 3), if n is even.
Western Australian
Junior Mathematics Olympiad 2017
Individual Questions 100 minutes

General instructions: Except possibly for Question 12, each answer in this part is a
positive integer less than 1000. No working is needed for Questions 1 to 11. Calculators
are not permitted. In general, diagrams are provided to clarify wording only, and are not
to scale.
Write your answers for Questions 1–11, and solution for Question 12 on the front and
back, respectively, of the Answer Sheet provided.

1. Let ABC be a right-angled triangle with right angle at A. Let D be on the line BC,
with B between C and D.
If the angle ∠ACB is 6 degrees more than a quarter of the angle ∠ABD, how many
degrees is ∠ABC? [1 mark]

2. When Geppetto adds 6 times his age 6 years from now to 7 times his age 7 years from
now, he gets 14 times his current age.
How old is Geppetto now? [1 mark]

3. The squares of two consecutive integers differ by 2017.


Of the two consecutive integers, one is odd and one is even; what is half of the even
one? [1 mark]

4. How many positive integers y are there such that the least common multiple of 280
and y is 1400? [2 marks]

5. A two-digit number ab is called good if a2 + b2 = 65.


What is the sum of all good numbers? [2 marks]

6. The average of the ages of the mother, the father and their three children is 21, whereas
the children alone average 11.
How many years is the age of the father, given that the father is 4 years older than
the mother? [2 marks]

7. If x2 + xy + y 2 = 92 and x + y = 11, what is xy? [2 marks]


8. The four sides of a rectangle of area 630
are divided into halves, thirds, quarters
and fifths respectively, as in the diagram.
What is the area of the shaded quadrilat-
eral? Enter 999 if you think it depends on
the lengths of the sides of the rectangle.
[2 marks]

9. Ayesha and Brian live 5 km apart and plan to meet at 1:00 pm. They decide to leave
their respective homes at the same time and cycle towards each other till they meet.
Ayesha cycles at 8 km/h and Brian at 7 km/h.
How many minutes after 12 noon should they leave home? [2 marks]

10. A block of wood in the form of a 5 cm × 8 cm × 13 cm rectangular prism has all six of
its faces painted pink.
If the wooden block is cut into 1 cm3 cubes, how many of these would have some pink
paint on them? [3 marks]

11. Janine has a box containing 49 red balls and 49 green balls and a jar containing one
red ball and one green ball.
She performs the following multi-step operation:
(i) First she closes her eyes, takes one ball from the jar, looks at it, notes its colour,
and puts it in the box.
(ii) If the ball moved to the box was red, she closes her eyes, takes two balls from the
box and puts them in the jar;
otherwise, if the ball moved to the box was green, she takes two red balls from
the box and puts them in the jar.
Suppose Janine repeats this operation 36 times and finds there are 36 red balls in the
box, how many green balls will there be in the box? [3 marks]

12. For full marks explain how you found your solution.
Two sides of an obtuse triangle have lengths 9 and 40.
The length of the third side is an integer.
How many such triangles are possible?
Note. An obtuse triangle is a triangle with one obtuse angle.
An angle θ is obtuse if 90◦ < θ < 180◦ . [4 marks]
Western Australian
Junior Mathematics Olympiad 2017
Team Question 45 minutes

General instructions: Calculators are (still) not permitted.

Prime Circles for Nora 1


Nora writes the numbers 1, 2, 3 and 4 in a circle as in the
diagram opposite, and notices that each pair of adjacent
numbers has a prime sum: 2 4
1 + 2 = 3, 2 + 3 = 5, 3 + 4 = 7, 4 + 1 = 5.

3
We call a prime circle for n an arrangement of the integers from 1 to n around a circle
such that the sum of any two adjacent numbers is a prime.
Two prime circles for n are considered the same (or equivalent) if one can be obtained
from the other by a rotation or a reflection. Otherwise we say the circles are different.
For instance, the three circles below are equivalent to the one above, via a clockwise
rotation, an anticlockwise rotation, and a reflection, respectively.

2 4 1

3 1 1 3 4 2

4 2 3

Clearly, using rotations we can decide that the number 1 is in a fixed location, for instance
at “12 o’clock” as on the answer sheet. Moreover, using the reflection in the line through
12 and 6 o’clock, two circles that have the numbers in the same order starting with 1, but
one clockwise and one anti-clockwise, are equivalent.
Finally, you might find convenient to write a prime circle for n as a sequence of n numbers,
starting at 12 o’clock and going clockwise. For example, the first and the last prime circles
above can be written as (1, 4, 3, 2) and (1, 2, 3, 4) respectively.

A. How many different prime circles for 4 are there?

B. Find a prime circle for 6.

C. How many different prime circles for 6 are there?


D. If n is odd, explain why there is no prime circle for n.

E. Find a prime circle for 8.

F. How many different prime circles for 8 are there?

G. Find a prime circle for 10.

H. Show that if you swap 3 and 9 from a prime circle for 10 you get a different prime
circle for 10.

I. Are there other numbers that can be swapped in a prime circle for 10?

J. Show that the number of different prime circles for 10 is divisible by 4.

K. Show that the number of different prime circles for 12 is divisible by 16.
Individual Questions Solutions
1. Answer: 52.
C Let ∠ABD = 4x.
Then ∠ACB = x + 6 and
we want ∠ABC = 180◦ − 4x.
x+6 Since for a triangle, an exterior angle is the
sum of the interior opposite angles,
4x = x + 6 + 90
B 3x = 96
A 4x
x = 32
∴ ∠ABC = 180 − 4x = 180◦ − 128◦

D = 52◦

2. Answer: 85. Let Geppetto’s age now be x. Then


6(x + 6) + 7(x + 7) = 14x
36 + 49 = x
∴ x = 85.

3. Answer: 504. Let n, n + 1 be the consecutive integers. Then


2017 = (n + 1)2 − n2
= 2n + 1
= n + (n + 1)
∴ n = 2016/2
= 1008
i.e. the even one of the two consecutive integers is n = 1008 and half of it is 504.

4. Answer: 8.
1400 = 8 · 25 · 7 = 23 · 52 · 7 and 280 = 23 · 5 · 7.
Hence y = 2a · 5b · 7c , where we require
max{3, a} = 3 =⇒ a = 0, 1, 2 or 3 (4 possibilities)
max{1, b} = 2 =⇒ b = 2 (1 possibility)
max{1, c} = 1 =⇒ c = 0 or 1 (2 possibilities)
Thus the number of choices for the triple (a, b, c) (and hence of y) is 4 × 1 × 2 = 8.

5. Answer: 220. Since 92 = 81, the digits a, b ∈ {1, 2, . . . , 8}.


The larger of the 2 digits is at least 6, since 52 < 12 · 65.
Now 65 − 62 = 29, is not a square.
This leaves checking whether 7 and 8 could be one of the digits. Indeed,
72 + 42 = 65 = 82 + 12 .
So there are 4 good numbers: 18, 47, 74, and 81, and their sum is
18 + 47 + 74 + 81 = 220.
6. Answer: 38. Let m, d, c1 , c2 , c3 be the ages of the mother, father and three children,
respectively. Then
1
5
(d + m + c1 + c2 + c3 ) = 21
and 31 (c1 + c2 + c3 ) = 11
∴ 15 (d + m + 3 · 11) = 21
d + m + 33 = 105
d + m = 72
d + d − 4 = 72, since d = m + 4
2d = 76
d = 38.
So the father is 38.

7. Answer: 29.
xy = (x2 + 2xy + y 2 ) − (x2 + xy + y 2 )
= (x + y)2 − (x2 + xy + y 2 )
= 112 − 92
= 121 − 92 = 29.
Remark. The solutions for x, y are irrational, being the solutions of the quadratic
equation
u2 − 11u + 29 = 0,
√ √
namely {x, y} = { 21 (11 + 5), 21 (11 − 5)}.

8. Answer: 336. Let the length and breadth of the rectangle be ` and b, respectively.
Then the shaded area is the area of the rectangle minus the areas of the four unshaded
triangles, i.e.
Shaded area = 630 − 21 · 34 ` · 25 b − 21 · 12 ` · 35 b − 21 · 12 ` · 13 b − 12 · 14 ` · 23 b
= 630 1 − 21 ( 34 · 25 + 12 · 35 + 21 · 13 + 14 · 32 ) , since `b = 630


= 630 1 − 21 ( 103 3
+ 16 + 16 )

+ 10
= 630 1 − 21 ( 35 + 13 )


= 630(1 − 12 · 14
15
)
8
= 630 · 15
= 21 · 16
= 336.

9. Answer: 40. The total of their speeds is 15 km/h.


So they will reduce the distance 5 km between them to zero, in 31 h = 20 minutes.
Therefore they should leave home at 12:40 pm, i.e. 40 minutes after 12 noon.
Alternatively, suppose Ayesha cycles d km, so that Brian cycles (5 − d) km.
Then the time t in hours they both travel is
t = d/8 = (5 − d)/7
7d = 8(5 − d)
= 40 − 8d
15d = 40
8
d= 3
1
∴ t = d/8 = h 3
= 20 min
and so they each leave 20 minutes before 1:00 pm and hence 40 minutes after 12 noon.

10. Answer: 322. The number of partially pink 1 cm3 cubes is the total number of cubes
minus the number of cubes in the inner prism with no paint on them, i.e.
No. of partially pink faces
= 5 × 8 × 13 − (5 − 2) × (8 − 2) × (13 − 2)
= 520 − 3 × 6 × 11
= 520 − 198
= 322
Alternatively, the number of partially pink 1 cm3 cubes is the number in the top and
bottom faces, plus the number in the front and back faces not in the top and bottom
faces, plus the number in the left and right faces neither in the top or bottom, or front
or back faces, i.e.
No. of partially pink faces
= 2 × 13 × 8 + 2 × 13 × (5 − 2) + 2 × (8 − 2) × (5 − 2)

= 2 × 13 × (8 + 5 − 2) + (8 − 2) × (5 − 2)
= 2 × (13 × 11 + 6 × 3)
= 2(143 + 18)
= 322.

11. Answer: 26. The overall effect of each operation is that one ball is added to the jar;
so after 36 operations there are 2 + 36 = 38 balls in the jar.
(Note that 36 operations can certainly be carried out, since, firstly, 36 < 98 so that
there are always sufficient balls in the box to carry out the operation. Secondly, since
the number of balls in the jar is never more than 36, and one of the balls in the jar is
initially red, at any one time at most 35 of the red balls initially in the box can have
been removed; so that there are always more than 2 red balls in the box. Thirdly, the
jar initially has 2 balls and hence never less than 1 ball.)
There are 100 balls altogether; so there are now 100 − 38 = 62 balls in the box.
Since 36 are red the remaining 62 − 36 = 26 are green.
12. Answer: 14. Let the third side be c. To form a valid triangle, we require
9 + c > 40 and 9 + 40 > c
∴ 31 < c < 49
In order for a triangle with sides a, b and `, with ` the longest side, to be obtuse, `
must be longer than the hypotenuse of a right angled triangle with legs a and b, i.e.
`2 > a2 + b2
Thus if c < 40 we require
402 > 92 + c2
c2 < 402 − 92
= 1519
c ≤ 38.
And, if c > 40 we require
c2 > 92 + 402 .
Note that 9, 40, 41 is a Pythagorean triad. To see this observe that,
92 = 40 + 41
= (41 + 40)(41 − 40)
= 412 − 402
92 + 402 = 412
Thus, if c > 40, to have an obtuse triangle c > 41.
Therefore, we have 32 ≤ c ≤ 38 or 42 ≤ c ≤ 48 which is a total of 14 possibilities.
Team Question Solutions
Prime Circles for Nora

A. 1 and 3 cannot be adjacent (as they add up to 4); so are opposite on the circle.
Similarly 2 and 4 cannot be adjacent (as they add up to 6); so they are opposite.
So up to rotation we get only (1, 2, 3, 4) or (1, 4, 3, 2), but these are the same via a
reflection.
So there is only one solution.

B. Answer: (1, 6, 5, 2, 3, 4). 1


This solution is shown in the diagram.

4 6

3 5

2
Up to equivalence the above is the only solution.

C. Two odd numbers cannot be adjacent as they would have an even sum larger than 2.
Up to rotation and reflection, we can assume a circle is (1, x, 3, y, 5, z).
4 cannot be adjacent to 5, so 4 = x.
6 cannot be adjacent to 3, so 6 = z.
Hence 2 = y.
There is only the one solution found in B.
Alternatively, two even numbers cannot be adjacent as they would sum to an even
number larger than 2.
Up to rotation and reflection, we can assume a circle is (2, x, 4, y, 6, z).
4 cannot be adjacent to 5, so 5 = z.
6 cannot be adjacent to 3, so 3 = x.
Hence 1 = y.
There is only the one solution found in B.

D. For n odd, there are more odd numbers than even numbers around the circle, so two
odd numbers will be adjacent.
Two positive odd numbers sum to an even number larger than 2.
So the circle cannot be a prime circle for n.
E. Answer: (1, 2, 3, 8, 5, 6, 7, 4) or (1, 6, 7, 4, 3, 8, 5, 2).
These two non-equivalent solutions are shown below.

1 1
4 2 2 6

7 3 5 7

6 8 8 4
5 3

Up to equivalence, these are the only correct solutions.


F. We must alternate odd and even numbers; otherwise we have adjacent numbers sum-
ming to an even number larger than 2.
Up to rotation we can have 1 at 12 o’clock.
Then up to reflection, there are only 3 possibilities for placing the other odd numbers:
(i) (1, x, 3, y, 5, z, 7, t), (ii) (1, x, 7, y, 3, z, 5, t), and (iii) (1, x, 5, y, 7, z, 3, t).
The prime sum condition to avoiding forbidden odd/even adjacent pairs:

1 − 8, 3 − 6, 5 − 4, 7 − 2, and 7 − 8.

Since 8 can neither be adjacent to 1 nor 7, case (iii) is impossible.


For case (i), 8 must be y, and
for case (ii), 8 has to be z (between 3 and 5 in both cases).
So we have:
(i) (1, x, 3, 8, 5, z, 7, t) or (ii) (1, x, 7, y, 3, 8, 5, t).
2 cannot be adjacent to 7 so we get:
(i) (1, 2, 3, 8, 5, z, 7, t) or (ii) (1, x, 7, y, 3, 8, 5, 2).
4 cannot be adjacent to 5 so we get:
(i) (1, 2, 3, 8, 5, z, 7, 4) or (ii) (1, x, 7, y, 3, 8, 5, 2).
6 cannot be adjacent to 3 so we get:
(i) (1, 2, 3, 8, 5, 6, 7, 4) or (ii) (1, 6, 7, y, 3, 8, 5, 2).
And hence we get two different solutions:
(i) (1, 2, 3, 8, 5, 6, 7, 4), or (ii) (1, 6, 7, 4, 3, 8, 5, 2)
after checking that they have no forbidden pairs.
Alternatively, we must alternate odd and even numbers; otherwise we have adjacent
numbers summing to an even number larger than 2.
Up to rotation we can have 2 at 12 o’clock.
Then up to reflection, there are only 3 possibilities for where to place the other even
numbers:
(i) (2, x, 4, y, 6, z, 8, t), (ii) (2, x, 4, y, 8, z, 6, t), and (iii) (2, x, 6, y, 4, z, 8, t).
The prime sum condition translates to avoiding forbidden odd/even adjacent pairs:

1 − 8, 3 − 6, 5 − 4, 7 − 2, and 7 − 8.

Since 7 cannot be adjacent to 2 nor 8, case (ii) is impossible.


For case (i), 7 must be y, and
for case (iii), 7 must be y (between 4 and 6 in both cases).
So we have:
(i) (2, x, 4, 7, 6, z, 8, t), or (iii) (2, x, 6, 7, 4, z, 8, t).
1 cannot be adjacent to 8 so we get:
(i) (2, 1, 4, 7, 6, z, 8, t), or (iii) (2, 1, 6, 7, 4, z, 8, t).
3 cannot be adjacent to 6 so we get:
(i) (2, 1, 4, 7, 6, z, 8, 3), or (iii) (2, 1, 6, 7, 4, z, 8, t).
5 cannot be adjacent to 4 so we get:
(i) (2, 1, 4, 7, 6, z, 8, 3), or (iii) (2, 1, 6, 7, 4, z, 8, 5).
And hence we get two different solutions:
(i) (2, 1, 4, 7, 6, 5, 8, 3), or (iii) (2, 1, 6, 7, 4, 3, 8, 5).
after checking that they have no forbidden pairs.
Alternative argument, with argument in a different order. We must alternate
odd and even numbers, otherwise we have adjacent numbers summing to an even
number larger than 2.
The prime sum condition translates to avoiding forbidden odd/even adjacent pairs:
1 − 8, 3 − 6, 5 − 4, 7 − 2, and 7 − 8.
Since 8 cannot be adjacent to either of 1 or 7, 8 must be between 3 and 5.
So up to rotation and reflection, we can assume our circle starts with (3, 8, 5.
There are two possibilities for where to place the other odd numbers:
(i) (3, 8, 5, x, 1, y, 7, z), (ii) (3, 8, 5, x, 7, y, 1, z)
2 cannot be adjacent to 7 so we get:
(i) (3, 8, 5, 2, 1, y, 7, z), or (ii) (3, 8, 5, x, 7, y, 1, 2).
4 cannot be adjacent to 5 so we get:
(i) (3, 8, 5, 2, 1, y, 7, z), or (ii) (3, 8, 5, x, 7, 4, 1, 2).
6 cannot be adjacent to 3 so we get:
(i) (3, 8, 5, 2, 1, 6, 7, z) or (ii) (3, 8, 5, x, 7, 4, 1, 2).
And hence we get two different solutions:
(i) (3, 8, 5, 2, 1, 6, 7, 4), or (ii) (3, 8, 5, 6, 7, 4, 1, 2)
after checking that they have no forbidden pairs.

G. Answer: (1, 2, 3, 8, 5, 6, 7, 4, 9, 10) for instance.


The essential properties for a prime circle for 10 are that the numbers alternate odd
and even, and that the following forbidden pairs do not appear:
1 − 8, 3 − 6, 5 − 4, 5 − 10, 7 − 2, 7 − 8, 9 − 6.
Examples for n = 10, up to rotation, reflection, and swap of 4/10, 3/9 are:
(1, 4/10, 3/9, 4/10, 7, 6, 5, 8, 3/9, 2)
(1, 6, 7, 4/10, 3/9, 4/10, 3/9, 8, 5, 2)
(1, 2, 3/9, 4/10, 7, 6, 5, 8, 3/9, 4/10)
(1, 2, 5, 6, 7, 4/10, 3/9, 8, 3/9, 4/10)
(1, 4/10, 7, 6, 5, 8, 3/9, 2, 3/9, 4/10)
(1, 4/10, 7, 4/10, 3/9, 2, 3/9, 8, 5, 6)
(1, 6, 7, 4/10, 3/9, 2, 5, 8, 3/9, 4/10)
(1, 2, 3/9, 4/10, 7, 4/10, 3/9, 8, 5, 6)
(1, 6, 7, 4/10, 3/9, 8, 5, 2, 3/9, 4/10)
(1, 4/10, 7, 6, 5, 8, 3/9, 4/10, 3/9, 2)
(1, 4/10, 7, 6, 5, 2, 3/9, 8, 3/9, 4/10)
(1, 4/10, 7, 4/10, 3/9, 8, 3/9, 2, 5, 6)

H. The odd/even forbidden pairs are:


1 − 8, 3 − 6, 5 − 4, 5 − 10, 7 − 2, 7 − 8, 9 − 6.
This is really the only condition, except for alternating odd/even.
We see that 3 and 9 have the same forbidden set of neighbours, namely {6}.
So if we swap 3 and 9, we still have a prime circle for 10.
We have to show they are different.
This is clear, since here we fix 8 positions while a rotation fixes none, and a reflection
fixes either 0 or 2.

I. For the same reasons we can swap 4 and 10, as they have the same forbidden set of
neighbours, namely {5}.

J. If we start from a prime circle, we can swap 3/9 and/or 4/10, giving us 4 different
prime circles.
From one of these four, if we perform these same swaps, we always get the same 4
circles. (In other words we can partition the set of solutions into parts of size 4).
Thus the total number is divisible by 4.
[The exact number is 48 but that’s a bit long to compute.]

K. The odd/even forbidden pairs are:


1 − 8, 3 − 6, 3 − 12, 5 − 4, 5 − 10, 7 − 2,
7 − 8, 9 − 6, 9 − 12, 11 − 4 and 11 − 10.
Hence we can swap 3/9, 5/11, 4/10, and 6/12, as each pair has the same forbidden
set of neighbours.
So each solution provides actually 2 · 2 · 2 · 2 = 16 solutions.
[The number of solutions is 16 · 32.]
Examples for n = 12, up to rotation, reflection, and swap of 4/10, 3/9, 6/12, 5/11.
With 3, 9 at distance 2:
(1, 4/10, 3/9, 2, 3/9, 8, 5/11, 6/12, 5/11, 6/12, 7, 4/10)
(1, 6/12, 5/11, 8, 3/9, 4/10, 3/9, 2, 5/11, 6/12, 7, 4/10)
(1, 6/12, 5/11, 2, 5/11, 8, 3/9, 4/10, 3/9, 4/10, 7, 6/12)
(1, 4/10, 7, 4/10, 3/9, 8, 3/9, 2, 5/11, 6/12, 5/11, 6/12)
(1, 6/12, 7, 6/12, 5/11, 8, 5/11, 2, 3/9, 4/10, 3/9, 4/10)
(1, 6/12, 5/11, 8, 5/11, 2, 3/9, 4/10, 3/9, 4/10, 7, 6/12)
(1, 4/10, 3/9, 4/10, 3/9, 8, 5/11, 2, 5/11, 6/12, 7, 6/12)
(1, 4/10, 3/9, 8, 3/9, 4/10, 7, 6/12, 5/11, 6/12, 5/11, 2)
(1, 2, 5/11, 8, 3/9, 4/10, 3/9, 4/10, 7, 6/12, 5/11, 6/12)
(1, 6/12, 5/11, 6/12, 5/11, 8, 3/9, 2, 3/9, 4/10, 7, 4/10)
(1, 2, 5/11, 6/12, 5/11, 8, 3/9, 4/10, 3/9, 4/10, 7, 6/12)
(1, 4/10, 3/9, 8, 3/9, 2, 5/11, 6/12, 5/11, 6/12, 7, 4/10)
(1, 6/12, 5/11, 8, 5/11, 6/12, 7, 4/10, 3/9, 2, 3/9, 4/10)
(1, 6/12, 5/11, 8, 5/11, 6/12, 7, 4/10, 3/9, 4/10, 3/9, 2)
(1, 4/10, 7, 6/12, 5/11, 8, 3/9, 4/10, 3/9, 2, 5/11, 6/12)
(1, 4/10, 3/9, 8, 3/9, 4/10, 7, 6/12, 5/11, 2, 5/11, 6/12)
(1, 6/12, 5/11, 8, 3/9, 4/10, 3/9, 4/10, 7, 6/12, 5/11, 2)
(1, 2, 3/9, 4/10, 3/9, 8, 5/11, 6/12, 5/11, 6/12, 7, 4/10)
(1, 4/10, 3/9, 4/10, 3/9, 8, 5/11, 6/12, 7, 6/12, 5/11, 2)
With 3, 9 at distance 4:
(1, 6/12, 5/11, 8, 3/9, 4/10, 7, 4/10, 3/9, 2, 5/11, 6/12)
(1, 4/10, 3/9, 8, 5/11, 6/12, 7, 6/12, 5/11, 2, 3/9, 4/10)
(1, 2, 3/9, 8, 5/11, 6/12, 5/11, 6/12, 7, 4/10, 3/9, 4/10)
(1, 6/12, 5/11, 6/12, 5/11, 8, 3/9, 4/10, 7, 4/10, 3/9, 2)
(1, 4/10, 3/9, 8, 5/11, 6/12, 5/11, 6/12, 7, 4/10, 3/9, 2)
(1, 4/10, 3/9, 2, 5/11, 8, 3/9, 4/10, 7, 6/12, 5/11, 6/12)
(1, 4/10, 3/9, 8, 5/11, 2, 3/9, 4/10, 7, 6/12, 5/11, 6/12)
With 3, 9 at distance 6:
(1, 6/12, 7, 4/10, 3/9, 8, 5/11, 6/12, 5/11, 2, 3/9, 4/10)
(1, 4/10, 3/9, 8, 5/11, 6/12, 5/11, 2, 3/9, 4/10, 7, 6/12)
(1, 6/12, 5/11, 2, 3/9, 8, 5/11, 6/12, 7, 4/10, 3/9, 4/10)
(1, 4/10, 3/9, 8, 5/11, 6/12, 7, 4/10, 3/9, 2, 5/11, 6/12)
(1, 6/12, 5/11, 8, 3/9, 2, 5/11, 6/12, 7, 4/10, 3/9, 4/10)
(1, 6/12, 5/11, 8, 3/9, 4/10, 7, 6/12, 5/11, 2, 3/9, 4/10)
Western Australian
Junior Mathematics Olympiad 2018
Individual Questions 100 minutes

General instructions: Except possibly for Question 12, each answer in


this part is a positive integer less than 1000. No working is needed for
Questions 1 to 11. Calculators are not permitted. In general, diagrams
are provided to clarify wording only, and are not to scale.
Write your answers for Questions 1–11, and solution for Question 12 on
the front and back, respectively, of the Answer Sheet provided.

1. Jane was designing a board game for her younger brother Sebastian, on
a 15 × 15 board. She decided that the number of non-calamity squares
should be 150 per cent of the number of calamity squares.
How many of the squares did Jane choose to be calamity squares?
[1 mark]

2. Alice went to a restaurant with some friends. The total bill divided
equally among everybody is 30 dollars per person. But it is Alice’s
birthday; so her friends insist she doesn’t pay. Instead, each of Alice’s
friends pays 33 dollars, which exactly covers the total bill.
How many friends are with Alice at the restaurant? [1 mark]

3. In a scrabble tournament, each game eliminates the losing player.


If there are 118 players at the start, how many games will need to be
organised, in order that there is just one winner at the end? [1 mark]

4. A palindromic number is a number that is the same when read from left
to right or from right to left. For instance 23432 is palindromic.
What is the difference of the smallest palindromic number greater than
2018 and the largest palindromic number less than 2018? [1 mark]

5. On an island, 20% of people are redheads and 30% have green eyes.
Among redheads 1 out of 4 has green eyes.
How many percent of the people on the island are neither redheads nor
have green eyes? [1 mark]
6. Consider a regular hexagon of area 100. Construct a new hexagon all of
whose vertices are the midpoints of the edges of the original hexagon.
What is the area of this new hexagon? [2 marks]

7. A rectangular prism has 2 faces of area 36, 2 faces of area 49 and 2 faces
of area 169.
What is the volume of the rectangular prism? [2 marks]

8. The two medians from the acute angles in a right-angle triangle have
length 22 and 31.
What is the length of the hypotenuse?
Note. A median is a line segment that joins a vertex of a triangle with
the midpoint of the opposite side. [3 marks]

9. A number is called special if it is the sum of a non-negative multiple of


4 and a non-negative multiple of 7.
So 11 and 12 are special since 11 = 1 × 4 + 1 × 7 and 12 = 3 × 4 + 0 × 7,
but 13 is not special.
How many special numbers are there in the interval 1 to 1000? [3 marks]

10. Find the number formed by the last three digits of the sum
2! + 4! + 6! + · · · + 2018!,
where for a positive integer n, n! is “n factorial” which is the product
of the integers from 1 to n, i.e. n! = 1 × 2 × · · · × n. [3 marks]

11. A race is organised for two teams of 7 runners each. The runner who
arrives first scores 1 point for their team, the runner who arrives second
scores 2 points for their team, and so on, so that finally, the runner who
arrives last scores 14 points for their team. Two runners never arrive at
exactly the same time. The winning team is the team with the lower
total score.
How many different winning scores are possible? [3 marks]

12. For full marks explain how you found your solution.
How many pairs (x, y) of non-negative integers are solutions of the fol-
lowing equation?
x2 − y 2 = 576
[4 marks]
Western Australian
Junior Mathematics Olympiad 2018
Team Question 45 minutes

General instructions: Calculators are (still) not permitted.

Bays, Shores and Headlands


The shaded area in the diagram below is an island surrounded by white
sea. The small squares are of three types:
Squares that are 34 land, called bays;
Squares that are 21 land, called shores; or
Squares that are 14 land, called headland s.

Bay

Shore

Headland
Note. The small squares are only of the three described types. No small
square is all land or all sea. Also, there is just one island (one can walk from
anywhere to anywhere else on the island) with one continuous coastline;
so there are no lakes within the island.
Post note: the coastline was not intended to go along gridlines.
No working is needed for parts A., B., C., but for other parts a full
explanation of how you found your answer must be given.

A. How many bays, how many shores and how many headlands are there
on the island in the diagram above?

B. Given the area of a small square is 1, what is the area of the island in
the diagram above?
C. The island on the given diagram is on a 6 × 7 grid.
Where possible, construct an island on each of the 3 × 4, 4 × 4 and 4 × 5
grids, attached to the question paper.
Remember: every small square on the grid must be a bay, a shore or a
headland.
Note. It is only necessary to draw the island coastline. It is not ne-
cessary to do any shading.

D. Can you make an island on a 3 × 3 grid?


Remember: every small square on the grid must be a bay, a shore or a
headland.

E. For some values of m and n, it is possible to have an island on an m × n


grid; for others, it is not possible.
Find a condition that says for which values of m and n, there is at least
one way to draw an island on an m × n grid.
How do you know your condition holds for all grids?

F. Let b, s and h be the numbers of bays, shores and headlands, respec-


tively on an island.
Prove h = b + 4.

G. Show that s must be even.

H. Show that: if m ≥ 4 and n ≥ 4 then s ≥ 4.

I. Find a formula for the area of an island in terms of m and n.

J. Find all possible values of s for a 4 × 4 grid.


Individual Questions Solutions
1. Answer: 90. The board has 152 = 225 squares. Let x be the number of
calamity squares. Then
225 = x + 1.5x
= 2.5x
225
∴x=
2.5
2 × 225
=
5
= 90.
So Jane chose 90 of the squares to be calamity squares.

2. Answer: 10. Let x be the number of friends that went with Alice to the
restaurant. Then
30(x + 1) = 33x
30 = 3x
x = 10.

3. Answer: 117. Each game eliminates 1 player; 117 players need to be


eliminated. So 117 games are required.

4. Answer: 110. The required difference is 2112 − 2002 = 110.

5. Answer: 55. Assume there are 100 people on the island.


Among the 20 redheads, 41 of them have green eyes.
So 5 people are redheads and have green eyes.
In all, 30 people have green eyes.
So 25 have green eyes, but are not redheads.
Overall, 80 = 100 − 20 are non-redheads. So,
80 − 25 = 55
are neither redheaded nor green-eyed.
Thus, the proportion of islanders who are neither redheads nor have
green eyes is 55%.
6. Answer: 75. From vertices of the original
and new hexagons on one half-side of the
original hexagon, draw a circumradius of
each of the original and new hexagons.
The circumradii and the half-side form
a half equilateral triangle with the ratio
of the new hexagon circumradius
√ to the
original circumradius being 3 : 2.
Since the length measures√ of the two
hexagons are in the ratio 3 : 2, their
area measures are in the ratio 3 : 4.
Hence, the area of the new hexagon is
3
4 · 100 = 75.

7. Answer: 546. Let the dimensions of the prism be `, b, h. Then we have

`b = 36, `h = 49, bh = 169.

Now the volume is `bh, and observe that

(`bh)2 = `b · `h · bh
= 36 · 49 · 169
= (6 · 7 · 13)2
= 5462
`bh = 546.

So the required volume is 546.

8. Answer: 34. Let the legs of the right-angle triangle be 2x and 2y. Then
we are given that

x2 + (2y)2 = 222
(2x)2 + y 2 = 312
p
and we need to find (2x)2 + (2y)2 . Adding the two equations, we have
5x2 + 5y 2 = 222 + 312
4x2 + 4y 2 = 45 (222 + 312 )
= 54 (484 + 961)
= 45 · 1445
= 4 · 289
= 22 · 172
p
(2x)2 + (2y)2 = 2 · 17 = 34.

9. Answer: 991. The key observation is that since we have the 4 consecu-
tive special numbers,
18 = 1 × 4 + 2 × 7
19 = 3 × 4 + 1 × 7
20 = 5 × 4 + 0 × 7
21 = 0 × 4 + 3 × 7,
all larger numbers are special, because they can be obtained by adding
a multiple of 4 to one of 18, 19, 20 or 21.
This leaves determining which numbers less than 18 are special. The
restriction,
n × 4 + m × 7 < 18
implies m ≤ 2. Checking, we find the special numbers less than 18 are:
for m = 0: 4, 8, 12 and 16,
for m = 1: 7, 11 and 15,
for m = 2: 14.
Since, in this search, we did not encounter 1, 2, 3, 5, 6, 9, 10, 13 or 17 (9 of
them), these numbers are not special, and they are the only non-special
numbers less than 1000.
Thus, in all, there are 1000 − 9 = 991 special numbers in the interval 1
to 1000.

10. Answer: 666. Since any of the factorials from 16! on, have among their
factors 4, 5, 10 and 15, whose product is 3000, all the factorials from
16! on, end in three zeros.
So, we need only consider the sum,

2! + 4! + 6! + 8! + 10! + 12! + 14! = 2


+ 24
+ 720
+ . . . 320
+ . . . 800
+ . . . 600
+ . . . 200
= . . . 666

So the number formed by the last three digits of 2! + 4! + 6! + · · · + 2018!


is 666.

11. Answer: 25. The minimum score for a team is 1 + 2 + · · · + 7 and the
maximum score for a team is 8 + 9 + · · · + 14. To see that all scores
between these two scores are possible, we need only exhibit one possible
7-tuple that sums to each score:

(1, 2, 3, 4, 5, 6, 7), (1, 2, 3, 4, 5, 6, 8), . . . , (1, 2, 3, 4, 5, 6, 14),


(1, 2, 3, 4, 5, 7, 14), . . . , (1, 2, 3, 4, 5, 13, 14),
(1, 2, 3, 4, 6, 13, 14), . . . , (1, 2, 3, 4, 12, 13, 14),
(1, 2, 3, 5, 12, 13, 14), . . . , (1, 2, 3, 11, 12, 13, 14),
(1, 2, 4, 11, 12, 13, 14), . . . , (1, 2, 10, 11, 12, 13, 14),
(1, 3, 10, 11, 12, 13, 14), . . . , (1, 9, 10, 11, 12, 13, 14),
(2, 9, 10, 11, 12, 13, 14), . . . , (8, 9, 10, 11, 12, 13, 14).

Observe that for any pair that are adjacent sequentially, just one entry
differs by 1 (the entry changing on each row is underlined). There are
1 + 7 × 7 = 50 possible scores, and by symmetry half of these, i.e. 25
are winning scores.
Note. Since 1 + 2 + · · · + 14 is odd (being a sum of 7 consecutive pairs
whose sum is odd), the teams cannot draw.
Alternative method. The total of all individual scores is

1 + 2 + · · · + 14 = (14 × 15)/2 = 105.


So if one team score is x, then the other team score is 105 − x. If x is
the winning score, then
x < 105 − x
2x < 105
x < 52.5.
Thus, the minimum winning score is 1 + 2 + · · · + 7 = 28 and the
maximum winning score is 52, if it is attainable. Indeed, the argument
in the previous method shows that all scores from 28 to 52 are attainable.
So the number of possible winning scores is
52 − 27 = 25.

12. Answer: 8. We have


26 32 = 242 = 576 = x2 − y 2 = (x + y)(x − y).
We essentially need to discover the number of ways we can partition the
factors of 26 32 between x + y and x − y.
Now x + y and x − y have the same parity, and since 26 32 is even, at
least one and hence both x + y and x − y are even.
So two of the 2s are allocated, and we are left to find the number of
ways we can partition the factors of 24 32 between 12 (x + y) and 21 (x − y).
The number of positive integer divisors of 24 32 , is (4 + 1)(2 + 1) = 15,
since those divisors are 2k 3` , where k ∈ {0, 1, 2, 3, 4} (4 + 1 possibilities)
and ` ∈ {0, 1, 2} (2 + 1 possibilities).
Each divisor of 24 32 , leads to a way of writing
24 32 = 2k 3` · 24−k 32−` ,
as the product of two factors. Most of these factorisations are written
twice, once with the factor on the left and once with the factor on the
right. But 12 = 22 3 is partnered with itself in one factorisation, leading
to the solution,
1 1
2 (x − y) = 2 (x + y) = 12,
in which case (x, y) = (24, 0). The remaining 14 divisors lead to 7 pairs
of non-equal factors of 24 32 , the smaller being a = 21 (x − y) and the
larger being b = 21 (x + y). Since a, b are positive integers and a ≤ b,
each leads to a solution for (x, y), where x = a + b and y = b − a. So,
in all, there are 8 solutions.
Note. Since, it may be of some interest, we explicitly list the solutions
for (x, y) in the following table, where 2k 3` ≤ 12, since a ≤ 12 ≤ b.
k ` a = 2k 3` b = 24−k 32−` x = a + b y = b − a
0 0 1 144 145 143
1 0 2 72 74 70
2 0 4 36 40 32
3 0 8 18 26 10
0 1 3 48 51 45
1 1 6 24 30 18
2 1 12 12 24 0
0 2 9 16 25 7
Alternative method. We have
26 32 = 242 = 576 = x2 − y 2 = (x + y)(x − y).
We essentially need to discover the number of ways we can partition the
factors of 26 32 between x + y and x − y.
Now x + y and x − y have the same parity, and since 26 32 is even, at
least one and hence both x + y and x − y are even.
Now 26 32 has (6 + 1)(2 + 1) = 21 positive integer divisors. Removing
the odd divisors 1, 3, 9 and their respective partners 576, 192, 64, leaves
15 divisors. There are an odd number of divisors since 576 is a perfect
square. Each of the 7 even divisors less than 24 is partnered with an
even divisor greater than 24, and 24 is partnered with itself. Thus, their
are 8 possibilities for the factor pairs (x − y, x + y) of 576.
Let A = x−y and B = x+y. Then since A and B are even, x = 12 (A+B)
and y = 12 (B − A) are integers, and hence the 8 possibilities for (A, B)
each lead to a solution pair (x, y). So, in all, there are 8 solutions.
Team Question Solutions
Bays, Shores and Headlands

A. There are 15 bays, 8 shores and 19 headlands.

B. Let b, s and h be the numbers of bays, shores and headlands, respec-


tively. We have b = 15, s = 8 and h = 19. So
3
Area = · 15 + 21 · 8 + 14 · 19
4
3 · 15 + 2 · 8 + 1 · 19 80
= = = 20.
4 4

C. There are many ways to do such islands. One example for each grid is
shown.
D. Answer: No. Since each corner small square must be a headland, each
middle edge small square must be a shore, leaving the centre small
square as wholly land, i.e. the centre small square is not a bay, shore or
headland (a contradiction). So, no, it is not possible to have an island
on a 3 × 3 grid.

E. Answer: mn must be even, with m ≥ 2 and n ≥ 2. First observe that


the smallest island is one of 4 headlands on a 2 × 2 grid. So m ≥ 2 and
n ≥ 2. Now colour the small squares black or white in a chessboard
fashion. Now suppose the grid has an island drawn on it, and go round
the coastline of the island once. As we pass from one small square
to the next, the colour alternates. When we get back to where we
started, we must have travelled through equal numbers of small black
and white squares, i.e. an even number of small squares, and if it’s a
valid island we must have used up all the small squares of the grid.
Thus the number of small squares in the grid is even, and so mn is
even.
In order to show that all even grids, with m ≥ 2 and n ≥ 2, allow
an island to be drawn, we exhibit a construction. Suppose, without
loss of generality, that the number of rows is even. Firstly, through the
middle of each pair of rows we construct, what we will call a caterpillar.
Then, if we join these caterpillars at the same end we form a comb (see
diagrams below). This construction shows that on any m × n grid for
which mn is even, and m ≥ 2 and n ≥ 2, we can draw an island.
(Other constructions are possible. Below right we show the caterpillars
connected at alternating ends to form a snake, and through near the
middle to form a tree.)

caterpillar

snake

comb tree
Thus we see that for an m × n grid to support an island, mn must be
even, and whenever mn is even, the m × n grid supports an island.
F. Imagine sailing around the coast in a clockwise direction. At each
headland you turn clockwise through 90◦ and at each bay you turn
anticlockwise through 90◦ (which is the same as −90◦ clockwise). At
shores you don’t turn at all. However, when you get back to the place
where you started you will have turned through a net 360◦ . This means
that
90h − 90b = 360,
which reduces to h = b + 4.

G. The total number of small squares in a grid is mn which must equal


b + s + h. Using the formula from F., this becomes
mn = b + s + h
=b+s+b+4
= 2b + s + 4
∴ s = mn − 2b − 4. (∗)
Since mn is even by E., the right hand side of (∗) is even, and hence s
is even.

H. Consider the 4 small squares at a corner. Of these, call the small square
on the corner C; let the two small squares sharing an edge with C, be
X and Y ; and let A be the small square sharing an edge with each of
X and Y , and sharing a vertex with C. Necessarily, C is a headland,
and each of X and Y must either be a headland or a shore.
Suppose both X and Y are headlands. Then A must also be a headland.
And so we get a small island on the small squares X, C, Y, A, not going
through all mn small squares of the grid, which is contradiction.
So at least one of X and Y must be a shore.
Since m ≥ 4 and n ≥ 4, the 4 sets of 4 small squares at the corners are
disjoint, and so there are at least 4 shores, i.e. s ≥ 4.
Note. In a 2 × 2 grid we get s = 0, and in a 2 × 3 grid we get s = 2.

I. Answer: 21 mn − 1. Let the area of the island be A. Then we have


3b s h
A= + + .
4 2 4
The number of small squares in the grid is mn and this must equal
h + b + s, which gives us s = mn − h − b. On substituting this into the
expression for A, we get
3b mn − h − b h
A= + +
4 2 4
mn h b
= − +
2 4 4
mn b + 4 b
= − + , using F.
2 4 4
mn
= − 1.
2

J. Answer: s must be 4 or 8. By G. and H., and the fact that h ≥ 4,


since the four corner small squares are headlands, the candidates for s
are 4, 6, 8, 10, 12.
Consider the 8 non-corner edge small squares (call them E-squares).
These must all be either shores or headlands. We consider three cases.
Case 1: all the E-squares are shores. Then the interior small squares
are just land, and hence this island is not admissible. This
rules out s = 12.
Case 2: one of the E-squares is a headland. Then the adjacent
E-square is also a headland. Thus h ≥ 6, b ≥ 2 and hence
s ≤ 8. So s = 10 is also ruled out.
Among the 8 E-squares, there is necessarily an even num-
ber of headlands.
Case 3: s = 6. Then h = 7 and b = 3, since b + h = mn − s = 10
and h = b + 4. Then, we can only have two headlands
among the E-squares, since there is an even number at
least 2. We must now fit one headland and 3 bays among
the 4 interior squares, which is impossible. So s = 6 is
ruled out.
This leaves s = 4 and s = 8. The following diagrams show that these
are indeed possible.

s = 8, h = 6, b = 2 s = 4, h = 8, b = 4
Alternative method. Each corner small square is a headland. Be-
tween two adjacent corner small squares there are either two headlands
or two shores. If there are shores between every pair of adjacent cor-
ners one can’t get into the interior of the island; so there’s at least
one pair with headlands in between. Without loss of generality, this
is the bottom pair. The left and right side edge small squares (called
E-squares in the other solution) between the headlands are then forced
to be shores. This leaves two possibilities for the top two E-squares
resulting in the 2 islands shown. Thus, up to rotation, there are only
two possible islands on a 4 × 4 grid and so for a 4 × 4, s can only be 8
or 4.

You might also like